SELECTIVITY SELECTIVITY Science 1983, 219, 245 Chemoselectivity preferential reactivity of one functional group (FG) over another - Chemoselective reduction of C=C over C=O: H2, Pd/C

O

O

- Chemoselective reduction of C=O over C=C: O

OH

NaBH4

O

O

+

NaBH4, CeCl3

OH only

- Epoxidation: MCPBA

+ OH

OH

O

O (2 : 1)

VO(acac)2, tBuOOH OH

OH

O

exclusively

Regioselectivity - Hydration of C=C: 1) B2H6 2) H2O2, NaOH

OH

R

R

OH R 1) Hg(OAc)2, H2O 2) NaBH4

- Friedel-Crafts Reaction: O RCOCl, AlCl3

R

+

R O

O SiMe3

RCOCl, AlCl3

R

OH

1

SELECTIVITY - Diels-Alder Reaction: R

R

O

O

R

+

+ O major O

R

minor O

R

+

+

R

O minor

major O

O

+ O

H

OAc

O

O

H

OAc

O

O

OAc

O

O

OAc

OAc

Raney Ni, H2

+ O

H

SPh

O

O

H

O

O

SPh

H

O

Change in mechanism: SPh PhSH, H+ R

R

R

SPh

PhSH, (PhCO2)2

Stereochemistry: Relative stereochemistry: Stereochemical relationship between two or more stereogenic centers within a molecule

H

H

HO cholesterol

H enantiomers same relative stereochemistry

H OH

syn: on the same side ( cis) anti: on the opposite side (trans) - differences in relative stereochemistry lead to diastereomers. Diastereomers= stereoisomers which are not mirror images; usually have different physical properties

2

SELECTIVITY Absolute Stereochemistry: Absolute stereochemical assignment of each stereocenter (R vs S) Cahn-Ingold-Prelog Convention (sequence rules) - differences in absolute stereochemistry (of all stereocenters within the molecule) leads to enantiomers. - Reactions can "create" stereocenters O Ph

MeMgBr

HO

H

Ph

MeMgBr

CH3 H

H 3C

OH

Ph

H

O

enantiomers (racemic product)

Ph H HO

CH3

Ph

H

MeMgBr

Diastereomeric transition states- not necessarily equal in energy Me

Me

Me

N

O

Ph

Zn

O

CH3 CH3 Zn

Me N

O

O CH3 CH3

H

Zn

H 3C

H

Zn Ph

H 3C

HO Ph

CH3

HO

CH3

H

H

Ph

Diastereoselectivity CH3MgBr Ph

CH3

Ph

CHO

HO

+

CH3

Ph

H

H OH anti

syn

Diastereomers

Cram Model (Cram's Rule): empirical O O M S

R

O

S

M

H 3C Nu

H

CH3MgBr

CH3MgBr

L R

L

favored

H Ph

3

SELECTIVITY

4

Felkin-Ahn Model S O

O M L

R

L

Nu

Nu

M R

S

disfavored

favored

Chelation Control Mode M OR O

O R S

M

HO

OR

CH3MgBr

CH3MgBr

M

S

favored

Nu

M

R

S

OR

R

HO TBSO

O

TBSO

MgBr

relative stereochemical control

H O

H O

OBn

OBn

Stereospecific Stereochemictry of the product is related to the reactant in a mechanistically defined manner; no other stereochemical outcome is mechanistically possible. i.e.; SN2 reaction- inversion of configuration is required Br2

H

H 3C

Br

Br2

H

meso

H CH3

Br

CH3 Br

+

H CH3

Br

CH3 H

Br H

CH3 Br

enantiomers (racemic)

Stereoselective When more than one stereochemical outcome is possible, but one is formed in excess (even if that excess is 100:0). CH3

H2, Pd/C

H H α-pinene O

only isomer O

O

H

H

O

H

+

CH3 H not observed O

O

LDA, CH3I N

O S

N

O

+

S

N

O S

(96 : 4) Diasteromers

Diastereoselective Enantiospecific

OXIDATIONS Oxidations Carey & Sundberg: Chapter 12 problems: 1a,c,e,g,n,o,q; 2a,b,c,f,g,j,k; 5; 9 a,c,d,e,f,l,m,n; 13 Smith: Chapter 3 March: Chapter 19 I. Metal Based Reagents 1. Chromium Reagents 2. Manganese Rgts. 3. Silver 4. Ruthenium 5. other metals II Non-Metal Based Reagents 1. Activated DMSO 2. Peroxides and Peracids 3. Oxygen/ ozone 4. others III. Epoxidations Metal Based Reagents Chromium Reagents - Cr(VI) based - exact stucture depends on solvent and pH - Mechanism: formation of chromate ester intermediate Westheimer et al. Chem Rev. 1949, 45, 419 JACS 1951, 73, 65. HO R2CH-OH

HCrO4

-

R R

H+

Cr

C O

O-

R

O-

O

R

+ HCrO3- +

H+

H + H2O

Jones Reagent (H 2CrO4, H2Cr2O7, K2Cr2O7) J. Chem. Soc. 1946 39 Org. Syn. Col. Vol. V, 1973, 310. - CrO3 + H2O → H2CrO4 (aqueous solution) K2Cr2O7 + K2SO4 - Cr(VI) → Cr(III) (black)

(green)

- 2°- alcohols are oxidized to ketones R2CH-OH

Jones reagent

R

acetone

R

O

- saturated 1° alcohols are oxidized to carboxylic acids. Jones reagent RCH2-OH

acetone

O R

hydration H

HO OH

Jones reagent

R

acetone

H

O R

OH

- Acidic media!! Not a good method for H+ sensitive groups and compounds

5

OXIDATIONS 1) Jones, acetone

SePh OH

6

SePh CO 2CH 3

2) CH2N2

Me 3Si

Me 3Si JACS 1982, 104, 5558

H17C 8

H17C 8

O

O

O

OH Jones acetone

O

O

O

JACS 1975, 97, 2870

O

Collins Oxidation (CrO3•2pyridine) TL 1969, 3363 - CrO3 (anhydrous) + pyridine (anhydrous) → CrO 3•2pyridine↓ - 1° and 2° alcohols are oxidized to aldehydes and ketones in non-aqueous solution (CH 2Cl2) without over-oxidation - Collins reagent can be prepared and isolated or generated in situ. Isolation of the reagent often leads to improved yields. - Useful for the oxidation of H+ sensitive cmpds. - not particularly basic or acidic - must use a large excess of the rgt.

CrO3•(C 5H5N)2 OH ArO

H

CH 2Cl 2

O

O ArO

O

JACS 1969, 91, 44318.

O O

CrO3 catalyzed (1-2 mol % oxidation with NaIO6 (2.5 equiv) as the reozidant in wet aceteonitrile. oxidized primary alcohols to carboxylic acids. Tetrahedron Lett. 1998, 39, 5323. Pyridinium Chlorochromate (PCC, Corey-Suggs Oxidation) TL 1975 2647 Synthesis 1982, 245 (review) CrO3 + 6M HCl + pyridine → pyH+CrO3 Cl- ↓ - Reagent can be used in close to stoichiometric amounts w/ substrate - PCC is slighly acidic but can be buffered w/ NaOAc PCC, CH 2Cl 2 OHC HO

JACS 1977, 99, 3864. O

O O

PCC, CH 2Cl 2 OH

O

CHO TL, 1975, 2647

OXIDATIONS - Oxidative Rearrangements Me

OH Me

PCC, CH 2Cl 2

JOC 1977, 42, 682 O

Me

Me PCC, CH 2Cl 2

JOC 1976, 41, 380

OH

O

- Oxidation of Active Methylene Groups PCC, CH 2Cl 2 O

O

O

JOC 1984, 49, 1647

PCC, CH 2Cl 2 O

O O

- PCC/Pyrazole PCC/ 3,5-Dimethylpyrazole JOC 1984, 49, 550. NH

NH

N

N

- selective oxidation of allylic alcohols OH OH PCC, CH 2Cl 2 H

3,5-dimethyl pyrazole

H

HO

H O

H (87%)

Pyridinium Dichromate (PDC, Corey-Schmidt Oxidation) TL 1979, 399 - Na2Cr2O7•2H2O + HCl + pyridine → (C5H5N)2CrO7 ↓ PDC

PDC CHO

CH 2Cl 2

OH

DMF

1° alcohol

-allylic alcohols are oxidized to α,β-unsaturated aldehydes

CO 2H

7

OXIDATIONS - Supported Reagents Comprehensive Organic Synthesis 1991, 7, 839. PCC on alumina : Synthesis 1980, 223. - improved yields due to simplified work-up. PCC on polyvinylpyridine : JOC, 1978, 43, 2618. CH 2 CH cross-link N

CH 2 CH

R2CH-OH

R2C=O

CH 2 CH

CrO3, HCl N

N Cr(III)

N Cr(VI)O3 •HCl

8

partially spent reagent

to remove Cr(III) 1) HCl wash 2) KOH wash 3) H2O wash

CrO3/Et2O/CH2Cl2/Celite Synthesis 1979, 815. - CrO3 in non-aqueous media does not oxidized alcohols - CrO3 in 1:3 Et2O/CH2Cl2/celite will oxidized alcohols to ketone and aldehydes C 8H17

C 8H17 CrO3 Et2O/CH 2Cl 2/celite (69%)

HO

Synthesis 1979, 815

O

H2CrO7 on Silica - 10% CrO3 to SiO2 - 2-3g H2CrO3/SiO2 to mole of R-OH - ether is the solvent of choice Manganese Reagents Potassium Permanganate KMnO4/18-Crown-6 JACS 1972 94, 4024.

(purple benzene)

O O

O K+

O

MnO 4O

O

- 1° alcohols and aldehydes are oxidized to carboxylic acids - 1:1 dicyclohexyl-18-C-6 and KMnO4 in benzene at 25°C gives a clear purple solution as high as 0.06M in KMnO4. O JACS 1972, 94, 4024 CO 2H CHO Synthesis 1984, 43 CL 1979, 443 CHO

OXIDATIONS

9

Sodium Permanganate TL 1981, 1655 - heterogeneous reaction in benzene - 1° alcohols are oxidized to acids - 2° alcohols are oxidized to ketones - multiple bonds are not oxidized Barium Permanganate (BaMnO4) TL 1978, 839. - Oxidation if 1° and 2° alcohols to aldehydes and ketones- No over oxidation - Multiple bonds are not oxidized - similar in reactivity to MnO2 Barium Manganate BCSJ 1983, 56, 914 Manganese Dioxide Review: Synthesis 1976, 65, 133 - Selective oxidation of α,β-unsatutrated (allylic, benzylic, acetylenic) alcohols. - Activity of MnO2 depends on method of preparation and choice of solvent - cis & trans allylic alcohols are oxidized at the same rate without isomerization of the double bond. OH

OH

HO

HO MnO 2, CHCl3

J. Chem. Soc. 1953, 2189 JACS 1955, 77, 4145.

(62%) O

HO

- oxidation of 1° allylic alcohols to α,β-unsaturated esters OH

MnO2, ROH, NaCN CO 2R

OH

CO 2Me

JACS1968, 90, 5616. 5618

MnO 2, Hexanes MeOH, NaCN

Manganese (III) Acetate α-hydroxylation of enones Synthesis 1990, 1119 TL 1984 25, 5839 O

O Mn(OAc)3, AcOH

AcO

Ruthenium Reagents Ruthenium Tetroxide - effective for the conversion of 1° alcohols to RCO2H and 2° alcohols to ketones - oxidizes multiple bonds and 1,2-diols.

OXIDATIONS Ph

OH O

H

O JOC 1981, 46, 3936

RuO4, NaIO4

OH CH 3

CO 2H

Ph

CCl 4, H2O, CH3CN

OH Ph

RuO4, NaIO4

10

Ph

CCl 4, H2O, CH3CN

CO 2H

H

96% ee

CH 3

94%ee

HO RuO2, NaIO4

O TL 1970, 4003

CCl 4, H2O

O

O

O

O

Tetra-n-propylammonium Perruthenate (TPAP, nPr4N+ RuO4-) Aldrichimica Acta 1990, 23, 13. Synthesis 1994, 639 - mild oxidation of alcohols to ketones and aldehydes without over oxidation OH

O TPAP MeO 2C

MeO 2C

OSiMe 2tBu

OSiMe 2tBu

O N+ -O Me

TL 1989, 30, 433

(Ph3P)4RuO2Cl3 RuO2(bipy)Cl2 - oxidizes a wide range of 1°- and 2°-alcohols to aldehydes and ketones without oxidation of multiple bonds. OH

CHO CHO OH

JCS P1 1984, 681.

H

H

Ba[Ru(OH)2O3] -oxidizes only the most reactive alcohols (benzylic and allylic) (Ph3P)3RuCl2 + Me3SiO-OSiMe3 - oxidation of benzylic and allylic alcohols TL 1983, 24, 2185. Silver Reagents Ag2CO3 ( Fetizon Oxidation) also Ag2CO3/celite - oxidation of only the most reactive hydroxyl O OH

Synthesis 1979, 401 O

Ag 2CO 3

O

OH

OH O

O

OH

O

OH

Ag 2CO 3, C 6H6

O O O

JACS 1981, 103, 1864. mechanism: TL 1972, 4445.

OXIDATIONS - Oxidation of 2° alcohol over a 1° alcohol OH

OH

Ag2CO3, Celite

OH

JCS,CC 1969, 1102

(80%)

O

Silver Oxide (AgO2) - mild oxidation of aldehyde to carboxylic acids AgO 2, NaOH RCHO

CHO

RCO 2H CO 2H

AgO 2

JACS 1982, 104, 5557 Ph Ph

Prevost Reaction Ag(PhCO2)2, I2 Ag(PhCO 2)2, I2

AcO

OAc

AcOH

AcO

Ag(PhCO 2)2, I2

OH

AcOH, H 2O

Other Metal Based Oxidations Osmium Tetroxide OsO 4 review: Chem. Rev. 1980, 80, 187. -cis hydroxylation of olefins old mechanism: O

OH

Os O O

OH

O

OsO 4, NMO

osmate ester intermediate

cis stereochemistry

- use of R 3N-O as a reoxidant TL 1976, 1973. OsO 4, NMO

O O

O

OH

O

OH

OH OH

TL 1983, 24, 2943, 3947 Stereoselectivity:

OsO 4 R3

R2 RO

H

R4

OsO 4, NMO

HO H R2 HO R3 RO H R4

11

OXIDATIONS - new mechanism: reaction is accelerated in the presences of an 3° amine R1

R1

O

O O

R2

Os O

O

[2+2]

R3N

R1 R2

O Os

O

12

O

Os O

O

O

R2

O NR3

[O] [3+2]

OsO2

R1 O O

R2

O

[O] hydrolysis

R1

Os O

R2

+

O

HO

OH

OsO4

- Oxidative cleavage of olefins to carboxylic acids. JOC 1956, 21, 478. - Oxidative cleavage of olefins to ketones & aldehydes. OH CHO CHO

OH OsO 4, NMO

O

O

NaIO4

OH

O

H2O

O O

O

O

O

O

OAc

O

OAc

OAc

JACS 1984, 105, 6755.

Substrate directed hydroxylations: -by hydroxyl groups

Chem. Rev. 1993, 93, 1307 HO

OsO4, pyridine

O

HO

HO O

HO

+

O

HO HO

HO 3:1 HO

OsO4, pyridine

O

HO O

TMSO TMSO

HO

CH3

HO

CH3 OH

OsO4, Et2O

HO OH

CH3

CH3

+

OH CH3

(86 : 14)

- by amides AcO

AcO OH

MeS

OsO4

MeS OH

HN

O OAc

CH3 OH

HN

O OAc

OXIDATIONS - by sulfoxides ••

••

OMe

O

OsO4

S

OMe OH

O S

OH 1) OsO4 2) Ac2O

S HN

OAc

(2 : 1)

••

••

O

13

O S

O

AcO

O

HN

(20 : 1)

- by sulfoximines O Ph S

O Ph S

OH

MeN

MeN

OsO4, R3NO

O

OH ∆

OH

OH OH

OH CH3 Raney nickel H 3C

OH OH OH CH3

- By nitro groups PhO2S

PhO2S

1) OsO4

N NHR

N

+

2) acetone, H

N NHR

N O2N

O2N

N N

N

O

N

O N

N HO

HO

NHR

N

NHR

N

N N

N

O

N

O

- OsO4 bis-hydroxylation favors electon rich C=C. OsO4 X

OH OH

X

+ OH OH

X= OH = OMe = OAc = NHSO2R

- Ligand effect:

80 : 20 98 : 2 99 : 1 60 : 40

OsO4 OH

K3Fe(CN)6, K2CO3 MeSO2NH2, tBuOH/H2O

OH OH

OsO4 (no ligand) Quinuclidine DHQD-PHAL

X

4:1 9:1 > 49 : 1

+

X

(directing effect ?) (directing effect ?)

OH OH

OH

OXIDATIONS Chem. Rev. 1994, 94, 2483.

Sharpless Asymmetric Dihydroxylation (AD) - Ligand pair are really diastereomers!!

14

dihydroquinidine ester

N "HO

Ar

OH"

H

H R3

OR'

R2

R3 OH

0.2-0.4% OsO4

R2

R1

acetone, H 2O, MNO

80-95 % yield 20-80 % ee

OH

R1 H OR' "HO

Ar

OH"

MeO Ar =

N

dihydroquinine ester

N R'= p-chlorobenzoyl

Mechanism of AD: L HO

OH

O O H 2O

O

O

Os

O

O O O

First Cycle (high enantioselectivity)

O

Os

O O

Second Cycle (low enantioselectivity)

[O]

[O]

O O

Os

O

O

L

Os O

L

O O

O O

Os

O

O O

O R 3N

HO

OH H 2O, L

- K3Fe(CN)6 as a reoxidant gives higher ee's- eliminates second cycle TL 1990, 31, 2999. - Sulfonamide effect: addition of MeSO2NH2 enhances hydrolysis of Os(VI) glycolate (accelerates reaction) - New phthalazine (PHAL) ligand's give higher ee's N

Et Et O

H

Et

N

N N

N N

O

H

O

H

OMe

MeO

O

H

MeO N

OMe

N N

N (DHQ)2-PHAL

(DHQD)2-PHAL JOC 1992, 57, 2768.

Et

N N

OXIDATIONS

15

- Other second generation ligands N

Et Et O

H MeO

Et N

Ph O

N

N

H

N

OMe

Ph

N

N O

H

O

OMe

N

N

PYR

IND

Proposed catalyst structure: O

H

O O

Os

N

MeO

N

Os

"Bystander quinoline (side wall)

Asymmetric Binding Cleft

O

N H

H

N

N

O

N

N

N

O

Phthalazine Floor

OMe

OMe

OMe O

Corey Model: JACS 1996, 118, 319 Enzyme like binding pocket; [3+2] addition of OsO4 to olefin.

N

O O

Os O N O

O

N

H

N N O

O

N

DHQL

Rs

RM

RL

H

DHQ

RL large and flat, i.e Aromatics work particularly well

OXIDATIONS Olefin

Preferred Ligand

ee's

PYR, PHAL

30 - 97 %

PHAL

70 - 97 %

IND

20 - 80 %

PHAL

90 - 99.8 %

PHAL

90 - 99 %

PHAL, PYR + MeSO2NH2

20 - 97 %

R1 R2 R1

R1 R2

R2

R1

R2 R3

R1 H R2

R3

R1 R4

"AD-mixes" commercially available pre-mix solutions of Os, ligand and reoxidant AD-mix α (DHQ)2PHAL, K 3Fe(CN)6, K2CO3, K2OsO4 (0.4 MOL % Os to C=C) AD-mix β (DHQD)2PHAL, K 3Fe(CN)6, K2CO3, K2OsO4 O HO O Campthothecin

N N O OMe N

OMe

OMe AD (DHQD)2PYR

O

N

N

O

94 % ee

O O

OH OH

OH

- Kinetic resolution (not as good as Sharpless asymmetric epoxidation) H Ph tBu

Ph H tBu

H Ph

H Ph

AD mix α 30% conversion

Ph H

OH OH tBu

tBu

olefins with axial dissymmetry

H

Ph

+

OH OH

+

tBu (4 : 1)

tBu enriched

16

OXIDATIONS 17 Asymmetric Aminohydroxylation TL 1998, 39, 2507; ACIEE 1996, 25, 2818, 2813, preparation of α-aminoalcohols from olefin. Syn addition as with the dihydroxylation regiochemistry can be a problem O Ph

O

N Na

CO2Me

Ph

O

OH

Cl Ph

O

NH

+ CO2Me

Ph

K2OsO6H4 (cat) Ligand

CO2Me

Ph N

OH

O

Ph

O

Ligand= PHAL AQN

4:1 1:4

Molybdenum Reagents MoOPH [MoO5•pyridine (HMPA)] JOC 1978, 43, 188. - α-hydroxylation of ketone, ester and lactone enolates. O

OR'

R

O

+

Mo O L

R

O

H R

R

Pd(OAc) 2, CH 3CN, 80° C

HO

H R

O

H R

- CO 2

O

O Pd

-

TL 1984, 25, 2791 Tetrahedron 1987, 43, 3903

O

OH

2

HO

CO

H OH JACS 1989, 111, 8039.

Pd2(DBA) 3•CHCl 3, CH 3CN, 80° C

OH

O

R

R

Pd(0) O

H

H

(Tsuji Oxidation)

O

O 2 CO

OH

R' OH

L

Palladium Reagents Pd(0) catalyzed Dehydrogenation (oxidation) of Allyl Carbonates Tetrahedron 1986, 42, 4361 R

O

THF, -78°C

O

H

O

O

Oxidation of silylenol ethers and enol carbonates to enones O

OTMS

Pd(OAc) 2, CH 3CN

O

O O

OTIPS Ph

O

Pd(OAc) 2, CH 3CN

(NH 4)2Ce(NO 3)6 DMF, 0°C

O

O Ph

TL 1995, 36, 3985

R

Oppenauer Oxidation

OXIDATIONS Organic reactions 1951, 6, 207

Synthesis 1994, 1007 OiPr +O Al

R1R2CHOH (CH3)2C=O

OiPr

OiPr +O Al O OiPr

H R1

R2

O R1

18

+ Al(OiPr)3 R2

Nickel Peroxide Chem Rev. 1975, 75, 491 Thallium Nitrate (TNN, Tl(NO 3)3•3H2O Pure Appl. Chem. 1875, 43, 463. Lead Tetraacetrate Pb(OAc)4 Oxidations in Organic Chemistry (D), 1982, pp 1-145. Non-Metal Based Reagents Activated DMSO Review: Synthesis 1981, 165; 1990, 857. Me

Me S+

S+

+ E

O-

Me

E

O

Organic Reactions 1990, 39, 297

Nu:

Nu

S

Me

Me

+

+ E-O Me

E= (CF3CO)2O, SOCl2, (COCl)2, Cl2, (CH3CO)2O, TsCl, MeCl, SO3/pyridine, F 3CSO2H, PO5, H3PO4, Br2 Nu:= R-OH, Ph-OH, R-NH2, RC=NOH, enols Swern Oxidation - trifluoroacetic anhydride can be used as the activating agent for DMSO O Me

Me

(COCl) 2

S + O-

CH 2Cl 2, -78°C

Me

R2CH-OH Me Me

Me

-CO, -CO 2

Cl -

Me S + Cl Me

O

R R

S+ O

Cl

S+ O

Et3N:

Me

R

S

+

O

Me

R

H B:

O Cl

O

DMSO, (COCl) 2 OH

Moffatt Oxidation (DMSO/DCC)

O

JACS 1965, 87, 5661, 5670.

Me

C 6H11

S + O-

CF 3CO 2H, Pyridine

Me + C 6H11 N C

TL 1988, 29, 49.

CH 2Cl 2, Et3N

N C 6H11

OH CO 2Me O

Me

NH S

+

O C

R2CH-OH

Me

R O

H

R B:

C 6H11 CHO DCC/ DMSO CO 2Me

CF 3CO 2H, Pyridine

JACS 1978, 100, 5565

O

S

SO3/Pyridine

S+ O

N

Me

R R

Me

S

JACS 1967, 89, 5505. CO 2Me HO

H

CONH 2 H

HO

OH

OH

CO 2Me H

SO 3, pyridine, DMSO, CH 2Cl 2

CONH 2 H HO

O

JACS 1989, 111, 8039.

OXIDATIONS Corey-Kim Oxidation

(DMS/NCS)

19

JACS 1972, 94, 7586. O

Me

Me S:

+

S + Cl

N Cl

Me

Me O N-Chlorosuccinimide (NCS)

Acc. Chem. Res. 1980, 13, 419

••

••

••

O O

singlet

"ene" reaction

H O

Tetrahedron 1981, 37, 1825



•• •• •O O • •• •• triplet

••

Oxygen & Ozone Singlet Oxygen

Ph3P:

H

O

O

O

OH Tetrahedron 1981, 1825

1) O2, hν, Ph2CO 2) reduction

Ozone

HO

Comprehensive Organic Synthesis 1991, 7, 541 O

O 3, CH 2Cl 2

O

O

-78°C

O

Ph3P:

O O

NaBH 4

+

O

O

H Jones

OH

RCOOH

Other Oxidations Mukaiyama Oxidation

BCSJ 1977, 50, 2773 O R

PrMgBr CH OH

R

N

R

N

N

N R

O

CH O MgBr

O

R

THF

R

OH Cl MeO

CH 3

O

O

NH

O O SEt SEt MeO

N

Cl

O N N

N

O

MeO

CH 3

OHC O

NH

OEt

O tBuMgBr, THF (70%)

SEt SEt MeO JACS 1979, 101, 7104

OEt

OXIDATIONS

20

O

OH

tBuMgBr, THF O N

N

N

N O

O

O

Dess-Martin Periodinane JOC 1983, 48, 4155. - oxidation conducted in CHCl3, CH3CN or CH2Cl2 - excellent reagent for hindered alcohols - very mild

JACS 1992, 113, 7277.

OAc

OAc

AcO

••

I

O

OAc

R

R2CH-OH

I O

+

+ 2 AcOH

O

R O

O

HO

Dess-Martin

O JOC 1991, 56, 6264

(99%) RO

RO

Chlorite Ion -oxidation of α,β-unsaturated aldehydes to α,β−unsaturated acids. Tetrahedron 1981, 37, 2091 NaClO 2, NaH2PO 4 OBn

- HClO2 OBn

OBn OH H

tBuOH, H 2O

CHO

CO 2H

-O-Cl-O

Selenium Dioxide - Similar to singlet oxygen (allylic oxidation) 1) SeO2 2) NaBH 4

OAc

OAc OH

Phenyl Selenium Chloride O

OLi PhSeCl

O SePh

H2O 2

Ph Se O-

THF

O - PhSeOH

H

- PhS-SPh will do similar chemistry however a sulfoxide elimination is less facile than a selenoxide elinimation. Peroxides & Peracids - R3N: → R3N-O - sulfides → sulfoxides → sulfones -Baeyer-Villiger Oxidation- oxidation of ketones to esters and lactones via oxygen insertion Organic Reactions 1993, 43, 251 Comprehensive Organic Synthesis 1991, vol 7, 671.

OXIDATIONS

21

m-Chloroperbenzoic Acid, Peracetic Acid, Hydrogen peroxide O

O

H O

O 2N

O

O

O

H

O R1

R2

O O

HO

O

NO 2

Cl

R1

H

Ar

O

C R2 O

R1

O

+

R2

ArCO2H

Ar

O O

- Concerted R-migration and O-O bond breaking. No loss of stereochemistry - Migratory aptitude roughly follows the ability of the group to stabilize positive charge: 3° > 2° > benzyl = phenyl > 1° >> methyl JACS 1971, 93, 1491 O

O mCPBA

O

HO

O CO2H

O

CHO O

HO

O

O

CO2H

HO

OH PGE1

O O CH3

O

mCPBA

Tetrahedron Lett. 1977, 2173 Tetrahedron Lett. 1978, 1385

CH3

(80 %) CH3

CH3

Oxone (postassium peroxymonosulfate)

Tetrahedron 1997, 54, 401

oxone

RCHO

RCOOH

acetone (aq)

Oxaziridines reviews: Tetrahedron 1989, 45, 5703; Chem. Rev. 1992, 92, 919 O N C R

R3 R2

- hydroxylation of enolates O R

O

Base

R

R'

O

_ R R'

O O

PhSO2 O R

Ph

N

R

R'

+ PhSO2N=CHPh

HO Ph O

_ R'

O

R' _ NSO2Ph

R

+ PhSO2N=CHPh Ph

R' NHSO2Ph

By-product supresed by using bulkier oxaziradine such as camphor oxaziradine

OXIDATIONS

22

Asymmetric hydroxylations O

O NaN(SiMe3)2, THF

MeO 2C

HO Tetrahedron 1991, 47, 173

MeO 2C OMe

OMe N Ar

MeO

O

SO 2 O MeO

KN(SiMe3)2

CO2Me

(67% ee) O

O

OH CO2Me

OH

O OH OH

N SO2 O

MeO

MeO

MeO

O

OH

OH

(>95% ee)

- hydroxylation of organometallics R-Li or R-Mg → R-OH

JACS 1979, 101, 1044

- Asymmetric oxidation of sulfides to chiral sulfoxides. JACS 1987, 109, 3370. Synlett, 1990, 643. Remote Oxidation (functionalization) Barton Reaction

Comprehensive Organic Synthesis 1991, 7, 39.

NOCl, CH2Cl2 pyridine OH

hν O

NO

- NO •

O •

OH

H



•NO

JACS 1975, 97, 430 OH

OH

NO

N

N ketone oxidation state

HO

C5H11

perhydrohistricotoxin

Epoxidations Peroxides & Peracids - olefins → epoxides Tetrahedron 1976, 32, 2855 - α,β-unsaturated ketones, aldehydes and ester → α,β-epoxy- ketones, aldehydes and esters (under basic conditions). O

(CH 2)n

tBuOOH triton B, C6H6

O

O (CH 2)n

JACS 1958, 80, 3845

OXIDATIONS O CO 2Me

CO 2Me mCPBA, NaHPO3

TL 1988, 23, 2793 O

O

H

H O

O

Henbest Epoxidation- epoxidation directed by a polar group OH

OH

OH mCPBA

+

O

OAc

O

10:1 diastereoselection OH

OAc mCPBA

+

O

O

1:4 diastereoselection O Ph

O NH

Ph

NH "highly selective"

mCPBA O

Ar O H H

O proposed transition state: -OH directs the epoxidation

O

O H

- for acyclic systems, the Henbest epoxidation is often less selective Rubottom Oxidation:

JOC 1978, 43, 1588

O

OTMS LDA, TMSCl

TMSO mCPBA

O

H2O

O OH

Sharpless Epoxidation tBuOOH w/ VO(acac)2, Mo(CO)6 or Ti(OR) 4 Reviews: Comprehensive Organic Synthesis 1991, vol 7, 389-438 Asymmetric Synthesis 1985, vol. 15, 247-308 Synthesis, 1986, 89. Org. React. 1996, 48, 1-299. Aldrichimica Acta 1979, 12, 63 review on transition mediated epoxidations: Chem. Rev. 1989, 89, 431. - Regioselective epoxidation of allylic and homo-allylic alcohols - will not epoxidize isolated double bonds - epoxidation occurs stereoselectively w/ respect to the alcohol.

23

OXIDATIONS - Catalysts: VO(acac)2; Mo(CO)6; Ti(OiPr)4 - Oxidant: tBuOOH; PhC(CH3)2OOH

VO(acac)2 tBuOOH

OH

OH

O

OH

OH

(CH2)n

O

(CH2)n

ring size 5 6 7 8 9

VO(acac)2 >99% >99 >99 97 91

MoO2(acac)2 -98 95 42 3

mCPBA 84 95 61 <1 <1

Acyclic Systems: L

M

1,3-interaction

O

R3 A1,2-strain

tBu

O O

R1 R3

L

Rc

Rt

R1

Rt

R2

Rc

O

O M

R2

L

A1,3-strain

Major influences: A1,2-Strain between Rg and R1 A1,3 -strain between R2 and Rc 1,3-interactions between L and R1

O L

(Rg and R2) (R1 and Rc) (L and R2)

VO(acac)2, tBuOOH O OH

+

O

OH

OH

(4 : 1)

tBu

CH3

H

O

L M

H H L

O

O

M

L O

O

L

tBu

O

H 3C H

H H

24

OXIDATIONS VO(acac)2, tBuOOH O OH

+

O

OH

OH

(19 : 1)

tBu L H 3C H

H

H 3C

O

H

M L

H O

O

O

H 3C

H

O

M

L

L

H

O

tBu

CH3 SiMe3

SiMe3

VO(acac)2, tBuOOH

O OH

SiMe3

+ OH

O

OH

(> 99 : 1)

- Careful conformational analysis of acyclic systems is needed. Homoallylic Systems L L O

O OH

V

OtBu

O

OH

dominent stereocontrol element

Titanium Catalyst structure: RO2C OR Ti

CO2R O

O

O

RO

O

OR O

Ti

OR OR

O OR CO2R O

OR Ti

CO2R O

O

O

RO

O

OR O CO2R

Ti O

RO Ti O

O

CO2R O

CO2R O

O

tBu OR

Disfavored

O O

tBu OR

Ti

O

Favored

O CO2R

25

OXIDATIONS

26

Asymmetric Epoxidation tBuOOH, Ti(OiPr), (+) or (-) Diethyl Tartrate, 3Å molecular sieves Empirical Rule R1

(+)- DET epoxidation from the bottom (-)- DET epoxidation from the top

R2

R3

OH

Catalytic system: addition of molecular sieves to "soak" up any water with 3A sieves, 5-10 mol % catalyst is used. Preparation of Allylic Alcohols: R

CO2R'

[(CH3)2CHCH2]2AlH

Na (MeOCH2CH2O)2AlH2

R

(DIBAL)

OH

(REDAL)

R CHO

R C C CH2OH R CO2R'

R

[(CH3)2CHCH2]2AlH

H2, Lindlar's Catalyst

OH

"In situ" derivatization of water soluble epoxy-alcohol (-)-DIPT

O

OH

(R)-glycidol OH

water soluble O (+)-DIPT

O

O OH

OH

(S)-glycidol O O S

NO2

organic soluble

O

Alkoxide opening of epoxy-alcohol product reduced by use of Ti(OtBu)4 and catalytic conditions OO R

OH

O OH OH

from Ti(OiPr)4

R

Stoicheometric vs Catalytic epoxidation: (+)-DET Ti(OiPr)4 tBuOOH

O

OH

stoicheometric: catalytic (6-7 mol %) in situ deriv. with PNB

OH

85% ee 47% yield >95% ee 78% yield 92 % ee >98 %ee after 1 recrystallization (+)-DET Ti(OiPr)4 tBuOOH

R

R

OH

yields: 50 - 100 % ee: > 95%

O

OH

OXIDATIONS Ring Opening of Epoxy-Alcohols OH

REDAL R O

AE R

OH

OH 1,3-Diol

R

OH

DIBAL

R

OH OH 1,2-Diol

Two dimensional amplification OH

OH

OH

(+)-DIPT, Ti(OiPr)4, tBuOOH, 3A sieves

+

(90 % ee) O OH

O

OH

95 : 5

major major

O

O

minor minor

major

minor

95 : 5

95 : 5

OH

OH

O

O

OH

90 % (>99.5 % ee)

OH O

O

OH

meso 9.75%

(+)-DIPT, Ti(OiPr)4, tBuOOH, 3A sieves

OH

R

+

R

OR Ti

CO2R O

O

O

RO

O

O tBu

OR

H

Ti O

O R

CO2R O

OH

0.25 %

Kinetic Resolution of Allylic Alcohols OH

OH

O CO2R

OH

27

OXIDATIONS R3

R4

R3

kinetic resolution -20 °C, 0.5 - 6 days OH

R2

R4

R4 O

+

OH

R2

R1

R3 R2

R1

OH

R1

40 - 50 % yield > 99 % ee

40 - 50 % yield high ee

Reiterative Approach to the Synthesis of Carbohydrate OR OR

OR

(+)-DET, Ti(OiPr)4, tBuOOH, 3A sieves

DIBAL

(MeO)2P(O)CH2CO2Me NaH

CHO

OH

CO2Me OR

OR

HOO OH

PhS

OR OH

OR OH

acetone, H+

O

mCPBA, Ac2O

O

-

HO

Pummerer

O SPh

SPh

PhS-

O O

O

O

DIBAL OAc

CHO

OR

OR

OR

O

O

O

CHO

HO

H

HO

H

HO

H

HO

SPh

O

CHO

H CH2OH L-glucose

Jacobsen Aysmmetric Epoxidation JACS 1990, 112, 2801; JACS 1991, 113, 7063; JOC 1991, 56, 2296. - Reaction works best for cis C=C conjugated to an aromatic ring H

H

H N

N

H N

NaOCl

Mn

Mn O

O Cl O tBu

O

O

tBu

tBu

O

N

5 mol % Cat. ,NaOCl, H2O, CH2Cl2

tBu

O (98% ee)

O

86% ee O

Methyltrioxoruthenium (MTO) Ru(VII) Sharpless et al. JACS 1997, 117, 7863, 11536. Ph

0.5 mol % MTO Ru (VII), pyridine, CH2Cl2 1.5 eq. 30% H2O2 (aq.)

Ph O

28

OXIDATIONS Oxaziridines - Asymmetric epoxidation of olefins

29

Tetrahedron 1989 45 5703 CH3

Ph

O2 O N S N

*

(Murray's Reagent)

C6F5

*

Ph

Dioxiranes

*

Reviews: Chem. Rev. 1989, 89, 1187; ACR 1989, 27, 205 Org. Syn. 1996, 74, 91 O

KHSO 5

O

"oxone"

O

- epoxidation of olefins O

OTBS TBSO TBSO

-

OTBS

O

O

O

TBSO TBSO

CH 2Cl 2, acetone (100%)

JOC 1990, 55, 2411 O

Asymmetric epoxidation JACS 1996, 118, 491. oxidation of sulfides to sulfoxides and sulfones oxidation of amines to amine-N-oxides oxidation of aldehydes to carboxylic acids hydroxylation of enolates 1) LDA 2) Cp 2TiCl2 3)

O

OH H

O

JOC 1994, 59, 2358

O

O

- bis-trifluoromethyldioxirane, much more reactive JACS 1991, 113, 2205. F3C

O

F3C

O

- oxidation of alcohols to carbonyl compounds. 1° alcohols give a mixture of aldehydes and carboxylic acids. - Insertion into 3° C-H bonds to give R3C-OH DCC-H2O2

JOC 1998, 63, 2564 R R N C N R

H2O2, MeOH

H

N R

N C

H O

O

O

O

R R

+ H

N R

C

N R

H

REDUCTIONS Carey & Sundberg Chapter 5 problems: 1a,b,c,d,f,h,j; 2; 3a-g, n,o; 4b,j,k,l; 9; 11; Smith: Chapter 4 March: Chapter 19

30

Reductions 1. Hydrogenation 2. Boron Reagents 3. Aluminium Reagents 4. Tin Hydrides 5. Silanes 6. Dissolving Metal Reductions Hydrogenations Heterogeneous Catalytic Hydrogenation Transition metals absorbed onto a solid support metal: Pd, Pt, Ni, Rh support: Carbon, alumina, silica solvent: EtOH, EtOAc, Et2O, hexanes, etc. -

Reduction of olefins & acetylenes to saturated hydrocarbons. Sensitive to steric effects and choice of solvent Polar functional groups, i.e. hydroxyls, can sometimes direct the delivery of H2. Cis addition of H2. R1

R1

R2

R2

H

H2, Pd/C R1

H

R2

R1 R2

- Catalyst can be "poisoned" - Directed heterogeneous hydrogenation O

O

H

H2, Pd/C O

O

OH

MeO

O

OH

MeO H2, Pd/C

H

O

O

CO2Me

MeO

O

CO2M2 (86 : 14)

MeO

Lindlar Catalyst ( Pd/ BaSO4/ quinoline)- partially poisoned to reduce activity; will only reduce the most reactive functional groups. acetylenes + H2, Pd/BaSO4/ quinoline → cis olefins Acid Chlorides + H2, Pd/BaSO4 → Aldehydes

R

SiMe 3

(Lindlar Reduction) (Rosemund Reduction) Org. Rxn. 1948, 4, 362

H2, Pd/BaSO4, Quinoline

SiMe 3 TL 1976, 1539 R O

O H2, Pd/BaSO4, pyridine CO 2Me

CO 2Me

JOC 1982, 47, 4254

REDUCTIONS HO

OH

H2, Lindlar Catalyst CH 2Cl 2: MeOH: Quinoline (90:9.5:0.5)

8π e-

JACS 1982, 104, 5555 6π edisrotatory

conrotatory

HO

OH

HO

H H HO

OH

H

OH

Ease of Reduction: (taken from H.O. House Modern Synthetic Reactions, 2nd edition) R COCl

R CHO

R NO 2

R NH 2

R

R'

R

R CH 2-OH

R CHO R CH CH

R CH 2 CH 2 R'

R'

O R

HO H R'

Ar

R'

R

O

R

R'

Ar

CH 3

+

HO R

R CH 2 NH 2

R C N

O R CH 2-OH R

+

HO R'

OR' O R'

R

R

N

CH 2

R' N R'

R'

requires high temperature & pressure

R CO 2- Na+

no reaction

Raney Nickel Desulfuriztion , Reviews: Org. Rxn. 1962, 12, 356; Chem. Rev. 1962, 62, 347. R

R S

R

Raney Nickel

R

H

R

H

(CH 2)n

O R S

31

REDUCTIONS O O

O

O O

HO

32

O

HO Raney Nickel JOC 1987, 52, 3346

EtOH (74%)

H

S

H

S

Homogeneous Catalytic Hydrogenation - catalyst is soluble in the reaction medium - catalyst not "poisoned" by sulfur - very sensitive to steric effects - terminal olefins faster than internal; cis olefins faster than trans R

R

>

>

R

R

R

R

>

R

R

R

>

R

R

>>

R

R R

- (Ph3P)3RhCl (Wilkinson's Catalyst); [R3P Ir(COD)py]+ PF6- (Crabtree's Catalyst) (Ph3P)3RhCl, H2

OH

OH

JOC 1992, 57, 2767

C 6H6 (92%)

Directed Hydrogenation Review: Angew. Chem. Int. Ed. Engl. 1987, 26 , 190 - Diasterocontrolled hydrogenation of allylic alcohols directed by the -OH group -

+

O K

O- K+

PPh3 O PPh3 Rh H H

(Ph3P)3RhCl, H2

H MeO

MeO

Ph Ph P Rh+ P Ph Ph Brown's Catalyst

BF4-

Ir +

P(C 6H11)3 N

PF6 -

(Crabtree's Ctalyst) JACS 1983, 105 , 1072

Regioselective Hydrogenation- allylic and homoallylic alcohols are hydrogenated faster than isolated double bonds MeO2C OH

MeO2C OH

REDUCTIONS

33

mechanism: L

+ M

H2

+ M

L

L

L

+

OH

L

S

M

L

S

O H

lose

H H

H2 (oxidative addition)

reductive elimination OH migratory insertion

HO H L

M L

+

H H

M

L

S

L

O H

Diastereoselective Hydrogenation: since -OH directs the H2, there is a possibility for control of stereochemistry - sensitive to: H2 pressure catalyst conc. substrate conc. solvent. Regioselective Hydrogenation- allylic and homoallylic alcohols are hydrogenated faster than isolated double bonds HO "Ir" (20 mol %)

HO

O

O

H

Brown's Catalyst OH

JACS 1984, 106, 3866

H2 (640 psi), CH2Cl 2

H

OMe

OMe

Me

(24 : 1)

OH

OMe H Me

OMe Brown's Catalyst H2 (1000 psi)

O

TL 1987, 28 , 3659

O OH

OH

Selectivity is often higher with lower catalyst concentration: OH

OH

OH

20 mol % Catalyst

2.5 mol % Catalyst

50 : 1

150 : 1

33 : 1

52 : 1

OH

REDUCTIONS

34

Olefin Isomerization: CH3 (3 : 1) OH

OH olefin isomerization

O

OH

major product

- Conducting the hydrogenation at high H 2 pressures supresses olefin isomerization and often gives higher diastereoselectivity. Other Lewis basic groups can direct the hydrogenation. (Ir seems to be superior to Rh for these cases) OMe

CO2Me

OMe Ir

CO2Me

+

Ir+

99 : 1

O

O

CO2H

Ir+ 7 : 1 Rh+ 1 : 1

O

O

N

O

O

N

O

130 : 1

1:1

Acyclic Examples Rh+ (2 mol %)

OH

Me CH 3 JCSCC 1982, 348

H2 (15 psi)

Me

L L M

OH

(97:3)

H O OH

H H

Ph H 3C

H H L

CH3 Ph

M L

Ph

anti

H

H

O H

32 : 1

CO2H

> 99 : 1

O

> 99 : 1

Rh+

OH

1,2-strain Ph

syn

REDUCTIONS L L R3 H OH R3

35

H O

M

R2 R1

OH

favored R3

H

syn

R1 R2

R1 R2 R3 H L

H

R1 R2

M

OH

disfavored R3

1,2-strain

R1 R2

O H

L

anti

- Supression of olefin isomerization is critical for acyclic stereocontrol ! L L

OH R2

H O

M

OH R2 H

R1

CH3 R1

CH3

R2

H

R1

anti

R2

olefin isomerization OH R2

OH

L L

R1

H O

M

R2

H H

R1 R2

CH2R2 R1

H

- Rh+ catalyst is more selective than Ir + for acyclic stereoselection. Acyclic homoallylic systems: HO R1

R3 R2

relative stereochemistry is critical

A E R2

A O H

M R3

1,3-strain R3

L L

E 1,2-strain R2

O H

L M

L

syn

REDUCTIONS Rh+

OH

(20 mol %)

36

OH

H2 (15 psi) OBz

OBz

TBSO

32 : 1

TBSO

Tetrahedron Lett. 1985, 26, 6005 Rh+ (20 mol %)

OH

OH

H2 (15 psi) OBz

OBz

TBSO

8:1

TBSO

Asymmetric Homogeneous Hydrogenation - Chiral ligands for homogeneous hydrogenation of olefins and ketones H O

P OMe

MeO

PPh2 PPh2

O

P

H DIOP

PPh2

PPh2

PPh2

PPh2

PPh2

PPh2

R= -CH3 PROPHOS = -Ph PHENPHOS = -C6H11 CYCPHOS

CHIRAPHOS

DBPP

DIPAMP Ph2P

PPh2 PPh2

X

PPh2

CO2tBu BPPM

PPh2

CO 2H

Ph HN

PPPFA

PPh2

P

BINAP

DIPHEMP Rh (I) L*, H2

CO 2H

Ph HN

Ph

(95% ee)

O CO 2Me

Ph O CO 2H

NHAc O

NH 2

HO O

OH L-DOPA

DIOP DIPAMP PPPFA BINAP NORPHOS BPPM

85% ee 96% ee 93% ee 100% ee 95% ee 91% ee

PPh2 PPh2

PPh2

PPh2

DUPHOS

Fe

N

X= CH2 DPCP X= N-R PYRPHOS

CAMPHOS

P

PPh2 PPh2

PPh2 PPh2 NORPHOS

NMe2

ACR 1983, 16, 106.

REDUCTIONS 37 General Mechanism: J. Halpern Science 1982, 217, 401 Asymmetric Synthesis 1985, vol 5, 41. CO2Me

Ph S

P

Rh

P

Ph

NHAc

S

P

Rh

NH O

P

(fast equilibrium)

CO2Me CH3

rate limiting

CO2Me

Ph

H2 (slow)

NHAc Ph P

H Rh

P

O

S

Ph

H CO2Me NH

S

P

(fast)

P

CO2Me H NH

Rh O

CH3 CH3

Detailed Mechanism: P

*

P

MeO2C

+ S

Ph

CO2Me NHAc

NH

P

*

S Rh

Ph

Rh P

P

Ph CH3

O

CO2Me

HN

H 3C

P

O

major complex

minor complex

H2 (slow) MeO2C H H P

* P

H2 (slow)

NH

NH

Ph

Rh

CH3

O

H 3C

Rh

*

S

H

CH3

NH

Ph

H N

H 3C O

P P * S

CO2Me

P

H

Rh O

CH3 O

CO2Me H

Ph

S

P

*

Rh

CO2Me Ph

(R) Minor product

HN MeO2C

O S

P

Rh

*

H P

Ph

MeO2C Ph

H N

CH3 O

(S) Major product

REDUCTIONS

+

MeO2C H

NH

H

Ph

Rh P P

Free Energy

*

NH

38

CH3

O

+

CO2Me H H

Ph

+

CO2Me

HN

Rh O

P P *

P

Ph H 3C

H 3C

Rh

*

P

O minor complex

MeO2C

*

Ph

Rh P

+

NH

P

CH3

O major complex

Reaction Coordinate

Ph Ph O O P Ru O P O Ph Ph

ACR 1990, 23, 345.

BINAP

O O

(BINAP)RuAc2, H2 (100 atm)

O

O

O R

O

O

50 °C, CH2Cl 2

(BINAP)RuAc2, H2 (100 atm)

R O

50 °C, CH2Cl 2

(95 - 98 % ee)

(94 % ee)

CO 2H

Ru(AcO)2(BINAP), H2 (135 atm), MeOH (97% ee)

MeO

CO 2H 97 % ee

MeO (S)-naproxen

MeO MeO

MeO (BINAP)RuAc2, H2 (4 atm)

NAc

MeO

MeO NAc

NH

MeO

OMe

OMe (95 - 100 % ee)

OMe

OMe

OMe

OMe Tetrahydropapaverine

Directed Asymmetric Hydrogenation (BINAP)Ru (II), H2 OH

CHO

OH

(96 - 99 % ee)

OH

OH

HO J. Am. Chem. Soc. 1987, 109, 1596 O Vitamin E

REDUCTIONS

39

Kinetic Resolution by Directed Hydrogenation

CF 3SO 3-

Rh+

MeO Ph P

P Ph

CO 2Me

MeO 2C

H2 (15 psi), L*Rh+

Et Ph OMe

CO 2Me

MeO 2C

0 °C (~60% conversion)

R

R=

OMe

R

> 96 % ee 82 % 93 %

Hydrogenation of Carbonyls 1,3-diketones: O R1

O

O

R1

OH

H2

O

H

OH

OH R2

R2 R1

O

H O H

syn

anti Ru2Cl 4(BINAP) Et3N, H2 (100 atm)

MeO 2C

99 : 1 16 : 1 (90 % ee) 32 : 1 49 : 1

R1

R2

R1

R2

Directed Reduction

M

O

R1

anti : syn=

OH

R1

O R2

OH

syn

anti

H O

M

OH

R2

-CH3 -CH2CH3 -iPr -CH2CH3

R2

R2

+ R1

R2=

OH

R1

OH

R2

-CH3 -CH3 -CH2CH3 -CH2CH3

O R2

Ru (II) H2 (700 psi)

O

R1

O

R1

R2

O

R1=

OH

O

OH JACS 1988, 110 , 6210

MeO 2C

(98% ee)

Decarbonylations O

(Ph3P)3RhCl

O

(Ph3P)3RhCl

R H R

H

- CO

R Cl R

Cl

- CO

REDUCTIONS

40

OHC

(Ph3P)3RhCl Fe

Fe

Fe

PhCH 3, ∆

JOC 1990, 55, 3688

Fe

Diimide HN=NH Review: Organic Reactions 1991, 40J. Chem. Ed. 1965, 254 - Only reduces double bonds - Syn addition of H 2 - will selectivley reduce the more strained double bond - Unstable reagent which is generated in situ K+O2C-N=N-CO2K+ + AcOH → H-N=N-H H2N-NH2 + Cu2+ + H2O2 → H-N=N-H HN=NH ACIEE 1965, 271 (76%) O

O +-

CO 2MeK

O 2C N N CO 2- K+

CO 2Me

AcOH, MeOH (95%)

NO 2

JACS 1986, 108 , 5908

NO 2

O HN HN

hν (254 nm)

S

NH NH

+ COS + CO

O O

S

O

N N H H

TL 1993, 34, 4137

hν, 16 hr (96%)

Metal Hydrides Review on Metal Hydride Selectivity:

Chem Soc Rev. 1976, 5 , 23 Comprehensive Organic Synthesis 1991, vol 8, 1. Boron Hydrides Review: Chem. Rev. 1986, 86 , 763. NaBH4 reduces ketones and aldehydes LiBH4 reduces ketones, aldehydes, esters and epoxides. THF soluble LiBH4/TMSCl stronger reducing agent. ACIEE 1989, 28, 218. Zn(BH4)2 reduces ketones and aldehydes R4N BH4 organic soluble (CH2Cl2) borohydrides. Synth Commun. 1990, 20, 907 LiEt3BH reduces ketones, aldehydes, esters, epoxides and R-X Li s-Bu3BH reduces ketones, aldehydes, esters and epoxides (hindered borohydride) Na(CN)NH3 reduces iminium ions, ketones and aldehydes Na(AcO)3BH reduces ketones and aldehydes (less reactive) NaBH2S3 reduces ketones and aldehydes

REDUCTIONS

41

Sodium Borohydride NaBH4 - reduces aldehydes and ketones to alcohols - does not react with acids, esters, lactones, epoxides or nitriles. - Additives can increase reactivity. Sodium Cyanoborohydride Na (CN)BH3 Reviews: Synthesis 1975, 136; OPPI 1979, 11 , 201 - less reactive than NaBH4 - used in reductive aminations (Borch Reduction) Na(CN)BH3 reduces iminium ions much more quickly than ketones or aldehydes R"-2NH, MeOH, AcO - NH4+ , pH~ 8

R O

R

R'

O CHO

R

N R' JACS 1971, 93 , 2897

R"-2NH, MeOH, AcO- NH4+

N H

Na(CN)BH3

N

R'

R'

R

R

R H

Na(CN)BH3

N+

N

- Related to Eschweiler-Clark Reaction H2CO, HCO 2H R NH 2

R

or

N H

Me

H2CO, H2/Pd

- Reduction of tosylhydrazones gives saturated hydrocarbon O H

H

1) TsNHNH 2, H+ 2) Na(CN)BH3

TL 1978, 1991

(90%)

H

HO

H

HO 1) TsNHNH 2, H+ 2) Na(CN)BH3

O

(100%)

JOC 1977, 42 , 3157

- migration of the olefin occurs w/ α,β-unsaturated ketones O JACS 1978, 100 , 7352 (75%)

- Epoxide opening O OH

NaBH3CN, BF3•OEt2, THF

OH HO

JOC 1994, 59, 4004

NaBH2S3

REDUCTIONS Synthesis 1972, 526 Can. J. Chem. 1970, 48 , 735.

Lalancette Reduction

42

NaBH4/ NiCl 2 Chem. Pharm. Bull. 1981, 29 , 1159; Chem. Ber. 1984, 117 , 856. Ar-NO2 → Ar-NH2 Ar-NO → Ar- NH2 R2C=N-OH → R2CH-NH2 NaBH4 / TiCl 4 Synthesis 1980, 695. R-COOH → R-CH2-OH R-COOR' → R-CH2-OH R-CN → R-CH2-NH2 R-CONH2 → R-CH2-NH2 R2C=N-OH → R2CH-NH2 R-SO2-R' → R-S-R' NaBH4 / CeCl 3 Luche Reduction reduced α,β-unsaturated ketones in a 1,2-fashion OH NaBH 4/ CeCl3

R R

NaBH 4

R

R

O

OH

O

R

+

R R

R

R R

R

H

R JCSCC 1978, 601 JACS 1978, 100 , 2226

O

OH CO 2Me

NaBH 4/ CeCl3

C 5H11

CO 2Me

MeOH

C 5H11

OH

OH

- selective reduction of ketones in the presence of aldehydes. OH

O CO 2Me

CO 2Me

NaBH 4/ CeCl3 EtOH, H2O CHO

CHO O CeCl 3 H2O

1) NaBH 4, CeCl3 2) work-up

R

JACS 1979, 101 , 5848

OH OH O

OH

CHO NaBH 4/ CeCl3 EtOH, H2O (78%)

CHO

REDUCTIONS Zinc Borohydride Zn(BH4)2 Synlett 1993, 885. ZnCl2 (ether) + NaBH 4 → Zn(BH4)2 - Ether solution of Zn(BH4)2 is neutral- good for base sensitive compounds - Chelation contol model Zn RO

O R1

H

O

OR

OH

H B H

H

R2

OR

R1 R2

H

R1 R2

Zn(BH4)2, Et2O, 0°C

OH

OH OH

O H

H-

Me

TL 1983, 24 , 2653, 2657, 2661 O

O Zn

R

Na + (AcO)3BH , Me4N + (AcO)3BH Review: OPPI 1985, 17 , 317 - used in Borch reductive amination TL 1990, 31 , 5595; Synlett 1990, 537 - selective reduction of aldehydes in the presence of ketones O

Bu4N (AcO) 3BH C 6H6, ↑↓

O OH

CHO (77%)

TL 1983, 24 , 4287 AcO O

Bu4N (AcO) 3BH C 6H6, ↑↓

CHO

Ph

H O

OAc B OH

O Ph

Ph

OH

-hydroxyl-directed reduction of ketones TL 1983, 24 , 273; TL 1984, 25 , 5449 OH

O

Me

O

OH Ph

N Me

Me

OH

Me4N (AcO) 3BH, CH 3CN, AcOH, −40°C

Me

O

Ph

N Me

O

O

Me O

O

TL 1986, 27 , 5939 JACS 1988, 110 , 3560

(98:2) OAc

H O R

B

O

OAc

OAc

H R

R

H

OAc

H O

R

minor

major

OH

B

O

O Na+ BH(OAc)3

OH

OH

50 : 1

43

REDUCTIONS OH

O

O

OH

Na+ BH(OAc)3

OH

O CO2R

CO2R

OH

Na+ BH(OAc)3

44

OH

OH CO2R

HO OBn

BnO

OBn

BnO

Me

MeO

OBn

O

OBn

Me4N (AcO) 3BH, CH 3CN, AcOH, −40°C

O

Me MeO

Me

O

MeO

O

O Me

Me

OH

O

N

OH

O

Me O

O HO

OH

OH

Me FK-506

OH OH

TL 1989, 30 , 1037

(Ph3P)2Cu BH4 reduction of acid chlorides to aldehydes reduction of alkyl and aryl azides to amines

JOC 1989, 45, 3449 J. Chem. Res. (S) 1981, 17

R4N BH4 organic soluble borohydride (CH2Cl2) R4N= BnEt3N or Bu4N Heterocylces 1980, 14, 1437, 1441 reduction of amides to amines reduction of nitriles to amines BnEt3N BH4 / Me 3SiCl reduction of carbolxylic acids to alcohols LiBH4/ Me 3SiCl

Synth. Commun. 1990, 20, 907

ACIEE 1989, 28, 218.

Alkyl Borohydrides Selectrides M + HB 3

M + = Li (L-selectride) K (K-selectride)

LS-selectride

Li + HB

3

- hindered reducing agent increased selectivity based on steric considerations CO 2H

O

CO 2H

OH

L-selectride THF

JACS 1971, 93, 1491 R

R HO

HO OH

OH

REDUCTIONS

45

- selective 1,4-reductions of α,β-unsaturated carbonyl cmpds. JOC 1975, 40 , 146; JOC 1976, 41 , 2194 O

O K-selectride, THF (99%)

- 1,4-reduction generates an enolate which can be subsequently alkylated. O

O a) K-selectride, THF b)

Br

K+ HBPh3 Syn. Comm. 1988, 18 , 89. - even greater 1,4-selectivity Li + HBEt3 (Super Hydride) - very reactive hydride source - reduces ketones, aldehydes, esters, epoxides and C-X (alkyl halides and sulfonates) O

HO

Li Et3BH, THF

HO

HCA 1983, 66 , 760

HO

H

H HO

HO

CH 3

OH

HO

1) TsCl, pyridine 2) Li Et3BH, THF

HCA 1988, 71 , 872

HO H

H

Boranes Hydroboration H2O 2, NaOH

B 2H6 B

R

R'

R

B 2H6

R' B

R

B 2H6

H

HO H

H H3O +

H

R

B

H2O 2, NaOH

R

R'

H

H

R-CH 2CHO

H

- BH3 reduces carboxylic acids to 1° alcohols in the presence of esters, nitro and cyano groups. - BH3 reduces amides to amines HO 2C

BH 3•SMe 2 O

O

THF

HO O

O

- Boranes also reduce ketones and aldehydes to the corresponding alcohols.

REDUCTIONS Hindered Boranes Disiamyl Borane (Sia2BH)

B H

Thexyl Borane

B H

H B 9-BNN

=

B H

B H O

Catecholborane

BH O BH

BH Pinylborane 2

2

B

B H

Alpine Borane

BCl

BCl IPC 2BCl (DIP-Cl)

2

2 B H

Borolane

Ph Oxazaborolidine

N B H

Ph O

Asymmetric Reduction of Unsymmetrical Ketones Using Chiral Boron Reagents Review: Synthesis 1992, 605. Alpine Borane Midland Reduction JACS 1979, 111 , 2352; JACS 1980, 112 , 867 review: Chem. Rev. 1989, 89 , 1553. O

alpine-borane THF, 0°C

OH Tetrahedron 1984, 40 , 1371 (94% ee)

46

REDUCTIONS B H

B =

9-BBN

B B

α-pinene

9-BBN

B H

Mechanism:

OH O

RL

Rs

RL

Rs

- works best for aryl- and acetylenic ketones - because of steric hindrance, alpine-borane is fairly unreactive Chloro Diisopinylcamphenylborane (DIP-Cl, Ipc2BCl) H.C. Brown Review: ACR 1992, 25 , 16. Aldrichimica Acta 1994, 27 (2), 43 BCl

2

- Cl increases the Lewis acidity of boron making it a more reactive reagent - saturated ketones are reduced to chiral alcohols with varying degrees of ee. O

OH

I

CO2tBu

Ipc2B-Cl, THF

I

CO2tBu JOC 1992, 57, 7044

PrO

PrO OMe

(90 % ee)

OMe

Borolane (Masamune's Reagent) JACS 1986, 108 , 7404; JACS 1985, 107, 4549 B H B O

OH

H MeSO 3H, pentane

(80% ee)

Asymmetric Hydroboration: a)

B H

b) H2O 2, NaOH

OH (99.5% ee)

(99.5% ee)

47

REDUCTIONS Oxazaborolidine (Corey) JACS 1987, 109 , 7925; TL 1990, 31, 611l ; TL 1992, 33 , 4141 Ph

Ph

O

N B

Ph

N B H

Ph O

+ BH3

CH 3 Catalytic

O

OH R

R

> 90 % ee OH

O

92 % ee

O

OH

I

86 % ee

I O

OH 93 % ee

Ph Ph O

O N B Me

OH

BH 3•THF, -0°C

TL 1988, 29 , 6409 (90% ee) CF 3

O

OH Cl

O Cl

• HCl N H

CH 3

94 % ee Fluoxetine (Prozac)

O

O

O

O 90 : 10

BzO

BzO

O

Aluminium Hydrides 1. LiAlH 4 2. AlH3 3. Li (tBuO)3AlH 4. (iBu)2AlH DIBAL-H 5. Na (MeOCH2CH2O)2AlH2

REDAL

OH

48

REDUCTIONS 49 Chem. Rev. 1986, 86, 763 Org. Rxn. 1951, 6, 469.

Lithium Aluminium Hydride LiAlH4 (LAH) - very powerful reducing agent - used as a suspension in ether or THF - Reduces carbonyl, carboxylic acids and esters to alcohols - Reduces nitrile, amides and aryl nitro groups to amines - opens epoxides - reduces C-X bonds to C-H - reduces acetylenic alcohols trans-allylic alcohols LAH

OH

R

OH

R

H2N

LAH, THF

N H

NH 2

H2N

(62%)

NH 2

N H

NH 2 Lindlar/ H2 H2N O

N H HO

CO 2Me

OH LAH, THF, ↑↓ TL 1988, 29 , 2793.

(100%) O

O

H O

H O

BINAL-H (Noyori) - Chiral aluminium hydride for the asymmetric reduction of prochiral ketones

OH OH

1) LiAlH4 2) ROH

H

O

Li +

Al O

OR

R= Me, Et, CF 3CH 2-

BINOL

BINAL-H

O O

O

BINAL-H,THF

Tetrahedron 1990, 46 , 4809

-100 to -78°C HO

(94% ee)

Intermediate for 3-Component Coupling Strategy to Prostaglandins I O

Li+ O CO 2Me

RO

Li+ RCu

RO OTBS

O

OTBS O CO 2H

CO 2Me

HO RO

OTBS

OH PGE 2

REDUCTIONS Alane

50

AlH3 LiAlH 4 + AlCl3 → AlH3 - superior to LAH for the 1,2-reduction of α,β-unsaturated carbonyls to allylic alcohols OMe Ph

1) AlH3, ether, 0°C 2) H3O +

O

O

HO

O Me

O JACS 1989, 111 , 6649

Me

O

OH

O

Diisobutyl Aluminium Hydride

DIBAL or DIBAL-H Al H

- Reduces ketones and aldehydes to alcohols - reduces lactones to hemi-acetals O

Al

O

OH CHO

work up

DIBAL O (CH 2)n

O

OH

O

(CH 2)n

(CH 2)n

(CH 2)n lactol

(stable complex)

- reduces esters to alcohols - under carefully controlled reactions conditions, will partially reduce an ester to an aldehyde Al

O DIBAL

R CO 2Me

if complex is unstable

R C OMe

fast

R CHO

R CH 2 OH

H if complex is stable R CHO O O

O

OBn O

DIBAL, CH 2Cl 2

O

HO OH

HO

HO

H

O

HO

OBn O

OBn

OH

OBn

OH JACS 1990, 112 , 9648 OBn

OBn DIBAL, CH 2Cl 2

CO 2iPr

iPrO2C

OHC

OBn O R

OBn O

TMS-Cl, Et3N OH

CHO

CH2Cl2

R

O

DiBAl-H OSiMe3

CH2Cl2, -78°C

TL 1998, 39, 909 R

H

Reduction of O-Methyl hydroxamic acids O R

O

R'-M R'

R

O OMe N Me

TL 1981, 22 , 3815

DIBAL or LAH R

H

REDUCTIONS

51

Sodium Bis(2-Methoxyethoxy)Aluminium Hydride REDAL Organic Reactions 1988, 36, 249 Organic Reactions 1985, 36, 1. MeO MeO

Na+

H

O

Al

O

H

- "Chelation" directed opening fo allylic epoxides OH O

REDAL R

OH

DIBAL-H

R

R

OH

Ph

1,2-diol OH

REDAL

O

OH

Ph

TL 1982, 23 , 2719

OH

1,3-diol Sharpless epoxidation

OH

OH

DME

LiAlH4 AlH3

OH

OH

+

OH

O

JOC 1988, 53 , 4081

Ph

2 : 98 95 : 5 OH

O

OH

BnO

+

OH

BnO

OH

BnO

OH REDAL DIBAL

O

OH Me HO

OH

O O Me

OH

OH

150 : 1 1 : 13

OH

OH

Me

OH

HO

O

OH

Me

Amphotericin B HO

O NH 2

Me

Me O

Me O

OH

Me OH

O

O O

sugar

sugar

Me Erythromycin A

Li+ (tBuO)3AlH

Lithium Tri(t-Butoxy)aluminium Hydride - hindered aluminium hydride, will only react with the most reactive FG's O R

Li(tBuO)3AlH Cl

R

H

Me

Cl

O

N+ R-CO 2H

O

Me

H

R

pyridine, -30°C

O H

Me N+ Me

O

Li(tBuO)3AlH CuI (cat), -78°C

R

TL 1983, 24 , 1543 H

Meerwein-Ponndorf-Verley Reduction: opposite of Oppenauer oxidation Synthesis 1994, 1007 Organic Reactions 1944, 2, 178 O H

O H Al(O-Pr)3, iPrOH

O

O O AcHN

O

O AcHN

OH

REDUCTIONS

52

Asymmetric M-P-V Reduction Bn

Ph

X

O

Ph

N O

Sm

X

O

OH

I

JACS 1993, 115, 9800

iPrOH, THF X= H, Cl, OMe yield: 83-100 % 96% ee

Dissolving Metal Reductions Birch Reductions reduction of aromatic rings Organic Reactions 1976, 23, 1. Tetrahedron 1986, 42, 6354. Comprehensice Organic Synthesis 1991, vol. 8, 107. - Li, Na or K metal in liquid ammonia H H

H H M, NH3

_• •

R

R

R

R

H

H H

- position of the double bond in the final product is dependent of the nature of the substituent R

R

R R= EWG

R= ERG

- ketones and nitro groups are also reduced but esters and nitrile are not. - α,β-unsaturated carbonyl cmpds are reduced in a 1,4-fashion to give an enolate which can be subsequently used to trap electrophiles Me HO

Me Me

O O

K, NH3, MeOH, -78°C

Me HO

O

JOC 1991, 56 , 6255 O

(92%)

O

Me Me OH

Me

Me O O

O O

a) Li, NH3, tBuOH b) CH 2O

JOC 1984, 59 , 3685

O

O HO

Other Metals - Mg Mg, MeOH X

X

X- CN, CO2R, CONR'2

H

H Mg, MeOH

EtO2C

(98%)

TL 1987, 28 , 5287 EtO2C

- Zn reduction of α-halocarbonyls O

O Br

Zn, PhH, DMSO, MeI

Me

JACS 1967, 89, 5727

REDUCTIONS Cl R

Zn

O

X

Y

Zn, AcOH

R

R'

X= Cl, Br, I Y= X, OH

R

R-OH O

CH

53

CH R'

"Copper Hydrides" LAH or DIBAL-H + MeCu → "CuH" - selective 1,4-reduction of α,β-unsaturated ketones (even hindered enones) O

O 1) MeCu, DIBAL, HMPA, THF, -50°C 2) MeLi 3)

JOC 1987,52 , 439

Br

O

O MeCu, DIBAL, HMPA THF, -50°C

H

H

JOC 1986,51 , 537

(85%)

O

H

H

O H

[(Ph3P)CuH]6 Stryker Reagent JACS 1988, 110 , 291 ; TL 1988, 29 , 3749 - 1,4-reduction of α,β-unsaturated ketones and esters; saturated ketones are not reduced - halides and sulfonates are not reduced - 1,4-reduction gives an intermediate enolate which can be trapped with electrophiles. O

Br

O [(Ph3P)CuH] 6, THF

TL 1990, 31 , 3237

Silyl Hydrides - Hydrosilylation Et3SiH + (Ph3P)3RhCl (cat) - selective 1,4-reduction of enones, 1,2-reduction of saturated ketones to alchohols. O

Et3SiH, (Ph3P)3RhCl (cat)

O SiEt3

H3O +

O TL 1972, 5085 J. Organomet. Chem. 1975, 94 , 449

O O O

iPr3SiH, Et2O Si O 2

Pt 2

OSi(iPr)3 O JOC 1994, 59, 2287 O

(87%)

- Buchwald Reduction JACS 1991, 113 , 5093 - catalytic reagent prepared from Cp2TiCl2 + nBuLi and stoichometric (Et)3SiH in THF will reduce ester, ketones and aldehydes to alcohols under very mild conditions. - α,b− unsaturated esters are reduced to allylic alcohols - free hydroxyl groups, aliphatic halides and epoxides are not reduced

REDUCTIONS

54

Clemmensen Reduction

Organic Reactions 1975, 22, 401 Comprehensive Organic Synthesis 1991, vol 8, 307. - reduction of ketones to saturated hydrocarbons Zn(Hg), HCl

H

O

Wolff-Kishner Reduction

H

Organic Reactions 1948, 4, 378 Comprehensive Organic Synthesis 1991, vol. 8, 327.

- reduction of ketones to saturated hydrocarbons H2N-NH2, KOH

H

O

H

Radical Deoxygenation Review: Tetrahedron 1983, 39 , 2609 Chem. Rev. 1989, 89, 1413. Comprehensive Organic Synthesis 1991, vol. 8, 811 Tetrahedron 1992, 48, 2529 W. B. Motherwell, D. Crich Free Radical Chain Reactions in Organic Synthesis (Academic Press: 1992) - free radical reduction of halide, thio ethers, xanthates, thionocarbanates by a radical chain mechanism. nBu3Sn-H, AIBN Ph-CH3 ↑↓

R3C-H

R3C-X S X= -Cl, -Br, -I, -SPh,

S

S Ph , O

O

Barton-McCombie Reduction JCS P1 1975, 1574 R3C-X → R3C-H

N N CN

SMe , O

CN AIBN

N

N

X= -OC(=S)-SMe, -OC(=S)-Im, -OC(=S)Ph O

O O

O HO

NaH, imidazole, THF, CS2, MeI

O

O

O

O

O

O

O

nBu3SnH, AIBN PhCH3, reflux

O

O

O

O

(85%) O

SMe S Xanthate R

R

Cl a) Ph

nBu3SnH, AIBN PhCH3, reflux

NMe2 , THF +

S

b) H2S, pyridine HO

(73%) Ph

(90%)

R

O Thionobenzoates

S Ph

O O O HO AcHN OBn

N

N

N (CH2Cl)2 59%

Ph N N

N

O O O O AcHN OBn S

Thiocarbonyl Imidazolides

nBu3SnH, AIBN PhCH3, reflux (57%)

Ph

O O AcHN

O OBn

REDUCTIONS - Cyclic Thionocarbonates: deoxygenation of 1,2- and 1,3-diols to alcohols

55

S OH

N

O

HO MeO MeO

S

N

O O MeO

N

N

HO

O

JCS P1 1977, 1718

MeO

(61%)

MeO

OMe

nBu3SnH, AIBN PhCH3, reflux

O

MeO

OMe

OMe

- Thionocarbonate Modification (Robbins) JACS 1981, 103 , 932; JACS 1983, 105 , 4059. iPr iPr

O

O

Si

PhOC(S)Cl, pyridine, DMAP

B

iPr iPr

O

nBu3SnH, AIBN PhCH3, reflux

B

O

O

> 90%

Si O iPr iPr

O Si

OH

iPr iPr

O

B

O

O

(58-78%)

Si O iPr iPr

O Si Si O iPr iPr

OPh S

S OAr O O

O CH3

nBu3SnH, AIBN, PhCH3

O O

O

Tetrahedron 1991, 47, 8969

O

88-91%

O

O O

O Best method for deoxygenation of primary alcohols

Ar= 2,4,6-trichlorophenyl, 4-fluorophenyl

- N-Phenyl Thionocarbamates iPr iPr

O

O

Si

U

PhNCS NaH, THF S

O Si O iPr iPr

Tetrahedron 1994, 34, 10193. iPr iPr

O

U

Si O iPr iPr

NHPh

(TMS)3SiH, AIBN PhH, reflux S

O

(78%)

O

O Si

O

(93%)

iPr iPr

O

O

Si

U

O Si O iPr iPr

NHPh

Thiocarbamates

- Methyl oxylates OAc

O

OAc

O MeO2CCOCl THF

OC

nBu3SnH, AIBN PhCH3, reflux

OC O

HO CO2Me

CO2Me MeO2C

O

OAc

O OC CO2Me

Methyl Oxylate Esters

- Water Soluble Tin Hydride: [MeO(CH2)2O(CH2)3]3SnH / 4,4'-Azo(bis-4-cyanovaleric acid) TL 1990, 31 , 2957 - Silyl Hydride Radical Reducing Agents - replacement for nBu3SnH (Me3Si)3SiH Chem Rev. 1995, 95, 1229. JOC 1991, 56 , 678; JOC 1988, 53 , 3641; JACS 1987, 109 , 5267 Ph2SiH2 / Et3B / Air TL 1990, 31 , 4681; TL 1991, 32 , 2569 - hypophosphorous acid as radical chain carrier JOC 1993, 58, 6838

REDUCTIONS - Photosensitized electron transfer deoxygenation of m-trifluoromethylbenzoates JACS 1986, 108, 3115, JOC 1996, 61, 6092, JOC 1997, 62, 8257 iPr iPr

O

O

Si

U N-methylcarbazole, hν, iPrOH, H2O

O Si O iPr iPr

O

iPr iPr

(85%)

O Si O

CF3

O

U

O

Si O iPr iPr

Dissolving Metal : JACS 1972, 94, 5098 O OH

O

nBuLi, (Me2N)2P(O)Cl

O

P(NMe2)2 Li, EtNH2, THF, tBuOH

O O

H

CH3 O

O

H

O

Radical Decarboxylation: Barton esters Aldrichimica Acta 1987, 20 (2), 35 S

hn or ∆

N S

R

N

nBu3SnH, ∆

O

O

R

- CO2

Radical Deamination Comprehensive Organic Synthesis 1991, vol. 8, 811 Reduction of Nitroalkanes

JOC 1998, 63, 5296 NO2 Bu3SnH, PhSiH3

O O

initiator, PhCH3 (reflux) (75%)

O O

R H

H

56

PROTECTING GROUPS Carey & Sundberg Chapter 13.1 problems # 1; 2; 3a, b, c ; Smith: Chapter 7

57

Protecting Groups T.W. Greene & P.G.M. Wuts, Protective Groups in Organic Synthesis (2nd edition) J. Wiley & Sons, 1991. P. J. Kocienski, Protecting Groups, Georg Thieme Verlag, 1994 1. 2 3. 4.

Hydroxyl groups Ketones and aldehydes Amines Carboxylic Acids - Protect functional groups which may be incompatible with a set of reaction conditions - 2 step process- must be efficient - Selectivity a. selective protection b. selective deprotection

Hydroxyl Protecting Groups Ethers Methyl ethers R-OH → R-OMe Formation: Cleavage: -

difficult to remove except for on phenols

CH2N2, silica or HBF4 NaH, MeI, THF AlBr3, EtSH PhSe Ph2P Me3SiI OMe

O

OH

O AlBr3, EtSH

TL 1987, 28 , 3659 O

O OBz

Methoxymethyl ether MOM R-OH → R-OCH2OMe

OBz

stable to base and mild acid

Formation: - MeOCH2Cl, NaH, THF - MeOCH2Cl, CH2Cl2, iPr2EtN Cleavage - Me2BBr2 TL 1983, 24 , 3969

PROTECTING GROUPS Methoxyethoxymethyl ethers (MEM) R-OH → R-OCH2OCH2CH2OMe

stable to base and mild acid

Formation: - MeOCH2CH2OCH2Cl, NaH, THF - MeOCH2CH2OCH2Cl, CH2Cl2, iPr2EtN Cleavage - Lewis acids such as ZnBr2, TiCl4, Me2BBr2

TL 1976, 809

S B Cl

MEM-O

HO

S

C5H11

O-Si(Ph)2tBu

O

C5H11

TL 1983, 24 , 3965, 3969 O

O-Si(Ph)2tBu

- can also be cleaved in the presence of THP ethers Methyl Thiomethyl Ethers (MTM) R-OH → R-OCH2SMe

Stable to base and mild acid

Formation: - MeSCH2Cl, NaH, THF Cleavage: - HgCl2, CH3CN/H2O - AgNO3, THF, H2O, base Benzyloxymethyl Ethers (BOM) R-OH → R-OCH2OCH2Ph

Stable to acid and base

Formation: - PhOCH2CH2Cl, CH2Cl2, iPr2EtN Cleavage: - H2/ PtO2 - Na/ NH3, EtOH Tetrahydropyranyl Ether

R-OH

(THP)

O H

+

, PhH

Formation Cleavage:

R

O

O

Stable to base, acid labile

- DHP (dihydropyran), pTSA, PhH - AcOH, THF, H2O - Amberlyst H-15, MeOH

Ethoxyethyl ethers (EE) JACS 1979, 101 , 7104; JACS 1974, 96 , 4745. R-OH

O H+

R

Benzyl Ethers (R-OBn) R-OH → R-OCH2Ph Formation: Cleavage:

O

O

(R-OEE)

base stable, acid labile

stable to acid and base

- KH, THF, PhCH2Cl - PhCH2OC(=NH)CCl3, F3CSO3H - H2 / PtO2 - Li / NH3

JCS P1 1985, 2247

58

PROTECTING GROUPS 2-Napthylmethyl Ethers (NAP) JOC 1998, 63, 4172 formation: 2-chloromethylnapthalene, KH cleavage: hydrogenolysis OH

O

ONAP

BnO

OH

H2, Pd/C

-

OH

BnO

(86%)

p- Methoxybenzyl Ethers (PMB) Formation: - KH, THF, p-MeOPhCH2Cl - p-MeOPhCH2OC(=NH)CCl3, F3CSO3H Cleavage:

O

59

TL 1988, 29 , 4139

H2 / PtO2 Li / NH3 DDQ Ce(NH4)2(NO3)6 (CAN) e-

o-Nitrobenzyl ethers Review: Synthesis 1980, 1; Organic Photochemistry, 1987, 9 , 225 O 2N

NaH, THF R-OH Cl

R

O

NO 2

Cleavage:

- photolysis at 320 nm NO 2

HO HO HO

O

HO hν, 320 nm, pyrex, H2O

O

HO HO

OH

O

OH JOC 1972, 37 , 2281, 2282.

OH

p-Nitrobenzyl Ether TL 1990, 31 , 389 -selective removal with DDQ, hydrogenolysis or elctrochemically 9-Phenylxanthyl- (pixyl, px)

TL 1998, 39, 1653

Formation:

Ph

Ph

Cl

OR

pyridine

ROH

+ O

Removal:

Ph

O Ph

OR

OH

hν (300 nm)

ROH O

CH3CN,H 2O

+ O

Trityl Ethers -CPh3 = Tr R-OH → R-OCPh3 - selective for 1° alcohols - removed with mild acid; base stable formation: - Ph3C-Cl, pyridine, DMAP - Ph3C + BF4Cleavage: - mild acid

PROTECTING GROUPS

60

Methoxytrityl Ethers JACS 1962, 84 , 430 - methoxy group(s) make it easier to remove R1 (p-Methoxyphenyl)diphenylmethyl ether 4'-methoxytrityl MMTr-OR R2

Di-(p-methoxyphenyl)phenylmethyl ether 4',4'-dimethoxytrityl DMTr-OR

C O R

Tri-(p-methoxyphenyl)methyl ether 4',4',4'-trimethoxytrityl TMTr-OR R3

Tr-OR < MMTr-OR < DMTr-OR << TMTr-OR O

O

HN R O

HN

O

80% AcOH (aq)

N

O

HO

O

20°C

N

O

HO OH

HO OH R = Tr 48 hr. R= MMTr 2 hr. R= DMTr 15 min. R= TMTr 1 min.

(too labile to be useful)

Oligonucleotide Synthesis (phosphoramidite method - Lessinger) Review: Tetrahedron 1992, 48 , 2223 S I L I C A

S I L I C A

OH OH OH

S I L I C A

O Si (CH 2)3 NH 2

O O

DMTrO O

O

DMTrO

O

B O

Cl 3CCOOH S

O

B' O

P

O

CN

O

CN

O

DMTrO

O

O

Base

O

B

O

P

-

O

B

O

coupling S DMTrO

O

O HO

B'

O

S

B'

O

O

O I2, H2O

Si (CH 2)3 N C (CH 2)2 C OH H

HO

B'

N (iPr)2

O

B

Si (CH 2)3 N C (CH 2)2 C O H O

O

O P

O

O

B'

O

O

O

S I L I C A DMTrO

O

P

OO-

Cl 3CCOOH

O

B

O

P O

B O

O S

O

Repeat Cycle

OO

S

O

PROTECTING GROUPS Silyl Ethers Synthesis 1985, 817 Synthesis 1993, 11 Synthesis 1996, 1031 R-OH → R-O-SiR3 formation: - R3Si-Cl, pyridine, DMAP - R3Si-Cl, CH2Cl2 (DMF, CH3CN), imidazole, DMAP - R3Si-OTf, iPr2EtN, CH2Cl2

61

Trimethylsilyl ethers Me3Si-OR TMS-OR - very acid and water labile - useful for transiant protection Triethylsilyl ethers Et3Si-OR TES-OR - considerably more stable that TMS - can be selectively removed in the presence of more robust silyl ethers with with F - or mild acid O OH

H2O/ACOH/THF (3:5:11), 15 hr

TESO OTBS

O

(97%)

Liebigs Ann. Chem. 1986, 1281 OTBS

Triisopropylsilyl ethers iPr3Si-OR TIPS-OR - more stabile to hydrolysis than TMS Phenyldimethylsilyl ethers J. Org. Chem. 1987, 52 , 165 t-Butyldimethylsilyl Ether tBuMe 2Si-OR TBS-OR TBDMS-OR JACS 1972, 94 , 6190 - Stable to base and mild acid - under controlled condition is selective for 1° alcohols t-butyldimethylsilyl triflate tBuMe 2Si-OTf TL 1981, 22 , 3455 - very reactive silylating reagent, will silylate 2° alcohols cleavage: - acid - F- (HF, nBu4NF, CsF, KF) TBSO HO CO2Me

HF, CH3CN

CO2Me

(70%) O

OTBS

O

HO

JCS Perkin Trans. 1 1981, 2055

t-Butyldiphenylsilyl Ether tBuPh2Si-OR TBDPS-OR ∑-OR - stable to acid and base - selective for 1° alcohols - Me3Si- and iPr 3Si groups can be selectively removed in the presence of TBS or TBDPS groups. - TBS can be selectively removed in the presence of TBDPS by acid hydrolysis. TL 1989, 30 , 19

PROTECTING GROUPS cleavage - F- Fluoride sources: -

nBu 4NF (basic reagent) HF / H2O /CH3CN HF•pyridine SiF4. CH2Cl2

TL 1979, 3981. Synthesis 1986, 453 TL 1992, 33 , 2289

Me O Si

OH

tBu Me

JOC 1981, 46 ,1506 TL 1989, 30 , 19.

AcOH / THF/ H2O Ph O Si tBu

O Si

Me

Ph tBu Ph

Ph Me tBu Si

62

Me tBu

O

Si

Me

OTHP

O OH

PPTS / EtOH

JACS 1984, 106 , 3748

Esters R-OH → R-O2CR' Formation: - "activated acid", base, solvent, (DMAP) Activated Acids Chem. Soc. Rev. 1983, 12, 129 Angew. Chem. Int. Ed. Engl. 1978, 17, 569. RCO2H → "activated acid" → carboxylic acid derivative (ester, amide, etc.) Acid Chlorides O

O O

O

N

N

+ N

R R

OH

R

R

N

Cl

+ N

N acyl pyridinium ion (more reactive)

1. SOCl2 2. PCl5 3. (COCl)2 Anhydrides O

O

P2O 5

2 R

OH

R

O O

R

Activating Agents: Carbonyl Diimidazole O

O O R

OH

N

+ N

R

N N

N

N Acyl Imidazole

NH +

CO

+

N

PROTECTING GROUPS

63

Dicyclohexylcarbodiimide C 6H11

O

NH

O R

+

OH

R

N C N

O

O

Nu:

O C N

R

+C 6H11 Nu

N H

N H

C 6H11

C 6H11

Ketene formation is a common side reaction- scambling of chiral centers C 6H11

O

NH

R

R

O C N

H

C O

"ketene"

C 6H11

Hydroxybenzotriazole (HOBT) - reduces ketene formation C 6H11

O R

O

N

NH

+

O C

N

N

N

O N

R

N

N

OH

C 6H11

N-Hydroxysuccinimide (NHS) O

C 6H11

O R

O

O

NH

HO

+

O C

N R

N

O N

O

C 6H11

O

2,2'-Dipyridyl Disulfide (Aldrithiol, Corey Reagent) Aldrichimica Acta 1971, 4 , 33 O R

OH

O

Ph3P:

+ N

S S

R

N

+ S

+

N

N

Ph3P=O

SH

Mukaiyama's Reagent (2-Chloro-1-methyl pyridinium Iodide or 2-Fluoro-1methyl pyridinium p-toulenesulfonate) Aldrichimica Acta 1987, 20 , 54 Chem. Lett. 1975, 1045; 1159; 1976, 49; 1977, 575 O

+ R

OH

F

+ N Me

O

TsO R

O

+ N

I-

Me

Acetates R-OH → R-O2CCH3 - stable to acid and mild base - not compatable with strong base or strong nucleophiles such as organometallic reagents Formation: - acetic anhydride, pyridine - acetyl chloride, pyridine

PROTECTING GROUPS Cleavage:

-

K2CO3, MeOH, reflux KCN, EtOH, reflux NH3, MeOH LiOH, THF, H2O enzymatic hydrolysis (Lipase) OAc

64

Org. Rxns. 1989, 37, 1. OAc

Porcine Pancreatic Lipase

TL 1988, 30 , 6189

OAc

OH

(96% ee)

Chloroacetates - can be selectively cleaved with Zn dust or thiourea. O Me

O OR

O

HO

AcO Me OAc O

Cl

Me

O OR

Cl

H2NNHCOSH

O

AcO Me

O

JCS CC 1987, 1026 O

OAc O

HO

Cl

O

O

OH

O

Trifluoroacetates Formation: - with trifluoroacetic anhydride or trifluoroacetyl chloride Cleavage: - K2CO3, MeOH Pivaloate (t-butyl ester) - Fairly selective for primary alcohols Formation: - tbutylacetyl chloride or t-butylacetic anhydride Cleavage: - removed with mild base Benzoate (Bz) - more stable to hydrolysis than acetates. Formation: - benzoyl chloride, benzoic anhydride, benzoyl cyanide (TL 1971, 185) , benzoyl tetrazole (TL 1997, 38, 8811) Cleavage: - mild base - KCN, MeOH, reflux 1,2 and 1,3- Diols

Synthesis 1981, 501

Chem. Rev. 1974, 74, 581

R2 O

OH R1

R

OH

Isopropylidenes

R3 H+ , -H2O

R2

R3

O

O

R

R1

(acetonides) H+

OH R

R1 OH

acetone or OMe MeO OMe or

Me

Me

O

O

R

R1

- in competition between 1,2- and 1,3-diols, 1,2-acetonide formation is usually favored - cleaved with mild aqueous acid

PROTECTING GROUPS

65

Cycloalkylidene Ketals - Cyclopentylidene are slightly easier to cleave than acetonides - Cyclohexylidenes are slightly harder to cleave than acetonides O

(CH 2)n -or-

OH

OMe

MeO

(CH 2)n

(CH 2)n

R1

R

O H+ , -H2O

OH

O

R

R1

Benzylidene Acetals PhCHO -orPhCH(OMe)2

OH R1

R

Ph O

O

+

H , -H2O

OH

R

R1

- in competition between 1,2- and 1,3-diols, 1,3-benzylidene formation for is usually favored - benzylidenes can be removed by acid hydrolysis or hydrogenolysis - benzylidene are usually hydrogenolyzed more slowly than benzyl ethers or olefins. p-Methoxybenzylidenes - hydrolyzed about 10X faster than regular benzylidenes - Can be oxidatively removed with Ce(NH 4)2(NO3)6 (CAN) OMe

OBn BnO

O

MeO

BnO

OH

(95%)

O

O

OBn

Ce(NH 4)2(NO3)6 CH 3CN, H2O

MeO

O

OH

Other Reactions of Benzylidenes - Reaction with NBS (Hanessian Reaction) H O

Ph O O HO

NBS, CCl 4

O

Br Ph

HO OMe

O HO

Org. Syn. 1987, 65, 243

O HO OMe

- if benzylidene of a 1° alcohol, then 1° bromide - Reductive Cleavage Ph O

Na(CN)BH3, TiCl4,CH 3CN

O

OH MeO 2C

MeO 2C O

MeO

CO 2Me

O O

Ph O

CO 2Me

Synthesis 1988, 373.

OBn O

TMS-CN BF3•OEt2

OH Tetrahedron 1985, 41, 3867

MeO

O

O H

Ph CN

PROTECTING GROUPS Ph O

O

BnO

OH

DIBAL-H TL 1988, 29 , 4085 O

O

OMe

OMe

Carbonates O

OH R1

R

(Im)2CO

O

OH

O

R

R1

- stable to acid; removed with base - more difficult to hydrolyze than esters Di-t-Butylsilylene (DTBS) TL 1981, 22 , 4999 - used for 1,3- and 1,4-diols; 1,2-diols are rapidly hydrolyzed - cleaved with fluoride (HF, CH 3CN -or- Bu4NF -or- HF•pyridine) - will not fuctionalize a 3°-alcohol OH

(t-Bu)2SiCl 2, Et3N CH 3CN, HOBT

O

tBu Si

O

OH

1,3-(1,1,3,3)-tetraisopropyldisiloxanylidene (TIPDS) - specific for 1,3- and 1,4-diols - cleaved with fluoride or TMS-I

tBu

TL 1988, 29 , 1561

O

O HN

HN HO

O

N

iPr2Si(Cl)-O-Si(Cl)iPr2 pyridine

O

O Si

O

N

O

O Si

HO OH

O OH

Ketones and Aldehydes - ketones and aldehydes are protected as cyclic and acyclic ketals and acetals - Stable to base; removed with H3O+ R

O

MeOH, H+

R1

R

R1 (CH 2OH)2, H+ PhH, -H2O -or(CH2OSiMe 3)2, TMS-OTf, CH2Cl 2 CH 2(CH 2OH)2, H+ , PhH, -H2O

OMe OMe

R

O TL 1980, 21 , 1357

R1 O 1,3-dioxolanes R O R1 O 1,3-dioxanes

66

PROTECTING GROUPS

67

Cleavage rate of substituted 1,3-dioxanes: Chem. Rev. 1967, 67 , 427. R O

R O

R O

> R1 O

>> R1 O

R1 O

- Ketal formation of α,β-unsaturated carbonyls are usually slower than for the saturated case. O

O O

CH 2(CH 2OH)2, H+ , PhH, -H2O

O

O

Fluoride cleavable ketal: O

O LiBF4

O

O

(88%) O

Me3Si

TL 1997, 38, 1873

O O

Base cleavable ketal: HO

O

SO2Ph

SO2Ph

DBU, CH2Cl 2

OH R1

R2

O pTSA, C6H6

R1

O

TL 1998, 39, 2401

O R1

R2

R2

Carboxylic Acids Tetrahedron 1980, 36, 2409. Tetrahedron 1993, 49, 3691 Nucelophilic Ester Cleavage: Organic Reactions 1976, 24, 187. Esters Alkyl Esters formation: - Fisher esterification (RCOOH +R'OH + H+) - Acid Chloride + R-OH, pyridine - t-butyl esters: isobutylene and acid - methyl esters: diazomethane Cleavage: - LiOH, THF, H2O - enzymatic hydrolysis Org. Rxns. 1989, 37, 1. - t-butyl esters are cleaved with aqueous acid - Bu 2SnO, PhH, reflux (TL 1991, 32, 4239) OH MeO 2C

CO 2Me

O R

Pig Liver Esterase pH 6.8 buffer

Bu2SnO, PhH, ↑↓ OR'

OH MeO 2C

CO 2H

O R

OH

TL 1991, 32, 4239

R= Me, Et, tBu

9-Fluorenylmethyl Esters (Fm) TL 1983, 24 , 281 - cleaved with mild base (Et2NH, piperidine) DCC RCO 2H

+ O

OH R

O

PROTECTING GROUPS 2-Trimethylsilyl)ethoxymethyl Ester (SEM) HCA 1977, 60 , 2711. - Cleaved with Bu 4NF in DMF DCC RCO 2H

HO

+

O

R

SiMe 3

O

O

SiMe 3

O

- Cleaved with MgBr2•OEt2 TL 1991, 32, 3099. 2-(Trimethylsilyl)ethyl Esters JACS 1984, 106 , 3030 - cleaved with Fluoride ion DCC RCO 2H

HO

+

R

O

SiMe 3

SiMe 3 O

Haloesters

- cleaved with Zn(0) dust or electrochemically DCC RCO 2H

HO

+

CCl 3

R

O

CCl 3

O

Benzyl Esters RCO2H + PhCH2OH → RCO2Bn Formation: Cleavage: -

DCC Acid chloride and benzyl alcohol Hydrogenolysis Na, NH3

Diphenylmethyl Esters DCC RCO 2H

HO

+

R

O

CHPh 2

CHPh 2 O

Cleavage:

- mild H3O+ - H2, Pd/C - BF3•OEt2

o-Nitrobenzyl Esters - selective removed by photolysis Orthoesters Synthesis 1974, 153 TL 1983, 24 , 5571

Chem. Soc. Rev. 1987, 75

O RCOCl

O

BF 3•OEt2

+ R OH

- Stable to base; cleaved with mild acid

O R

O O

O O

68

PROTECTING GROUPS

69

Amines Carbamates 9-Fluorenylmethyl Carbamate (Fmoc) Acc. Chem. Res. 1987, 20 , 401 - Cleaved with mild base such as piperidine, morpholine or dicyclohexylamine NaHCO 3 H2O, dioxane

+

R2NH

O

O Cl

O

R2N

O

2,2,2-Trichloroethyl Carbamate O

O Cl 3C

O

R2NH, pyridine Cl 3C

Cl

O

N

R

R

- Cleaved with zinc dust or electrochemically. O N

O

CCl 3

S

Te Te S

TL 1986, 27 , 4687

NaBH 4

EtO2C

NH EtO2C

S

S

2-Trimethylsilylethyl Carbamate (Teoc) - cleaved with fluoride ion. O

O Me 3Si

O

O +

O N

Me 3Si

R2NH

O

N

R R

O SiMe 3 OH Cl

O

MeO

OTBS

O N

Cl Bu4NF, THF

O CH 3

H N

MeO

O CH 3

(100%)

H O SEt

H O SEt

OEt

OEt

SEt

SEt JACS 1979, 101 7104

t-Butyl Carbamate

(BOC) O

O O

R2NH

Cleavage:

tBuO

O

OtBu R2N

OtBu

- with strong protic acid (3M HCl, CF3COOH) - TMS-I O B Cl

Allyl Carbamate

(Alloc)

O

TL 1985, 26 , 1411 TL 1986, 27 , 3753

PROTECTING GROUPS O O

R2NH

O O

O O R2N

O

- removed with Pd(0) and a reducing agent (Bu3SnH, Et 3SiH, HCO2H) O HN

1) Pd(OAc) 2, Et3N, Et3SiH 2) H3O +

O CO 2Me

Benzyl Carbanate

NH 2 TL 1986, 27 , 3753

CO 2Me

(Cbz) O BnO

O

Cl

R2NH

Cleavage:

-

R2N

O

Ph

Hydrogenolysis PdCl 2, Et3SiH TMS-I BBr3 hν (254 nm) Na/ NH3

m-Nitrophenyl Carbamate JOC 1974, 39 , 192 NO 2

O R2N

O

- removed by photolysis Amides Formamides - removed with strong acid R2NH

+

O

HCO 2Et R2N

H

Acetamides - removed with strong acid R2NH

+

O

Ac2O R2N

Trifluoroacetamides Cleavage: - base (K2CO3, MeOH, reflux) - NH3, MeOH R2NH

Sulfonamides p-Toluenesulfonyl

+

O

(CF3CO) 2O R2N

(Ts) R2NH

pTsCl, pyridine R2N

CH 3

SO 2

CF 3

70

PROTECTING GROUPS Cleavage:

Ts

- Strong acid - sodium Naphthalide - Na(Hg)

N

N

Na(Hg), MeOH Na2HPO 4

Ts

H

N

N

H JOC 1989, 54 , 2992

(65%) N

N

Ts

H

Trifluoromethanesulfonyl Tf

Tf N

N

N

NH

Na, NH3

HN

JOC 1992, 33, 5505 NH

N

HN

Tf

Tf

Trimethylsilylethanesulfonamide (SES) TL 1986, 54 , 2990; JOC 1988, 53, 4143 - removed with CsF, DMF, 95°C SO 2Cl

Me 3Si

R2NH

Et3N, DMF

R2N O

SiMe 3

S O

tert-Butylsulfonyl (Bus) JOC 1997, 62, 8604

R-NH2

tBuSOCl, Et3N, CH2Cl2

mCPBA -orRuCl3, NaIO4

O R 2N

S

tBu

R2N SO2tBu

CF3SO3H, CH2Cl2, anisole

R-NH2

71

C-C BOND FORMATION

72

Carbon- Carbon Bond Formation 1. Alkylation of enolates, enamines and hydrazones C&S: Chapt. 1, 2.1, 2.2 problems Ch 1: 1; 2; 3, 7; 8a-d; 9; 14 Ch. 2: 1; 2; 4) Smith: Chapt. 9 2. Alkylation of heteroatom stabilized anions C&S :Chapt. 2.4 - 2.6) 3. Umpolung Smith: Chapt. 8.6 4. Organometallic Reagents C&S: Chapt. 7, 8, 9 problems ch 7: 1; 2; 3, 6; 13 Ch. 8: 1; 2 Smith: Chapt. 8 5. Sigmatropic Rearrangements . C&S Chapt. 6.5, 6.6, 6.7 # 1e,f,h,op Smith Chapt. 11.12, 11.13 Enolates Comprehensive Organic Synthesis 1991, vol. 2, 99. - α-deprotonation of a ketone, aldehyde or ester by treatment with a strong nonnucleophillic base. - carbonyl group stabilizes the resulting negative charge. O H H

O-

O

B:

H -

R H

H

R

H

R H

- Base is chosen so as to favor enolate formation. Acidity of C-H bond must be greater (lower pKa value) than that of the conjugate acid of the base (C&S table 1.1, pg 3) O H 3C O H 3C

CH3

pKa = 20

unfavorable enolate concentration

MeO- pKa = 15 tBuO- pKa = 19

more favorable enolate concentration

O CH2

OEt

pKa = 10

- Common bases: NaH, EtONa, tBuOK, NaNH2, LiNiPr2, M N(SiMe3)2, Na CH2S(O)CH3 Enolate Formation: - H+ Catalyzed (thermodynamic) O

OH H+

- Base induced (thermodynamic or kinetic) O

:B

O-

+

B:H

H

Regioselective Enolate Formation Tetrahedron 1976, 32, 2979. - Kinetic enolate- deprotonation of the most accessable proton (relative rates of deprotonation). Reaction done under essentially irreversible conditions. O - Li+

O LDA, THF, -78°C

C-C BOND FORMATION typical conditions: strong hindered (non-nucleophilic) base such as LDA R2NH pKa= ~30

73

Li

N

Ester Enolates- Esters are susceptible to substitution by the base, even LDA can be problematic. Use very hindered non-nucleophillic base (Li isopropylcyclohexyl amide) O

O OR'

LDA, THF, -78°C N

E+

R

R

O

O- Li+ N Li

OR'

R

OR'

THF, -78°C

R

- Thermodynamic Enolate- Reversible deprotonation to give the most stable enolate: more highly substituted C=C of the enol form O - K+

O - K+

O tBuO- K+,

tBuOH

kinetic

thermodynamic

typical conditions: RO- M+ in ROH , protic solvent allows reversible enolate formation. Enolate in small concentration (pKa of ROH= 15-18 range) - note: the kinetic and thermodynamic enolate in some cases may be the same - for α,β-unsaturated ketones O

thermodynamic site

kinetic site

Trapping of Kinetic Enolates - enol acetates 1) NaH, DME 2) Ac2O

Ph O

Ph

+

Ph O

O

kinetic

O

isolatable separate & purify

CH3Li, THF

Regiochemically pure enolates

O

CH3Li, THF

Ph

Ph O- Li+

O- Li+

- silyl enolethers

Synthesis 1977, 91. 1) LDA 2) Me3SiCl

Ph O

C-C BOND FORMATION Acc. Chem. Res. 1985, 18, 181.

Ph

+

Ph OTMS

OTMS

kinetic

isolatable separate & purify CH3Li, THF -orBu4NF -or- TiCl4 Ph

Ph

Geometrically pure enolates

CH3Li, THF

O- M+

O- M+

- tetraalkylammonium enolates- "naked" enolates - TMS silyl enol ethers are labile: can also use Et3Si-, iPr3Si- etc. - Silyl enol ether formation with R 3SiCl+ Et3N gives thermodyanamic silyl enol ether - From Enones 1) MeLi 2) E+

1) Li, NH3 2) TMS-Cl O

O

TMSO

OSiMe3

H

H

E

O

OSiMe3

TMS-Cl, Et3N

TMS-OTf Et3N

O

OSiMe3 Li, NH3, tBuOH TMS-Cl

- From conjugate (1,4-) additions O

O- Li+

O E+

(CH3)2CuLi

E

Trap or use directly

- From reduction of α-halo carbonyls O Br

Zn or Mg

O- M+

Alkylation of Enolates (condensation of enolates with alkyl halides and epoxides) Comprehensive Organic Synthesis 1991, vol. 3, 1. 1° alkyl halides, allylic and benzylic halides work well 2° alkyl halides can be troublesome 3° alkyl halides don't work

74

C-C BOND FORMATION O

75

O a) LDA, THF, -78°C b) MeI

Me

- Rate of alkylation is increased in more polar solvents (or addition of additive) O (Me2N)3P

R NMe2 R= H DMF R-CH3 DMA

HMPA

O

O

O

S

H 3C

O

CH3N

CH3

CH3N

NMe2 NCH3 Me2N

DMSO

TMEDA

Mechanism of Enolate Alkylation: SN2 reaction, inversion of electrophile stereochemistry X C

180 ° M+ -O

Alkylation of 4-t-butylcyclohexanone: O

O R

E R

equitorial anchor

E H

H

A

E tBu

favored

A

Chair

tBu

R

R

B

O

O- M+ H O

E tBu

B

Twist Boat

R E

on cyclohexanone enolates, the electrophile approaches from an "axial" trajectory. This approach leads directly into a chair-like product. "Equitorial apprach leads to a higher energy twist-boat conformation. Alkylation of α,β-unsaturated carbonyls O- M+ R1 O

R2 Kinetic

R1

E

O R1

H

R2 E

H

R2 H

H

O- M+ R1

O R2

H

E

R1

R2 H

Thermodynamic

E

C-C BOND FORMATION

76

Stork-Danheiser Enone Transposition: - overall γ-alkylation of an α,β-unsaturated ketone O

O

LDA PhCH2OCH2Cl

HO CH3 CH3Li

PhO

OMe

H3O

PhO

CH3

+

PhO

OMe

OMe

O J. Org. Chem. 1995, 60, 7837.

Chiral enolates- Chiral auxilaries. D.A. Evans JACS 1982, 104 , 1737; Aldrichimica Acta 1982,15 , 23. Asymmetric Synthesis 1984, 3, 1. - N-Acyl oxazolidinones O

O R

H 2N

OH

Me

Ph

O

N Me

Ph

norephedrine

O

O

H 2N

R

OH

O

N

valinol O

O R

R

LDA, THF

O

N

O

O N

O LiOH, H2O, THF

O

R

OH

Et-I Me

Ph O

O R

N

O

Complimentary Methods for enantiospecific alkylations

Me Ph major product (96:4) O O LDA, THF

R

N

O LiOH, H2O, THF

O

R

OH

Et-I

Diastereoselectivity: 92 - 98 % for most alkyl halides

major product (96:4)

Enolate Oxidation Chem. Rev. 1992, 92, 919. R

O

O

O N

O

NaN(SiMe3)2, THF, -78°C

R

N

N

(88 - 98 % de)

SO2Ph O

LDA, THF

R

O tBuO

O

OH

O Ph

O

Boc N

N

OtBu O

O N

N HN Boc

O

1) HO2) CH2N2 3) TFA 4) Raney Ni

(94 - 98 % de)

O R

OMe NH2

C-C BOND FORMATION O R

Bu

Bu

O

B

N

O

Bu2BOTf, Et3N

O

O N

N

O

N

1) LiOH 2) H2, Pd/C

O

O R

N3

R

D- amino acids

O

O N

R

KN(SiMe3)2, THF

O

OH NH2

Ph

O

O

Ph

Ph

O

O

N3-

Br

Ph

R

O

O

NBS R

R

77

O N

O

N3

SO2N3

Ph

Ph

Oppolzer Camphor based auxillaries Tetrahedron, 1987, 43, 1969. diastereoselectivities on the order of 50 : 1 SO2Ph N O

R

Ar Ar

N O

R

O SO2Ph O

R N

O SO2N(C6H11)2

O

S O2

R H O

H

LDA, NBS

Et2Cu•BF3

O

O SO2N(C6H11)2

O

H

O Br

O SO2N(C6H11)2

HO

O SO2N(C6H11)2

Asymmetric Acetate Aldol O

S

O N

O

TIPSO

H

1) Br

Sn(OTf)2, CH2Cl 2, R3N, -40°C 2) TIPS-OTf, pyridine 3) NH3

O

Br

NH2

J. Am. Chem. Soc. 1998, 120, 591 J. Org. Chem. 1986, 51, 2391

85 %, 19:1 de

Chiral lithium amide basess CH3 MeO CO2Et OMe

Ph

N Li THF, -78°C (CH3)2C=O

CH3

OMe MeO

O OMe O

NH2 O

(72% ee)

C-C BOND FORMATION H N

O

Ph N

H

But O

N Li

Li N (97 % ee)

THF, HMPA TMSCl

tBu

OTMS

Ph

N

tBu

N Me

Lewis Acid Mediated Alkylation of Silyl Enolethers- SN1 like alkylations OTMS

O

tBu-Cl, TiCl4, CH2Cl2, -40°C

CH3

(79%) SPh

OTMS R

note: alkylation with a 3° alkyl halide

C(CH3)3

O

Cl

SPh

ACIEE1978, 17, 48 TL 1979, 1427

O Raney Ni

R

R

(95 %)

TiCl4, CH2Cl2, -40°C (78%)

Enamines Gilbert Stork Tetrahedron 1982, 38, 1975, 3363. - Advantages: mono-alkylation, usually gives product from kinetic enolization O

O

N

N

"Thermodynamic"

"Kinetic"

O O N H

can not become coplanar

O

O

•• N

+ N

R-I

R

H2O

O E

H+, (-H2O) enamine

-Chiral enamines O

N

Imines

E

Isoelectronic with ketones Me O

Ph N

Li

OMe

N LDA, THF, -20°C

Ph

1) E 2) H3O+

O E

E = -CH3, -Et, Pr, PhCH2-, allylee 87 - 99 %

78

Hydrazones

C-C BOND FORMATION isoelectronic with ketones Comprehensive Organic Synthesis 1991, 2, 503 O

N

N

N

Me2N-NH2

-N

N

LDA, THF

79

N

-

+

H , (-H2O)

N

E+

N

O

hydrolysis E

E

- Hydrazone anions are more reactive than the corresponding ketone or aldehyde enolate. - Drawback: can be difficult to hydrolyze. - Chiral hydrazones for asymmetric alkylations (RAMP/SAMP hydrazones- D. Enders "Asymmetric Synthesis" vol 3, chapt 4, Academic Press; 1983) OMe

MeO

N

N

H 2N

SAMP

RAMP

N

LDA

OMe

N

N

NH2

O

O3

OMe

N

H I

OTBS

(95 % de)

TBSO

TBSO

N

1) LDA 2) Ts-CH3, THF -95 - -20 °C OMe 3) MeI, 2N HCl

N

O CH3 (100 % ee)

Me O Li R1 E (C,C)

MeO

••

N N

R2

H

R1

N N

R2

Z (C,N)

H E

E

Aldol Condensation

Comprehensive Organic Synthesis 1991, 2, 133, 181. O

H

R

a) LDA, THF, -78°C b R'CHO

O

β-hydroxyl aldehyde (aldol)

OH

H

R' R

- The effects of the counterion on the reactivity of the enolates can be important Reactivity Li+ < Na+ < K+ < R4N+ addition of crown ethers

C-C BOND FORMATION - The aldol reaction is an equilibrium which can be "driven" to completion. M

O- M+

O +

R

RCHO

O

H

R'

O

work-up

OH

H

R'

80

R' R

R

In the case of hindered enolates, the equillibrium favors reactants. Mg2+ and Zn2+ counterions will stabilize the intermediate β-alkoxycarbonyl and push the equillibrium towards products. (JACS 1973, 95, 3310) O

O- M+

OH

PhCHO, THF

M= Li M= MgBr

Ph

16% yield 93% yield

- Dehydration of the intermediate β-alkoxy- or β-hydroxy ketone can also serve to drive the reaction to the right. O

O

O

tBuO- Na +, tBuOH O

JACS 1979, 101 , 1330 O

H O

H O

Enolate Geometry - two possible enolate geometries O - Li+

O - Li+

O LDA, THF, -78°C

H

+ H Z - enolate

E - enolate

- enolate geometry plays a major role in stereoselection. OM

Z -enolate R

R2

1

O

R3CHO R

R

OM R

H

1

3

erythro (syn)

R2

H

E -enolate

OH

1

O

R3CHO R

OH

1

R

R2

R

3

threo (anti)

2

- Zimmerman-Traxler Transition State : Ivanov condensation JACS 1957, 79 , 1920. +

O -

Ph

H

+

MgBr - +

PHCHCO2 MgBr

Br

H O Ph

O Mg

Ph H

"pericyclic" T.S.

OMgBr

C-C BOND FORMATION

81

Analysis of Z-enolate stereoselectivity R2 O

R3

O

M

R2

R2 O

R3

M

O

R3

O

M

O

O H

H

R1

H

R1

H

H

R3

R1

H

R1

OH

R2

erythro (syn) favored R2 O

H

R2 O

H

R2

M

O

M R1

R3

O

M

O

R3

R1

R3

R1

OH

R1

H

R3

H

H

O

H

O

R2

threo (anti) disfavored

Analysis of E-enolate stereoselectivity R3

H O

R3

M R

O

3

O R2

H

R1

H

O

O

M

R2

H

H

R2

H

R

OH

R1

R3 R2

threo (anti) favored H

H O

O

R1

H

R1

O

M

O

M

H

H

O

M

O

H

O

2

R1

R3

R2

R2

R3

R3

R1

O

M

O

OH

R1

R1

R3 R2

erthro (syn) disfavored

Analysis of Boat Transition State for Z-Enolates R2 O

R3

O

O

M

R3

H R1

H

R3

R1

H

O

HO O

M

R2

R2 R1

H

Favored Chair

Boat H O

R2 O

O

O

M

H

HO

R1 H R3

R1

Disfavored Chair

R3 R3

R2

staggered

O R2 R1 H Boat: R1-R2 1,3-interaction is gone

M

C-C BOND FORMATION

82

Analysis of Boat Transition State for E-Enolates R3

H O

M

R3

O

O O

HO

R1

R2

R1

H

H R3

O

M

H

R2

R1

R2

Favored Chair

Boat H

H O

O O

M

O

HO

R3

H R1

R2

R3

O

M

H

R2

R1

R3

staggered

R1

R2

Disfavored Chair

Boat: R1-R2 1,3-interaction is gone

Summary of Aldol Transition State Analysis: 1. Enolate geometry (E- or Z-) is an important stereochemical aspect. Z-Enolates usually give a higher degree of stereoselection than E-enolates. 2. Li+, Mg 2+, Al3+= enolates give comparable levels of diastereoselection for kinetic aldol reactions. 3. Steric influences of enolate substituents (R1 & R2) play a dominent role in kinetic diastereoselection. O- M+

O

Path A R2

R1

R1

R3

Path B

H

R2

O- M+

O H

R1

HO

R1

Path A

R2

HO R3 R2

When R1 is the dominent steric influence, then path A proceeds. If R2 is the dominent steric influence then path B proceeds. 4. The Zimmerman-Traxler like transition state model can involve either a chair or boat geometry. Noyori "Open" Transition State for non-Chelation Control Aldols Absence of a binding counterion. Typical counter ions: R4N+, K+/18-C-6, Cp2Zr2+ - Non-chelation aldol reactions proceed via an "open" transition state to give syn aldols regardless of enolate geometry. Z- Enolates: R1

O-

R1 Favored

H H

R3

R2

R3

O-

R1

H R3 O

R2

H

H

R2

HO

R1

Favored R1 O-

O-

R3 R2

Syn Aldol O

H

HO

R3

H H R R2 3 O

O

O

Disfavored H

R3 H R2

O

O R1

H

O-

R1

O-

R1 H

R2 O

Disfavored

R3 R2

Anti Aldol

C-C BOND FORMATION

83

E- Enolate: -

O

-

R1

-

R1

O

O

favored H R3

R3

R2

H

R3

H

R1

-

O

H R3

O

Syn Aldol

R1

R1

O H

R2

HO

R3

H H R3 R2

R1 H

O

O

R3 R2

R2

O favored

disfavored H

R1

H

-

O

HO

H

H R2

O

O -

R1

O

O disfavored

R3 R2

R2

Anti Aldol

NMR Stereochemical Assignment. Coupling constants (J) are a weighted average of various conformations. H

O

O

R1

Syn Aldol JAB = 2 - 6 Hz

HB R3

R2 HA 60 °

60 °

HA H O

O

HB

HB

R3

R2

R2 O

R3

OH

R3 O

R1

R2 R1

HA

HA

H

R1

O

HB

non H-bonded

O

H

OH B

R1

Anti Aldol JAB = 1 - 10 Hz

R3 R2 HA

60 ° HA H O

O

R3

R3

OH

O

R2

R2 O

HB

HA

HB

HB

R1

R2 R1

60 °

HA

H

R1

O

R3

non H-bonded

Boron Enolates:

Comprehensive Organic Synthesis 1991, 2, 239. Organic Reactions 1995, 46, 1; Organic Reactions 1997, 51, 1. OPPI 1994, 26, 3. - Alkali & alkaline earth metal enolates tend to be aggregates- complicates stereoselection models. - Boron enolates are monomeric and homogeneous - B-O and B-C bonds are shorter and stronger than the corresponding Li-O abd Li-C bonds (more covalent character)- therefore tighter more organized transition state. Generation of Boron Enolates: O

R2B-X iPrEtN

OBR2

X= OTf, I R= Bu, 9-BBN

C-C BOND FORMATION R3N:

_

H R1

+ BL2OTf O R2

H

R3N:

OBL2

Z-enolate

OBEt2

_

H

+ BL2OTf O H

R1 R2

R2

R1

R1 R2 E-enolate

O

OBR2

R 3B R OSiMe3

OBR2

R2B-X

+ Me3 Si-X

O

OBR'2

R' 3B N2

R

Hooz Reaction

R'

R

Diastereoselective Aldol Condensation with Boron Enolates O

O

OBEt2

Ph

Ph

Ph pure Z-enolate

R2

R1

OH

R3CHO R1

R3 R2

Z-enolate OBEt2

O R1

R3 R2

R2

generally > 95 : 5 syn : anti

OH

R3CHO

R1

R

100% Syn Aldol

O

OBEt2

OBEt2

RCHO

generally ~ 75 : 25 anti : syn

E-enolate

Asymmetric Aldol Condansations with Chiral AuxilariesD.A. Evans et al. Topics in Stereochemistry, 1982, 13 , 1-115. - Li+ enolates give poor selectivity (1:1) - Boron and tin enolates give much improved selectivity Bu

Bu O Me

B

O N

Bu2BOTf, EtNiPr2 , -78°

O

O - O + N

O

OH RCHO

R

O

O N

Me

> 99:1 erythro O

O

1) Bu2BOTf, EtNiPr2 , -78°

Me N

O

2) RCHO Ph

OH R

O X

Me

O

84

C-C BOND FORMATION L

L B _

O

H +

O

R

L

L B _

O

O

O

N

RCHO

H R

O

B _

O

+

O

R

O

N

L

L

B _

O

O

+

N

L

L

L

L

O

O

B _

O

O

+

+

R

N

N

O

O

O

O

preferred conformation R2 O

O L

H O

B

R3 L

R3 N

H

L

H R3

O

B

N

L

O

O

O

O

Favored O O

R2

Disfavored

O

O

OH

N

O

R3

O

OH

N

R2

R3 R2

Oppolzer Sultam L 2B

O R2

N

O R2

N

S O2

S O2

O S O

N

O

S O2

OH

O

R3CHO R2

R3

N S O2

R2

R3

N

1) LDA 2) Bu3SnCl R3 Sn

OH

O R3CHO R2

85

C-C BOND FORMATION Chiral Boron BOTf

O

OH

StBu

O

Ph

OH StBu

iPrEt2N, PhCHO,-78°C

when large, higher E-enolate selectivity

O

ArO2SN SPh

Ph

StBu

1 : 33 (> 99 % ee)

Ph

Ph

R

+

O

B Br

NSO2Ar

OH

O

Ph

OH SPh

+

Ph

SPh

R

iPrEt2N, PhCHO,-78°C

O

R

> 95 : 5 (> 95 % ee)

• In general, syn aldol products are achievable with high selectivity, anti aldols are more difficult Mukaiyama-Aldol- Silyl Enol Ethers as an enolate precursors. Lewis acid promoted condensation of silyl ketene acetals (ester enolate equiv.) with aldehydes: proceeds via "open" transition state to give anti aldols starting from either E- or Z- enolates. OSiMe3

RCHO, TiCl4, CH2Cl2, -78°C

OH

OH CO2Et

R OEt

+

CO2Et

R

CH3

CH3

R= iPr (anti : syn) = 100 : 0 C6H11 94 : 6 Ph 75 : 25

OSiMe3

RCHO, TiCl4, CH2Cl2, -78°C

OH R

OEt

OH CO2Et

+

CO2Et

R

CH3

CH3

R= iPr (anti : syn) = 52 : 48 C6H11 63 : 37 Ph 67 : 33

Asymmetric Mukiayama Aldol: Ph H 3C

O OSiMe3 NMe2

RCHO, TiCl4, CH2Cl2, -78°C

OH R

O

OH Rc

+

R

O Rc

(90-94% de) syn : anti = 85 : 15 selectivity insenstivie to enolate geometry

86

C-C BOND FORMATION Ph N SO Ph 2 O

O

iPrCHO, TiCl4, CH2Cl2, -78°C

HO 96 % de anti : syn = 93 : 7

Rc

OSitBuMe2

CH3

O

RCHO, TiCl4, CH2Cl2, -78°C

O

87

HO + Syn product

Rc CH3

OSitBuMe2 SO2N(C6H11)2

E-Enolate R= Ph % de= 90 nPr 85 iPr 85

anti : syn = 91 : 19 94 : 6 98 : 2

Z-Enolate R= iPr % de= 87

anti : syn = 97 : 7

Mukaiyama-Johnson Aldol- Lewis acid promoted condensation of silyl enol ethers with acetals: OSiMe3

OH

O TiCl4 or SnCl4

Mukaiyama-Johnson Aldol

R RCHO or RCH(OR')2 CH2Cl2, -78°C

via Ti or Sn enolate

O

O TiCl4, CHCl2, -78 °C

O

O

O HO

O

O O

OTMS

+ Cl4Ti

O+

O

Cl4Ti

O

O

OSiMe3

OTMS Ph

TiCl4, (CH3)2C(OEt)2 (78 %)

O

OEt

Ph

Fluoride promoted alkylation of silyl enol ethers

Acc. Chem. Res. 1985, 18, 181 O

OSiMe3 nBu4NF, THF, MeI

C-C BOND FORMATION

88

Meyer's Oxazolines: O

(ipc)2BOtf iPrEt2N, Et2O

N

1) RCHO 2) 3N H2SO4 3) CH2N2

O

H 3C

CO2Me

OH

(ipc)2B Ester equiv.

R

+

CO2Me

(~ 30%)

N

H 3C

R

OH

R= nPr %ee (anti) = 77 anti : syn = 91 : 9 C6H11 84 95 : 5 tBu 79 94 : 6

Anti-Aldols by Indirect Methods: SePh O PhSe

C6H11

1) (C5H7)2BOTf R3N

1) TBS-Cl 2) DiBAl-H

3) NaIO4 4) CH2N2

O

CO2Me

HO

CH3

O3

R

R

HO

Anti Aldol Product

CHO

OTBS CO2Me

OTBS O 1) LDA, THF, -78 °C 2) RCHO

N

OH

1) HIO6 2) CH2N2

O

N

MeO2C

R CH3

R CH3

Anti Aldol O

MOMO O

O MeO

O

MOMO 1) LDA, THF, -78 °C

N O

R

syn aldol CH3

3) TsCl 4) Ba(CN)BH3

C6H11

HO

chiral auxillary

CO2Me

R

R

2) RCHO

OTBS

1) HF 2) [O]

OTBS

N

N

KBEt3H, Et2O, -78 °C

O

MeO

CH3 CH3

CH3

syn : anti 1 : 99

OMOM

CH3 OMOM

2) RCOCl

HO

O

MOMO Zn(BH4)2

HO

N

CH3

syn : anti 97 : 3

CH3 OMOM

Syn Aldols by Indirect Methods: O O

O N

O

1) LDA, THF, -78 °C

O

O

O

Zn(BH4)2 O

2) RCOCl

O N

R CH3

O

OH

N

R CH3

syn : anti = 100 : 1

C-C BOND FORMATION Aldol Strategy to Erythromycin: O 9

10

8

11

OH

12 13 O 1

O

3

2

1

6

OH

4

Erythromycin seco acid

CO2H

5

15 14

4 7

OH

OH

O

OH

OH

3

OH

2

[O]

[O] syn aldol

Erythromycin aglycone

3

CHO

CO2H

OH

O

syn aldol

OH syn aldol

4

+

CHO

OH 1

CHO

CHO O

+ CO2H

OH syn aldol

CHO

2

+

CHO

O 1 HO2C

O

O

O

O

LDA, CH3CH 2COCl

N

O

O

OH

OH

O

OH

3

5

9

11

13

O

O

O

TiCl4, iPr 2EtN, CH2Cl2 O

N 1

83%, (96:4)

O

N

90% (> 99:1)

O

O

1) 9-BBN, THF 2) Swern oxid.

3

5

73% (85:15)

O O

O

5

O

O

Ph

O CHO

N

Ph

Ph

O

1) Zn(BH3)2 2) (H3C)2C(OMe)2 CSA

OH

CHO Ph

O

N

O

(100%)

Ph

O

OH

O

O

O

Sn(OTf)2, Et3N, CH2Cl2 O

N CH3CH 2CHO

O

O

OH

N 9

11

13

1) Na BH(OAc)3 2) TBS-OTf, 2,6-lutidine 3) AlMe3, (MeO)MeNH•HCl MeO 72% (>99:1)

84% (> 96:4) Ph O

OH

OTBS

11

13

EtMgBr 86%

8

Ph 1) PMBC(NH)CCl3 TfOH 2) (PhMe2Si)2NLi, TMS-Cl 48 %

PMBO TMSO

OTBS

O N CH3

OH

OTBS

89

C-C BOND FORMATION X

L

H3C H Sn H O L O

X O

O

O

X

H

O O

O N

O 3

H3C CH3

O

CHO 7

O

R

H H

anti-syn

+

O O

13

1) Zn(BH3)2 2) DDQ

O

O

OH

O

N

OTBS

O

O

O

OH

PMBO O

3

5

7

9

13

70%

O

O

O

p-MeOC 6H4 O

OTBS

1) LiOOH 2) TBAF

O

N

O

O

O

O

OH

Cl3C6H2COCl iPr2EtN, DMAP

HO 63%

Ph

11

OTBS

1) NaH, CS2, MeI 2) nBu3SnH, AIBN

N

Ph p-MeOC 6H4 O

O

Ph O

95%

O

O L

83% (95:5)

p-MeOC 6H4 O

L

Ti O

J. Am. Chem. Soc. 1990,112, 866

L

H3C CH3

BF3•OEt2, CH 2Cl2, -78 °C

OTBS

11

Ph O

H CH3 CH3

X

CH3 CH3

8

O

Disfavored

O

H3C H

OH

X

PMBO TMSO

5

O

Disfavored

O

O

L

CH3

H

anti-syn

H L CH3 Sn H O O L

O

Ti

CH3 CH3

H CH3 CH3

X

L

R

H

O

L

OH

13

p-MeOC 6H4

(86%

O 9

O 9

10

1) Pd(OH)2, iPrOH 2) PCC 3) 1M HCl, THF

8 11

7

O

12

O

58 %

13

O

4

1

O

OH 5

6 5

13

11

OH

1

3

O

3

2

O

OH

O

Michael Addition - 1,4-addition of an enolate to an α,β-unsaturated carbonyl to give 1,5-dicarbonyl compounds -

O

+

O

O

O M

Ph

R

Ph

R

Organometallic Reagents Grignard reagents: O R-Br

Mg(0)

OH R-MgBr R

THF

O O

OH R-MgBr

THF

R

often a mixture of 1,2- and 1,4-addition

+ R

90

C-C BOND FORMATION

91

O OH R-MgBr

O R-MgBr

1,2-addition

R

THF, CeCl3

O 1,4-addition

CuI,THF, -78C R

Organolithium reagents - usually gives 1,2-addition products - alkyllithium are prepared from lithium metal and the corresponding alkyl halide vinyl or aryl- lithium are prepared by metal-halogen exchange from the corresponding vinyl or aryl- haidide or trialkyl tin with n-butyl, sec-butyl or tbutyllithium. Li(0)

R-Br

R-Li

Et2 O X

Li

nBu-Li

X= Br, I, Bu3Sn

Et2 O

Organocuprates Reviews: Synthesis 1972, 63; Tetrahedron 1984, 40 , 641; Organic Reactions 1972, 19 , 1. - selective 1,4-addition to α,β-unsaturated carbonyls CuI, THF

2 R-Li

R2CuLi O

O R2CuLi

R

- curprate "wastes" one R group- use non transferable ligand _

MeO

MeO

Cu

Li+

Cu R

R-Li

non-transferable ligand Other non transferable ligands _ _ + Bu3P Cu R Li Me2S Cu R Li+

_

_ NC Cu R

Li

+

F3B Cu R

Li+

22Li

Cu R

S

Mixed Higher Order Cuprate B. Lipshutz Tetrahedron 1984, 40 , 5005 Synthesis 1987, 325.

+

CN

Addition to Acetals O R

CH3

n-C6 H13

(n-C6H13)2CuLi

H3C O

Tetrahedron Asymmtetry 1990, 1, 477. O

BF3•OEt 2 R

1) PCC 2 NaOEt OH

LA

O O H

CH3

O O

R

CH3 Nu:

H

CH3

OH Chiral axulliary is destroyed 99 % ee LA

R

n-C6 H13

O H R

O Nu

TL 1984, 25, 3087

C-C BOND FORMATION

92

TMS O

1) TiCl4

O

2) [O] 3) TsOH

JACS 1984, 106, 7588 OH

98 % ee

Stereoselective Addition to Aldehydes - Aldehydes are "prochiral", thus addition of an organometallic reagent to an aldehydes may be stereoselective. - Cram's Rule JACS 1952, 74 , 2748; JACS 1959, 84 , 5828. empirical rule O R

1

*

OH

-

M S

1) "R2 - " 2) "H + "

R1

R

M S

2

L

L

O

OH

M

S

M

R2

L R1

-

R

S

1

L

R

2

- Felkin-Ahn TL 1968, 2199; Nouv. J. Chim. 1977, 1 , 61. based on ab initio calculations of preferred geometry of aldehyde which considers the trajectory of the in coming nucleophile (Dunitz-Burgi trajectory). O L

R2 R

S

vs.

M

R2

-

L

-

S

1

O

M

better

R

1

worse

- Chelation Control Model- "Anti-Cram" selectivity - When L is a group capable of chelating a counterion such as alkoxide groups + M

OH

O R

OR' R1

*

S M

2

R

1

OR' "Anti-Cram" Selectivity

M S

M+ O OR'

HO

R2 M R

1

-

M

S

OR' R2 R1

S

Umpolung - reversal of polarity Aldrichimica Acta 1981, 14, 73; ACIIE 1979, 18, 239. i.e: acyl anion equivalents are carbonyl nucleophiles (carbonyls are usually electophillic) O R

Benzoin Condensation O-

KCN PhCHO Ph

H CN

usually -

R

O +

Comprehensive Organic Synthesis 1991, 1, 541. OH Ph - CN Cyanohydrin anion

PhCHO

HO Ph

OCN

Ph

-O Ph

CN

O

OH Ph

Ph

OH

Ph Benzoin

C-C BOND FORMATION 93 Thiamin pyrophosphate- natures acyl anion equivalent for trans ketolization reactions H

NH2

NH2

_

+ N

N N

H 3C

+

S

N

N

S

OPO3PO3

H 3C

H 3C

N

OPO3PO3

H 3C

Thiamin pyrophosphate CHO H

H 2C

OH

H

+

H

OH

H 2C

OPO3

thiamin-PP

O HO

OH

H 2C

OH

H 2C

+

OPO3

OPO3

glyceraldehyde-3-P (C3 aldose)

D-ribulose-5-P (C5 ketose)

D-ribose-5-P (C5 aldose)

H

OH

H

OH

CHO

O

OH

H

H 2C

OH

H

H

OH

H

OH

H

OH

H 2C

OPO3

sedohepulose-7-P (C7 ketose)

Trimethylsilycyanohydrins O R

TMSO

TMS-CN

CN

R

H

LDA, THF

TMSO

H

CN

acyl anion equivalent

_

R

O

NC OMs OEE

NaHMDS, THF, -60°C

CSA, tBuOH

CN O

Tetrahedron Lett. 1997, 38, 7471

(72%)

OEE

O

O

O

O

O

Dithianes B:, THF S

S

S

R

H

R

O

Hg(II)

R'-I

S -

S

S

R

R'

R

R'

Aldehyde Hydrazones B:

H N

R

N

tBu

R

H

Heteroatom Stabilized Anions Sulfones R

LDA, THF

S O

H

O R

E

E

(Dithiane anion is an example) O

_ Ph

tBu

N

E+

N

Ph R

S O

O

R'

R'

OH

R'

Al(Hg)

R'

R'

OH

R' Ph

R

O

S O

R O

Sulfoxides O

R'

_ Ph R

S O

LDA, THF

Ph R

S O

R'

R'

OH

Raney Ni

R' Ph R

R' R'

S O

R

OH

C-C BOND FORMATION Epoxide Opening

94

Asymmetric Synthesis 1984, 5, 216.

Basic (SN2) Condition

Nu: R

R

Nu

Steric Approach Control O

HO

Acid (SN1-like) Condition R

R

Nu: attachs site that best stabilizes a carbocation

OH

O+ Nu

Nu

H

OH

O

OH

BnO

OH

BnO

+

OH

BnO

TL 1983, 24, 1377

OH Me2CuLi AlMe3

O

6:1 1:5 OH

Me3Al

JACS 1981, 103, 7520

S

S

OH S

_

O

JOC 1974, 3645 S

O S Ph

+

S _

S

O

OH

Ph

(69 %) 1) TBS-Cl 2) MeI, CaCO 3, H+

S

OH

Tetrahedron Lett. 1992, 33, 931

Ph

Cyclic Sulfites and Sulfates (epoxide equivalents)

Synthesis 1992, 1035.

O OH R2

R1

SOCl2, Et3N

O

S O

R1

OH

O

RuCl3, NaIO4

O

S O

O

R1

Nu:

R2

O

R1

R2

R2

sulfate

sulfite O

O

O

S

-

O

S O

H

H2O

R2 Nu

R1

HO

H R2 Nu

R1 H2O

O

O

O S O R1

O R2

Nu:

O S O R1

OH R2 Nu

H

Nu: Nu2

R1

H R Nu1 2

C-C BOND FORMATION CO2Me

O SO2

O

2) TBS-Cl

CH3

OBn

MeO OMe

O SO2

carpenter Bee pheromone

O

CO2Me

O

OMe

OBn

OMe

H N

H3C

OMe NCH3

OMe

1) Ac2O 2) HCl



O

OH

MeO OBn

R2

1) H2, Rh 2) HF

OTBS

1) (CH3)2CuLi

CO2Me

R1

NaH

O

O SO2

CO2Me

CO2Me

R2

R1

MeO2C

MeO meso

OBn

OBn

HO

OBn OMe

MeO

OMe

MeO

NCH3

BnO

NCH3

BnO OAc

Irreversible Payne Rearrangement OH

O OH

O O SO2 OH

Bu 4NF

OTBS O

O

Payne Rearrangement of 2,3-epoxyalcohols Sigmatropic Rearrangements Nomenclature: σ bond that breaks

Asymmetric Synthesis 1984, 3, 503.

1

2 3 R

1

3 R

2



1

2 3 R

1

R

R

H

σ bond that breaks

σ bond that forms

[3,3]-rearrangement

3 4

1

3

2

3 2

Aldrichimica Acta 1983, 16, 60

2



5

4

1 R

H

1

1

[1,5]-Hydogen migration

5

σ bond that forms

3,3-sigmatropic Rearrangements Cope Rearrangemets- requires high temperatures R



R

Organic Reaction 1975, 22, 1

95

C-C BOND FORMATION

96

Chair transition state: CH3

CH3 Z

220 °C

H 3C H

H 3C E

H

E,Z (99.7 %)

Z,Z (0 %)

E,E (0.3 %)

CH3

H HH

H 3C H 3C

CH3

H

CH3 Z

E

H 3C H 3C E

E,Z (0 %)

H 3C

Z

Z,Z (10 %)

E,E (90 %)

"Chirality Transfer" H

Ph R

S CH3

E Ph

CH3

CH3

(87 %) Diastereomers

Ph Ph H 3C

H 3C

Z CH3 R (13 %) H

R E

Ph

R

CH3 R H

E Z

CH3

Ph CH3

Diastereomers Ph Ph

H 3C

H 3C R Z

Z H S CH3

- anion accelerated (oxy-) Cope- proceeds under much milder conditions (lower temperature) JACS 1980, 102 , 774; Tetrahedron 1978, 34, 1877; Organic Reactions 1993, 43, 93; Comprehensive Organic Synthesis 1991, 5, 795. Tetrahedron 1997, 53, 13971.

C-C BOND FORMATION O

OMe

OH

97

KH, DME, 110°C

OMe

KH

OH

O

O-

Ring expansion to medium sized rings OH

O

KH, ∆

9-membered ring

Claisen Rearrangements - allyl vinyl ether to an γ,δ-unsaturated carbonyl Chem. Rev. 1988, 88, 1081.; Organic Reactions 1944, 2, 1.; Comprehnsive Organic Synthesis 1991, 5, 827. ∆ O

O

O

OH

CHO O

220 °C

Hg(OAc)2

JACS 1979, 101 , 1330

O

O

O

H

H

H

O

O

O

Chair Transition State for Claisen E-olefin

R

O

O R

H

X

X

X=H X= OEt, NMe2, etc R

O

1,3-diaxial interaction

X R

E/Z = 90 : 10 E/Z = > 99 : 1

Z-olefin R

X

X

H O

O

new stereogenic centers R old stereogenic center

O O H

X

R X

C-C BOND FORMATION 98 - Chorismate Mutase catalyzed Claisen Rearrangement- 105 rate enhancement over non-enzymatic reaction CO2H

O

CO2H

HO2C

Chorismate mutase

J. Knowles JACS 1987, 109, 5008, 5013

O

CO2H

OH

OH

Chorismate

Prephenate

- Claisen rearrangement usually proceed by a chair-like T.S. HO2C

H HO2C

O

H CO2H

H H

O

Chair T.S

OH

OH

Opposite stereochemistry

H H O

CO2H CO2H

CO2H

H

Boat T.S

H

OH

O CO2H

OH OH

OH

OH +

O

J. Org. Chem. 1976, 41, 3497, 3512 J. Org. Chem. 1978, 43, 3435

O

+

O R

R

O

H

H

O

s CH3

R

O R

H

CH3

H s CH3

O H

CH3 CH3

OH

CH3

OH

O

CH3 H

CH3 CO2R

CH3

OH

CH3

OH

Tocopherol 94 - 99 % ee

hydrophobically accelerated Claisen - JOC 1989, 54, 5849

C-C BOND FORMATION Johnson ortho-ester Claisen: EtO OEt OH

OEt

O

H3C-C(OEt)3



OEt [3.3]

O

O

- EtOH

H+

Ireland ester-enolate Claisen.

Aldrichimica Acta 1993, 26, 17. OTMS

O

LDA, THF TMS-Cl

O

OH [3.3]

O

O

O LDA, THF TMS-Cl

OBn

O

Me CO2H

Me Me

Me

JOC 1983, 48, 5221

OBn

Eschenmoser NMe2

R OH

EtO

OEt



O

O NMe2

NMe2

BF3

R

R

"Chirality Transfer" R

R N

O

N

R

Ph

N

O

aldehyde oxidation state

O

Ph

Ph (86 - 96 % de)

R= Et, Bn, iPr, tBu

[2,3]-Sigmatropic Rearrangement H

Z

Comprehensive Organic Synthesis 1991, 6, 873. H

H

:X Y

X Y:

R

R

R

H X

Y:

R

R1

R H

-Wittig Rearrangement

:X

Y

Organic Reactions 1995, 46, 105 _

base

O

O

SnR3

BuLi

:X Y

R

R1

X Y:

E

HO

_ O

Synthesis 1991, 594.

99

C-C BOND FORMATION TBDPSO

TBDPSO

KH, 18-C-6, Me3SnCH2I

TBDPSO nBuLi

H MeO

H

O

MeO

H

O

MeO

OH

O

O

O SnMe3

TBDPSO

+

O

H MeO

H3C

O

OH

(58%)

(42%)

CH3 Ph _

O

CH3

H3C

CH3

CH3 H

(87 %)

OH

Ph

H

_

O

CH3 Ph

H3C

Sulfoxide Rearrangement R

J. Am. Chem. Soc. 1997, 119, 10935

Li

TBDPSO H

MeO

100

R

S -

O

S

(13 %) OH

Ph

(MeO) 3P HO

O

O

O CO2Et

CO2Et

(MeO) 3P

Ph

S

HO

O-

Ene Reaction Comprehensive Organic Synthesis 1991, 5, 1; Angew. Chem. Int. Ed. Engl. 1984, 23, 876; ; Chem. Rev. 1992, 28, 1021. H

H

- Ene reaction with aldehydes is catalyzed by Lewis Acids (Et2AlCl) R

R

H O H

O H

OH CHO

JOC 1992, 57, 2766

Et2AlCl CH2Cl2 -78°C

Ph O

O

O Ph O

O H

OH

SnCl4

Ph O SnCl4

99.8 % de

O OH + syn isomer

H3C

(94 : 6)

C-C BOND FORMATION O TiCl2 O OH

O H

CO2Me

CO2Me

PhS

+

H

OH

OH

O R

Tetrahedron Lett. 1997, 38, 6513

(97% ee)

+

+

PhS

CO2Me

CO2Me R

syn (90 % ee)

Angew. Chem. Int. Ed. Engl. 1989, 28, 38 CH3

CH3 C6H13

CH3

CO2Me R

(9 : 1)

anti (99 % ee)

- Metallo-ene Reaction

PhS

C6H13

H2O

C6H13

(10 %)

H 3C ClMg

Cl

C6H13

MgCl H 3C

C6H13

H2O H 3C

ClMg

H 3C (> 1%)

intramolecular

BrMg +

(11 : 1)

+

MgBr

BrMg

1)

CH3

1) Mg(0), Et2 ) 2) 60 °C

Li

CHO 2) SOCl2

MgCl MgCl

Cl

CH3

O

CH3 OH

Cl

SOCl2

CH3

CH3 MgCl

O2 H

1) PCC 2) MeLi 3) O3

CH3 CHO

4) KOH 5) H2 6) Ph3 P=CH2

MgCl

H3C

H

H O

1) Mg(0), Et2 ) 2) 60 °C

Capnellene

H

OH

101

C-C BOND FORMATION Synthesis of Phyllanthocin O

A. B. Smith et al. J. Am. Chem. Soc. 1987, 109, 1269. O

O (Me3Si)2 NLi

O

N

Ph

O

O

1) LAH 2) BnBr

N

Br Ph

CH3

CH3

BnO 1) O3 2) H2, Lindlar's

BnO

MeAlCl CHO

BnO

1) MEM-Cl 2) O3

H

BnO

BnO

CH3

O H

OH

CHO OMEM

O

1) 2)

-

O

O

2) H

O

MEMO

3) Swern

O

1) DBU 2) H2, Pd/C

O 3) RuO4

O

O

O O

(CH3 )2S(O)CH 2-

CH O 3

O

BnO

O

O

O

1) LDA, TMSCl 2) BnMe3 NF, MeI

O

O

HO2C

O BnO

O

O BnO

O

+

O

H3O +

O

1) ZnCl2

BnO

CH3

O

CH3 MeO2C

O

O O

Ph

Phyllanthocin

102

C=C BOND FORMATION C=C Bond Formation 1. 2. 3. 4. 5. 6. 7. 8. 9. 10 11.

103

C&S Chapt. 2 # 5,6,8,9,12

Aldol Condensation Wittig Reaction (Smith, Ch. 8.8.A) Peterson Olefination Julia-Lythgoe Olefination Carbonyl Coupling Reactions (McMurry Reaction) (Smith Ch. 13.7.F) Tebbe Reagent Shapiro and Related Reaction β−Elimination and Dehydration From Diols and Epoxides From Acetylenes From Other Alkenes-Transition Metal Catalyzed Cross-Coupling and Olefin Metathesis

Aldol Condensation -Aldol condensation initially give β-hydroxy ketones which under certain conditions readily eliminated to give α,β-unsaturated carbonyls. OMe

OMe LDA, THF, -78°C O O

OMe OR

O

O

O

Tetrahedron 1984, 40, 4741

CHO

-78C → RT

OMe OR

O

Robinson Annulation : Sequential Michael addition/aldol condensation between a ketone enolate and an alkyl vinyl ketone (i.e. MVK) to give a cyclohex-2-en-1one JOC 1984, 49 , 3685 Synthesis 1976, 777. O

O

O

O

O

L-proline

Weiland-Mischeler Ketone (chiral starting material)

Mannich Reaction - α,β-unsaturated carbonyls (α-methylene carbonyls) O

H2C=O, Me2NH•HCl Me

H 2C N + Me

O Cl-

Mannich Reaction

C=C BOND FORMATION

104

Wittig Reaction review: Chem. Rev. 1989, 89, 863. mechanism and stereochemistry: Topic in Stereochemistry 1994, 21, 1 - reaction of phosphonium ylide with aldehydes, ketones and lactols to give olefins R

X

Ph3P

R X

O R1

strong base

+ PPh3

R

-

+ PPh3

R

Ph3P O

PPh3

phosphorane

ylide

+ Ph3P O -

R2

_

- Ph3P=O

R2

R R R1 2

R

R R1 2

R

R1

oxaphosphetane

betaine

- Olefin Geometry O L

S Z- olefin

Ph3P L

S O L

S

E- Olefin

Ph3P S

L

- With "non-stabilized" ylides the Wittig Reaction gives predominantly Z-olefins. Seebach et al JACS -

O

O O L

S

L

S

S

L

L

S

O

+ PPh3

S

- Ph3P=O L L

S S

S

S

S

L

L

Z-olefins

- "Stabilized ylides" give predominantly E-olefins O + Ph3P

L

O PPh3

Ph3P L

S

S

PPh3 +

+ PPh3

L

L

_

L CO2Me

S

S L

CO2Me

C=C BOND FORMATION

105

- Betaine formation is reversible and the reaction becomes under thermodynamic control to give the most stable product. - There is NO evidence for a betaine intermediate. - Vedejs Model: R O

H R

Ph Ph P

Pukered (cis) oxaphosphetane (kinetically favored)

Cis

H Ph

H

R

Ph Ph P

R H

Planar (trans) oxaphosphetane (thermodynamically favored)

Trans

O

Ph

Phosphonate Modification (Horner-Wadsworth-Emmons) (EtO)3P

R X Arbazov Reaction

R

_ R

P(OEt)2

P(OEt)2 O

O

- R is usually restricted to EWG such as CO2H, CO2Me, CN, SO2Ph etc. and the olefin geometry is usually E. - Still Modification

TL 1983, 24, 4405. (CF3CH2O)2P

CO2Me

O

- CF3CH2O- groups make the betaine less stable, giving more Z-olefin. O CHO

Me3Si

(CF3CH2O)2P -

CO2Me Me3Si +

CO2Me

(Me3Si)2N K , THF, 18-C-6

Peterson Olefination

JACS 1988, 110, 2248

Z:E = 25:1

review: Synthesis 1984, 384

Organic Reactions 1990, 38 1. -

O

O

Me3Si

CO2Me

LDA, THF Me3Si

_

CO2Me

R1

R2

R1 Me3Si MeO2C

Silicon can stabilize an α- negative charge O R1

R2

Me3Si MeO2C

- Me3Si-O

R1 R2

H

usually a mixture of E and Z olefins CO2Me

R2

H

C=C BOND FORMATION Julia-Lythgoe Olefination

106

TL 1973, 4833 Tetrahedron 1987, 43, 1027 OH

SO2Ph LDA, THF, -78°C

OTBS

1) MsCl, Et N OTBS 2) Na(Hg),3MeOH

TL 1991, 32, 495

mixture of olefins

CHO

TBSO

R

PhO2S

R

R Tetrahedron Lett. 1993, 34, 7479

O O

H O

O 1) tBuLi, THF, HMPA, -78°C 2) Na(Hg) NaHPO4 THF, MeOH, -40°C

OBz

OSitBuPh2

HO

O

OSitBuPh2

HO

O

HO

O

O

O

OH

OH

OH

Milbemycin α1

OR OTBDPS

O

SmI2, HMPA/THF

O

R= H, Ac

O

SO2Ph

O

O

OBz

+ PhO2S

O O

OTBDPS

Synlett 1994, 859

75 % yield E:Z = 3 : 1

Ramberg-Backlund Rearrangement Br OSiPh2tBu

1) Na2S•Al2O3 2) mCPBA

OSiPh2tBu

O

OSiPh2tBu

S

SOCl2

OSiPh2tBu

O

Br Cl O

OR

S

MeLi, THF

OR

O

O

OR

S

- SO2

OR

OR

O thiiarane dioxide

OR JACS 1992, 114, 7360

Carbonyl Coupling Reactions (McMurry Reaction) Reviews: Chem. Rev. 1989, 89, 1513. - reductive coupling of carbonyls with low valent transition metals, Ti(0) or Ti(II), to give olefins O "low-valent Ti" R1

R2

R1

R1

R2

R2

+

R1

R2

R2

R1

usually a mixture of E and Z olefins

excellent method for the preparation of strained (highly substituted) olefins - Intramolecular coupling gives cyclic olefins

C=C BOND FORMATION

107

CHO "low-valent Ti"

JACS 1984, 106, 723

CHO

OMe

OMe

TiCl4, Zn, pyridine (86 %)

OMe

OHC

Tetrahedron Lett. 1993, 34, 7005 OMe OH OH

O

Tebbe Reagent Cp2Ti(CH2)ClAlMe2 - methylenation of ketones and lactones. The later gives cyclic enol ethers. H2 C Ti AlMe2 Cp Cl Cp

O

O

O

200 °C O

- Cp2TiMe2 will also do the methylenation chemistry JACS 1990, 112, 6393. Shapiro and Related Reactions Organic Reactions 1990, 39, 1 : 1976, 23, 405 - Reaction of a tosylhydrazone with a strong base to give an olefin. NNHTs Et

N

2 eq. nBuLi

_ N

Et

N N

Ts

Et

Et

- N2

_

THF

Me

_

Me

Me

E

_ E+

Me

Et

Me

Bamford-Stevens Reaction- initial conversion of a tosylhydrazone to a diazo intermediate H

H

R1 R2

R3

H

R1 R2

base

R1 R2

R3

NNHTs

N N _

R3 +N _ N

Ts

a: aprotic- decomposition of the diazo intermediate under aprotic conditions gives and olefin through a carbene intermediate. H R1 R2

R3

H

- N2

R1 R2

+N _ N

R1

R3 ••

R3

R2

Carbene

b. protic- decomposition of the diazo intermediate under protic conditions an olefin through a carbonium ion intermediate. H R1 R2

H R3

+N _ N

H+

R1 R2

R3 + N N

H

- N2

H R1 R2

R3 + H

-H

R1

+

R2

R3

C=C BOND FORMATION

108

β- Eliminations Anti Eliminations - elimination of HX from vicinal saturated carbon centers to give a olefin, usually base promoted. - base promoted E2- type elimination proceeds through an anti-periplanar transition state. B:

H R1 R3

R2 R4

R1

R2

R3

R4

X

- typical bases: NaOMe, tBuOK, DBU, DBN, DABCO, etc. N

N

N

N

N N

DBN

DBU

DABCO

- X: -Br, -I, -Cl, -OR, epoxides O

O

O

B:

OH

R

R

O

R

N AlEt2

R'

OH

R

- or TMS-OTf, DBU

"unactivated"

BCSJ 1979, 52, 1705 JACS 1979, 101, 2738

R'

Syn Elimination - often an intramolecular process H R1

R2

R1

R2

X R3

R4

R3

O X=

R4

O

Se

S Ph

Ph

Cope Elimination- elimination of R2NOH from an amine oxide OLI

O

+ Me2N=CH2 I -

O NMe2

THF, -78°C

ON Me2

mCPBA THF

O

O 1) Me2CuLi + 2) Me2N=CH2 CF3CO2-

NMe2

JACS 1977, 99 , 944 Tetrahedron 1977, 35 , 613

O - Me2NOH

C=C BOND FORMATION Selenoxide Elimination

109

Organic Reactions 1993, 44, 1.

O N SePh R'

R

R

Bu3P: O

R'

R

- or NO2

OH

mCPBA

R'

H

SeAr

- ArSeOH

R'

R

JACS 1979, 101, 3704

Se O

SeAr

SeCN

H 3C H3CHN

O H 3C N

H 3C

PhSeO2H, PhH, 60 °C

O N

H3CHN

(82%)

Ph

O H 3C

O

O

JOC 1995, 60, 7224 JCS P1 1985, 1865

N

N

Ph

O

Dehydration of Alcohols - alcohols can be dehydrated with protic acid to give olefins via an E 1 mechanism. - other reactions dehydrate alcohols under milder conditions by first converting them into a better leaving group, i.e. POCl3/ pyridine, P2O5 Martin sulfurane; Ph2S[OCPh(CF3)2]2 JACS, 1972, 94, 4997 dehydration occurs under very mild, neutral conditions, usually gives the most stable olefin OH

MSDA, CH2Cl2

BzO

JACS 1989, 111 , 278

BzO

H

H

Burgess Reagent (inner salt) JOC, 1973, 38, 26 occurs vis a syn elimination _ + MeO2CNSO2NEt3 H R1 R2

_ + MeO2CNSO2NEt3

R3 R4

MeO2C

- N SO2 H O

PhH, reflux

OH

Olefins from Vicinal Diols Corey-Winter Reaction

R1

R2

R4 R3

R1

R4

R2

R3

JACS 1963, 85, 2677; TL 1982, 1979; TL 1978, 737 S

OH R'

Im2C=S

O

O

R

R3P: R

OH

R

R'

R'

C=C BOND FORMATION

110

- vic-diols can be converted to olefins with K2WCl6 JCSCC 1972, 370; JACS 1972, 94, 6538 - This reaction worked best with more highly substituted diols and give predominantly syn elimination. - Low valent titanium; McMurry carbonyl coupling is believed to go through the vic-diol. vic-diols are smoothly converted to the corresponding olefins under these conditions. JOC 1976, 41 , 896 Olefins from Epoxides Ph2P -

O R1 H

HO H

H R2

R2

HO H

MeI

H PPh2

R1

R2

H PPh2Me +

R1

+ PPh2Me

HO R2

-

NC-Se -

O H H

R1

R2 NCO H R1

H

R1

R2

H

H

"inversion " of R groups

H

R1

R1

-Ph2MeP=O

R2

-

H SeCN

OR 2

R1 R1

-

NCOR 2

H

H

R1

R2 H

H

R2

H Se

Se -

NCSe O H

Se

R1

H

H

R2

H

"retention" of R groups

From Acetylenes - Hydrogenation with Lindlar's catalyst gives cis-olefins R

R'

Co2(CO)8

R

H2, Rh

R'

R

Co2(CO)6

R'

Bu3SnH

Tetrahedron Lett. 1998, 39, 2609 H

SnBu3

R

R'

From Other Olefins Sigmatropic Rearrangements - transposition of double bonds Birch Reduction

Tetrahedron 1989, 45, 1579 OMe

OMe

H3O+

O

H

C=C BOND FORMATION

111

Olefin Isomerization- a variety of transition metal (RhCl3•H2O) catalyst will isomerize doubles bonds to more thermodynamically favorable configurations (i.e. more substituted, trans, conjugated) RhCl3•3H2O

JOC 1987, 52 , 2875

EtOH OH

OH

Cl

Cl Ti

JACS 1992 114 , 2276

+

up to 80% ee

Olefin Inversion Tetrahedron 1980, 557 - Conversion of cis to trans olefins - Conversion of trans to cis- olefins R

R'

R

hν, PhSSPh

R' OH

OH

I2, CH2Cl2

CO2Me

HO

C5H11 OH

CO2Me C5H11

OH

OH

JACS, 1984, 107, xxxx

Transition Metal Catralyzed Cross-Coupling Reactions Coupling of Vinyl Phosphonates and Triflates to Organometallic Reagents - vinyl phosphates review: Synthesis 1992, 333. O

OP(OEt)2 Li, NH3, tBuOH

R2CuLi

R

R O

O

O

R2CuLi

(EtO)2PCl

- preparation of enol triflates

Synthesis 1997, 735

O

OTf

LDA, THF, -78°C

kinetic product

Tf2NPh

O

R

OP(OEt)2

O

Tf2O, CH2Cl2

tBu

N

tBu

OTf thermodynamic product

C=C BOND FORMATION - reaction with cuprates.

112

Acc. Chem. Res. 1988, 25, 47

OTf

Ph

TL 1980, 21 , 4313

Ph2 CuLi tBu

tBu

- palladium (0) catlyzed cross-coupling of vinyl or aryl halides or triflates with organostannanes (Stille Reaction) Angew. Chem. Int. Ed. Engl. 1986, 25, 508.; Organic Reactions 1997, 50, 1-652 SnMe3

O

O

1)LDA, Tf2NPh 2) Pd(PPh3)4

O

JOC 1986, 51 , 3405

O

O

OH

OTBS

Bu3Sn I

I Bu Sn 3 O

O

O

OH

J. Am. Chem. Soc. 1998, 120, 3935

OH

HO TBSO Bu3Sn

HO TBSO

(-)-Macrolactin A

I

BOMe TESO CHO 1)

2

MgBr

TESO

O3, NaBH4

TESO

3) TBSCl, imidazole (76%)

CHO

1) Dess-Martin Periodinane 2) Ph3P+CH2I, INaHMDS

(38%)

TBSO

I

TESO

CO2H

SnBu3

(42%)

OH TBSO

3) DIBAL (50%)

Bu3SnCHBr2, CrCl2

PdCl2(MeCN)2 (65%)

TESO

Bu3SnH, AIBN

OH I

OH

TBSO

TESO

OTES

I

TBSO

PdCl2(MeCN)2

TESO

OH b) I2 (83%

OPiv Bu3Sn

1) HF, MeCN 2) Me4NBH(OAc)3

OH

a) nBu3SnH, (Ph3P)2PdCl2

OTES

(77%) OPiv

TBSO

3) Dess-Martin Periodinane (70%)

2) TESOTf, 2,6-lutidine (52%) TESO

1) O3, Me 2S 2) allyl bromide, Zn

I

Bu3Sn CO2H

TESO

CO2H

C=C BOND FORMATION

113

I TESO OH

+

Bu3Sn

Ph3P, DEAD

TESO

CO2H

(50%)

TESO TESO Bu3Sn

OH

I O

1) Pd2(dba)3 iPr2NEt 2) TBAF

O

O HO

(35%)

TESO

O

HO TESO

(-)-Macrolactin A

palladium (0) catalyzed carbonylations- coupling of a vinyl triflate with a organostanane to give α,b-unsaturated ketones. O OTf

Ph

Me3SnPh Pd(0), CO

But

JACS 1994, 106 , 7500

tBu

Nickel (II) Catalyzed Cross-Coupling with Grignard Reagents (Kumada Reaction): Pure Appl. Chem. 1980, 52, 669 Bull Chem. Soc. Jpn. 1976, 49, 1958 R-MgBr (dppp)NiCl2

X

R

R= alkyl, vinyl or aryl

Aryl or vinyl halide or triflate

Br

AcO

OMe

Br Ni

1) N H

OAc

HO Ni

R1

N H

OH

(±)-Neocarazostatin B

Tetrahedron 1998, 54, 4413, Acc. Chem. Res. 1995, 25, 446. +

olefin metathesis R2

catalyst

catalyst (CH 2)n

R1

R2

Ring-Opening Metathesis Polymerization (ROMP) (CH 2)n

catalyst (CH 2)n

Tetrahedron Lett. 1998, 39, 2537, 2947,

(2 equiv), 65 °C 2) LiAlH4 , Et2 O (72%)

Olefin Metathesis

OMe

Br

(CH 2)n

n

Ring-Closing Metathesis (RCM)

C=C BOND FORMATION Metathesis Catalysts: iPr H3C F3C F3C

iPr

(C6H11)3P Cl

N

Ru

O Mo O

F3C F3C

Ph

Ph

Ph

Cl (C6H11)3P

(C6H11)3P Cl Ru Cl (C6H11)3P

CH3

Schrock' s Catalyst

Grubbs' Catalyst

Mechanism: +

R1

R2 [M]

R1

[M]

R1 [M] R1 R2

R2

+

[M]

Ph

114

C≡C BOND FORMATION

115

C≡C Bond Formation 1. 2. 3. 4. 5. 6.

From other acetylenes From carbonyls From olefins From Strained Rings Eschenmosher Fragmentation Allenes

From Other Acetylenes - The proton of terminal acetylenes is acidic (pKa= 25), thus they can be deprotonated to give acetylide anions which can undergo substitution reactions with alkyl halides, carbonyls, epoxides, etc. to give other acetylenes. R R1-X R

_

R

H

R1

O R2

OH

R3

R

R R2 3

M+ O Et2AlCl

OH R

- Since the acetylenic proton is acidic, it often needs to be protected as a trialkylsilyl derivative. It is conveniently deprotected with fluoride ion. R

H

F-

B:, R3SiCl

R

SiR3

R

H

Acetylide anions and organoboranes R1

_

Li+

R3B

_ R1

BR3

Li+

I2

R1

R

JOC 1974, 39 , 731 JACS, 1973, 95 , 3080

Palladium Coupling Reactions: O

O O

Cl iPr3Si

SnBu3

O JACS 1990, 112, 1607

Cl

(Ph3P)4Pd iPr3Si

SiiPr3

C≡C BOND FORMATION

116

OTBS OTBS

CO2Me Br

CO2Me

C5H11

OTBS

OTBS

OTBS

Pd(Ph3P)4, CuI iPr2NH, PhH

TBSO

JACS 1985, 107, 7515

C5H11

OH HO CO2H OH

Copper Coupling- 1,3-diynes +

R1

Cu(OAc)2

R2

R1

Adv. Org. Chem. 1963,4 , 63

R1

Nicholas Reaction - acetylenes as their Co2(CO)8 complex can stabilize an α-positive charge, which can subsequently be trapped with nucleophiles. OR4 R1

(CO)6Co2

Co2(CO)8

OR4

R1

R2

(CO)6Co2 R1

R2

(CO)6Co2

Nu:

R1

oxidative decomplexation

Nu

Nu R1

R2 R3

R2

R3

N

CO2Me

R R3 2

R3

R3

N

Tf2O, CH2Cl2, MeNO2, -10°C

+

CO2Me

N

I2

CO2Me JCSCC 1991, 544

O

TBSO tBu

HO

N

tBu

O

(OC)6Co2

Co2(CO)6

Co2(CO)6-acetylene deocomplexation: JOC 1997, 62, 9380 From Aldehydes and Ketones R'-X RCHO

P3P, CBr4

Br

R Br

2 eq. nBuLi R

_

Li+

ClCO2Et

R

R'

R

CO2Et

R

H

H2O

C≡C BOND FORMATION Br2C OHC

OSitBuPh2

OHC

OSitBuPh2

OSitBuPh2

CBr4, Ph3P

OSitBuPh2

Br2C

a) nBuLi, THF, -78°C b) ClCO2Me

MeO2C

117

OSitBuPh2 OSitBuPh2

MeO2C

JACS 1992, 114 , 7360

- by conversion of ketones to gem-dihalides followed by elimination O

PCl5

R'

R

Cl

Cl

R

Cl

tBuOK

R'

- HCl

R'

R

tBuOK

R

- HCl

R'

- by conversion of ketones to enol phosphates followed by elimination LDA, (EtO)2P(O)Cl

O

O

R'

R

P(O)(OEt)2 LiNH , NH 2 3

R

JOC 1987, 52 , 4885

R'

R'

R

- Insertion reaction of a vinyl carbene (terminal acetylenes) N2

R

C

N2CHP(O)(OMe)2 R'

tBuOK

R

+

-N2 R'

••

O

C

R

R

R'

JOC 1982, 47 , 1837

R'

O Me3Si

_

N2 Synlett 1994, 107

THF, -78°C → RT

Via Elimination Reactions of Vinyl Halides - Treatment of vinyl halides with strong base gives acetylenes. X Base

R'

R

R

R'

- HX

H X= Cl, Br, I, OTf, O3SR, OP(O)R2

Base= LDA; tBuOK; NaNH2, DBU

- Addition of Grignard reagents to 1,1-difluoroethylene yields an acetylide anion which can be subsequently trapped with electrophiles. H2C=CF2

R-MgX

E+

_

R

R

E

TL 1982, 23 , 4325 JOC 1976, 41 , 1487

Strained Rings Topics in Current Chemistry 1983, 109, 189. - Cyclopropenones and cyclobutendiones can be photolyzed or thermolyzed (FVP) to give acetylenes. O

O

O hν (209 nm) 8 °K

C

- CO

O

- CO

JACS 1973, 95 , 6134

C O

Benzyne

C≡C BOND FORMATION 1) 370 °C 2) 12°K

O

118

ACIEE 1988,27 , 398 JACS 1991, 113 , 6943

- CO

R1 R2

FVP

R1

(retro- D-A)

+

R2

CL 1982, 1241

Eschenmoser Fragmentation Ph N

O

Ph

N R

R O

R'

HCA 1972, 55 , 1276

O R'

R'

HN NH2 iPr iPr

O

O

R

Base -orheat

O

tBuOOH, triton B, C6H6

O

JOC 1992, 57, 7052

iPr AcOH, CH2Cl2

Allenes Tetrahedron 1984, 40, 2805 - from dihalocyclopropanes Br

R

R'

Br

R'

R

_

••

R

Br

R'

H

R'

R

H

- From SN2' Reactions Nu:

X

H

Nu

R R'

R

R'

- from sigmatropic rearrangements from propargyl sulfoxides and phosphine oxides. Ph OH

O

R R'

Ph

:P

Ph2PCl, Et3N R

O Ph2P R

R'

R' H

TL 1990, 31 , 2907 JACS 1990, 112 , 7825

FUNCTIONAL GROUP INTERCONVERSIONS

119

Functional Group Interconversions C&S Chapter 3 #1; 2; 4a,b, e; 5a, b, d; 6a,b,c,d; 8 1 2 3 4 5 6 7

sulfonates halides nitriles azides amines esters and lactones amides and lactams

Sulfonate Esters - reaction of an alcohols (1° or 2°) with a sulfonyl chloride R OH

O

R'SO2Cl

R'=

R O S

CH3

mesylate

CF3

triflate

R'

O sulfonate ester

CH3

- sulfonate esters are very good leaving groups. competing side reaction

tosylate

Elimination is often a

Halides - halides are good leaving groups with the order of reactivity in SN2 reactions being I>Br>Cl. Halides are less reactive than sulfonate esters, however elimination as a competing side reaction is also reduced. - sulfonate esters can be converted to halides with the sodium halide in acetone at reflux. Chlorides are also converted to either bromides or iodides in the same fashion (Finkelstein Reaction). O R O S

R'

X-

X= Cl, Br, I

R X

O

R Cl

NaI, acetone reflux

- conversion of hydroxyl groups to halides: R OH

- R-OH to R-Cl - SOCl2 - Ph3P, CCl4 - Ph3P, Cl2 - Ph3P, Cl3CCOCCl3

R

I

Organic Reactions 1983, 29, 1 R X

FUNCTIONAL GROUP INTERCONVERSIONS

- R-OH to R-Br - PBr3, pyridine - Ph3P, CBr4 - Ph3P, Br2 - R-OH to R-I - Ph3P, DEAD, MeI Nitriles - displacement of halides or sulfonates with cyanide anion KCN, 18-C-6 DMSO

R X

R C

N

- dehydration of amides O R

-

R C N

NH2

POCl3, pyridine TsCl, pyridine P2O5 SOCl2

- Reaction of esters and lactones with dimethylaluminium amide TL 1979, 4907 Me

H 3C

Me2AlNH2 O O

OH

JOC 1987, 52, 1309

NC Ar

Ar

- Dehydration of oximes R CHO

N

H2NOH•HCl R

OH P2O5

R C

H

N

- Oxidation of hydrazones O O N NMe2

(97%)

- Reduced to aldehydes with DIBAL. DIBAL

RC≡N

C N

RCHO

Tetrahedron Lett. 1998, 39, 2009

120

FUNCTIONAL GROUP INTERCONVERSIONS

121

Azides - displacement of halides and sulfonates with azide anion LDA, THF NBS

O

O

O

NaN3

O

O

O

SO2N(C6H11)2

SO2N(C6H11)2

Br

SO2N(C6H11)2 N3

O

O

O

TL 1986, 27, 831

HO

SO2N(C6H11)2

NH2

NH2

- activation of the alcohol R OH

+

+ N

+ N

F

Me

N

Me

+

EtO2C N N CO2Et

O

Me

TsO -

R OH

+

R N3 OR

Ph3P, NaN3

+ PPh3

EtO2C

N N _ CO2Et

DEAD

activated alcohol + R O PPh3

R-OH

R N3

+

Ph3P=O

JOC 1993, 58, 5886 HO (PhO)2P(O)-N3 O

N3

O O

DBU, PhCH3

SN2

P(OPh)2 +

+ DBU-H

-

+ N3

(91 %)

O (97.5 % ee)

Ar

(99.6 % ee)

- Photolyzed to aldehydes Amines - Gabriel Synthesis O N - K+

O R

O

- reduction of nitro groups R NO2 H2, Pd/C Al(Hg), H2O NaBH4 LiAlH 4 Zn, Sn or Fe and HCl H2NNH2 sodium dithionite

X

N R

H2NNH2

O

R NH2

R NH2

FUNCTIONAL GROUP INTERCONVERSIONS - reduction of nitriles R C N

R CH2 NH2

H2, PtO 2/C B2H6 NaBH4 LiAlH 4 AlH3 Li, NH3 - reduction of azides R N3 H2, Pd/C B2H6 NaBH4 LiAlH 4 Zn, HCl (RO)3P Ph3P thiols

R NH2

- reduction of oximes (from aldehydes and ketones) N R

OH NH2 R'

R

R'

R'

R

N

H2, Pd/C Raney nickel NaBH4, TiCl4 LiAlH 4 Na(Hg), AcOH - reduction of amides O R

N

R'

R''

R''

H2, Pd/C B2H6 NaBH4, TiCl4 LiBH4 LiAlH 4 AlH3 - Curtius rearrangement O

O NaN3 R

R N O isocyanate

H2O

O

∆ - N2

R

•• N

••

Cl

••

R

•• •• N N N +

nitrene R NH2

122

FUNCTIONAL GROUP INTERCONVERSIONS - reductive aminations of aldehydes and ketones - Borsch Reaction - Eschweiler-Clark Reaction - alkylation of sulfonamides Tf

Tf

N

HN

N

HN

Tf

Tf

K2CO3, DMF 110°C

Tf Br

Br

Tf

N

N

N

N

Tf

NH HN

Na, NH3

TL 1992, 33 , 5505

NH HN

Tf

cyclam

- transaminiation O

Ph

N

PhCH2NH2

Ph

N

NH2

Can. J. Chem. 1970, 48, 570

H3O +

tBuOK

H+

Esters and Lactones - Reaction of alcohols with "activated acids" - Baeyer-Villigar Reaction Organic Reactions 1993, 43, 251 - Pd(0) catalyzed carboylation of enol triflates OTf

CO2R

CO, DMF Pd(0), ROH

TL 1985, 26 , 1109

- Arndt-Eistert Reaction O R

Angew. Chem. Int. Ed. Engl. 1975, 15, 32. O

CH2 N2 Cl

Et2 O

O



N2

R

Wolff Rearrangement

C O H

O

R'OH

R

OR'

ketene

O

O

TsN3, Et3N CO2Me

R

•• CH

R

ROH

R diazo ketone

N2

R

- Diazoalkanes: carbene precursors R-CHO

1) NH2NH2 2) Pb(OAc)4, DMF

R-N2

R3N H 2N R

- Halo Lactonizations

JOC 1995, 60, 4725

TsN3

N

N2

R

R

review: Tetrahedron 1990, 46 , 3321 +

I I

I2-KI CO2H

R

H2O, NaHCO3

O

O H O

O

123

FUNCTIONAL GROUP INTERCONVERSIONS Pd(OAc)2 (5 mol %) CO2H

JOC 1993, 58, 5298

O

DMSO, air (86%)

O

- Selenolactonization PhSe O

H 2O 2

O

PhSeCl, CH2Cl2

O

O

JACS 1985, 107 , 1148

O

OH

- Mitsunobu Reaction

Synthesis 1981 , 1; Organic Reactions, 1991, 42, 335 Mechanism: JACS 1988, 110 , 6487 O DEAD, Ph3P

OH R

O

R''CO2H

R'

R

R''

Inversion of alcohol stereochemistry

R'

Amides and Lactams - reaction of an "activated acid" with amines - Beckman Rearrangement Organic Reactions 1988, 35, 1 O R

N R'

R

OH

PCl5

O

R'

R

NR'

- Schmidt rearrangement O R

O

HN3 R'

H+

R

NR'

- others O OTf

NR2

CO, DMF Pd(0), R2NH

TL 1985, 26 , 1109

OH O

O O

PhCH2NH2

N H

AlMe3 OTHP

Ph

TL 1977, 4171

OTHP

-Weinreb amide Tetrahedron Lett. 1981, 22, 3815 O

DIBAL O R

O

H3CNH(OCH3) •HCl OR'

AlMe3

R

R N

H

OCH3 O

CH3 R1-M

R

R1

124

THREE-MEMBERED RING FORMATION 3 Membered Rings 1.

2.

3.

epoxides a. peracids, hydroperoxides and dioxiranes b. transition metal catalyzed epoxidations c. halohydrins d. Darzen's condensation e. sulfur ylides cyclopropanes a. Simmons-Smith reactions b. diazo compounds c. sulfur ylides d. S N2 displacements aziridines a. nitrenes b. SN2 displacements

Epoxides - peracid, hydroperoxide and dioxirane oxidation of alkenes - transition metal catalyzed epoxidation of alkenes - Sharpless epoxidation - Metal oxo reagents (Jacobsen's reagent) - from halohydrins Br

NBS, H2O

base O

OH bromohydrin

- Darzen's Condensation O _ BrCHCO2Et

B: BrCH2CO2Et

R

R R'

OO

CO2Et

R'

R'

Br

- sulfur ylides Chem. Rev. 1997,97, 2421. O

O Me S + CH2Me Corey

+ Me2SCH2-

Ph S

CH2-

+ _ SPh2

NMe2 Johnson sulfoximine

CO2Et

R

Trost

125

THREE-MEMBERED RING FORMATION

126

- dimethylsulfoxonium methylide and dimethylsulfonium methylide (Corey's reagent) review: Tetrahedron 1987, 43 , 2609. O

O R

-

O

DMSO, NaH R'

O

SMe2

R

R

R'

R'

- cyclopropyldiphenylsulfonium ylide (Trost's reagent) ACR 1974, 7, 85. +

O R

_

O

SPh2

-

O

R'

O

SPh2

R

BF3

R

R'

- sulfoximine ylides (Johnson's reagent) O Ph S CH2NMe2

O R

ZnCu +

R

ACR 1973, 6 , 341

O R

R'

Cyclopropanes - Simmons-Smith Reaction

R'

R'

R'

Org. Reactions 1973, 20, 1. ICH2ZnI

CH2I2

- polar groups (-OH, -NR2- CO2R) can direct the cyclopropanation OH

OH JACS 1979, 101 , 2139

ZnCu, CH2I2

- sulfur ylides O

O

O Me2S CH2-

Ph

Ph

O _ Ph S NMe2

Tetrahedron 1987, 43 , 2609

Ph

Ph

O O

ACR 1973, 6 , 341

THREE-MEMBERED RING FORMATION

127

- diazo alkanes and diazo carbonyls Synthesis 1972, 351; 1985, 569 - cyclopropanation with diazoalkanes; olefin requires at least on electron withdrawing group. N N

CO2Me MeO2C

CH2N2

(MeO)2HC

CO2Me

CO2Me

JACS 1975, 97 , 6075

MeO2C



MeO2C (MeO)2HC

(MeO)2HC

- diazoketones; photochemical or metal catalyzed decomposition of diazoketones to carbenes followed by cyclopropanation of olefins. Org. Rxns. 1979, 26, 361; Tetrahedron 1981, 37, 2407 CuSO4 , ∆

N2

JACS 1970, 92 , 3429 JACS 1969, 91 , 4318

O

O

Rh2(OAc)2

O Ph

O

O

O

Ph Ph

N2

O

JACS 1993, 115, 9468

O

O O

Ph

91 % yield 89 % de

Asymmetric cyclopropanation: Doyle, Chem Rev. 1998, 98, 911 Aldrichimica Acta 1997, 30, 107 O

O CO2Et

CO2Et

TsN3 , Et3N N2 C5H11

C5H11 O

O CO2Et

- SPh

JACS 1977, 99, 1940

CO2Et C5H11

C5H11 SPh

- SN2 Reactions DBU

O O

O Br

O O

O

CO2Et

JACS 1978, 99 , 1940

CO2Et

- Electrophillic Cyclopropanes review: ACR 1979, 12 , 66 in many ways, cyclopropanes react silmilarly to double bonds - homo-1,4-addition CO2R Nu:

CO2R

Nu

CO2R CO2R

ACR 1979,12 , 66

THREE-MEMBERED RING FORMATION Nu:= malonate anion, amines, thiolate anion, enamines, cuprates (usually requires double activation of cyclopropane) Me

H MeO2C

MeO2C

Me2CuLi O

TL 1976, 3875

O

- hydrogenation CO2Me

CH2I2, Zn-Cu

O

ArCO3H, Na2HPO4

CO2Me

CO2Me

H

H

H

O CO2Me

H2, PtO2, AcOH

TL 1982, 23 , 1871

H

- hydrolysis OH

OH

OH CH2I2, Zn-Cu

H , MeOH

MeO

OMe

JACS 1967 89 , 2507

+

O

Aziridines R2 R1

Ts

Cu(acac)2 R3

PhN=ITs

N R2

R1

J. Org. Chem. .1991, 56, 6744 R3

128

FOUR-MEMBERED RING FORMATION 4 Membered Rings 1. 2.

cyclobutanes & cyclobutenes oxatanes

Cyclobutanes - [2+2] cycloadditions - photochemical cycloadditions (2πs +2πs) Acc. Chem. Res. 1968, 1, 50; Synthesis 1970, 287; Acc. Chem. Res. 1971, 4, 41; Organic Photochemistry 1981, 5, 123; Angew. Chem. Int. Ed. Engl. 1982, 21, 820; Acc. Chem. Res. 1982, 15, 135; Organic Photochemistry 1989, 10, 1 Organic Reactions 1993, 44, 297 - for synthetic purposes, cyclic α,β-unsaturated carbonyl are the most useful. intersystem crossing

1

E*

E

3

E*

- symmetry requirements 2πs + 2πs

*



O

O

~ 340 nm

HOMO LUMO

- enones with olefins O

O +

CH2

O

O

O

hν +

CH2

JCSCC 1966, 423

+

(90%)

O

O

O

O



H E.J. Corey JACS 1963, 85 , 362

+

+

H Caryophellene Base

Hot Ground State? O

O

O H2C=CH2



2πs + 2πa thermal cyclization

CH3 N H

H2C=CH2 hν, CH2Cl2 CH3

O

N CH3

isomerization

cis ring-fusion

H

CH3

O Ph

H

H

CH3 O

JACS 1986, 108 , 306

HO CH3

grandisol

129

FOUR-MEMBERED RING FORMATION O

O

O

130

H

H +

JACS 1984 106 , 4038

hν H

H 2:1

- enones with acetylenes O O



O

O

O

JACS 1982 104, 5070

- DeMayo Reaction O

O

O

hν, pyrex

O

O

CO2Me

H

pTSA, MeOH reflux

JACS 1986, 108 , 6425

O O H

O

R



O

R

O

R O

O

MeO2C

H

O

O

70-80 % de

favored

O

O O

controls conformation O

O



H 3C

O

O

CH3

O

H

CH3

O disfavored

TL 1993, 34, 1425

H 3C H 3C

H

- Photochemistry of Ketones (Norrish Type I and II reactions) H

H hν

Norrish I cleavage

O •



254, 307 nm Norrish II cleavage HO

Yang reaction

OH •



H

H H

O

O• •

H

H 3C

O

O

H

O

CH3 CH3 controls addition

O

H

O

O

H 3C

H

OH +

FOUR-MEMBERED RING FORMATION - filtering photchemical reaction to prevent Norrish reactions quartz 180 nm Vycor 200 nm Pyrex 280 nm Uranium glass 320 nm - Yang Reaction MOMO

MOMO O hν (254 nm)

OH

C6H6 SEMO

CH3

JACS 1987, 109 , 3017

CH3

SEMO

- thermal cycloadditons (2πa + 2πs) - symmetry requirements 2πa + 2πs

O

HOMO

LUMO

- ketenes O



H O

O

H

O Cl

Et3N O

H O Cl

Zn

O

Cl O N2

R

R



O H

- thermal cyclization of ketene with olefins Tetrahedron 1986, 42, 2587; 1981, 37, 2949; Organic Reactions 1994, 45, 159. LUMO of ketene 2πa

pz

py

O HOMO of olefin 2πs

131

FOUR-MEMBERED RING FORMATION Cl

O CH3

Ph

O

Cl

Cl

CH2N2

CH3 O

O

O

Cl CCl3

CH3

Zn-Cu, Et2O

Cl

Ph

H 3C H 3C

N+ 2) H2O

H 3C

Ar

JACS 1987, 109 , 4753

H

O

1)

CH3

R*O

CH3

H 3C

132

N+

JACS 1982, 104, 2920 H

OMe

OMe

-reaction of ketene with enamines H NR2

O

O H

NR2

O

O

NR2

R'

R'

- reaction of ketene with imines to give β-lactams (Staudinger Reaction) R

H N

O H NR O

O O

R

N

+

H

CH2Cl2 -78°C → 0°C

N

Ph O

Bn

N

O

N

Ph

N

Ph

H

O

O

R

O H

O

Bn

R NBn

O (90-96 % de)

- reaction of difluorodihaloethylene with olefins Organic Reactions 1962, 12, 1 F

F2C=CX2 R

F X= F, Cl

X X

R

- reaction of difluorodihaloethylene with acetylenes- biradical mechanism F

F2C=CX2 R

F X= F, Cl

R

X

hydrolysis

O

X R

O

FOUR-MEMBERED RING FORMATION

133

- SN2 Reaction O

O KH, DMSO

JACS 1980, 102 , 1404

OTs

O

_

CN

CN HO

OR

JACS 1974, 96, 5268, 5272

OH

OR

Grandisol

- acyloin reaction

Organic Reactions 1976, 23, 259

CO2Me

OTMS

Na, TMS-Cl

(CH2)n

O

F-

(CH2)n CO2Me

(CH2)n

OTMS

OH

R

CO2Me

R

O

R

CO2Me

R

OH

- benzocyclobutanes

ACIEE 1984, 23, 539; Synthesis 1978, 793 O

O

O

O

O benzocyclobutane

O

o-quinodimethane

- cyclotrimerization of 1,5-diyenes with an acetylenes SiMe3

Me3Si

SiMe3

CpCo(CO)2

Me3Si

- sulfur ylides +

O R

_

SPh2

R'

O BF3

O R

R' R'

R

Oxatanes Organic Photochemistry 1981, 5, 1 - [2+2] cycloaddition (Paterno-Buchi Reaction) O R

R

R' O

R

O



R' hν (254 nm)

R'

O R R'

FOUR-MEMBERED RING FORMATION

O

O



O

Me2C CMe2

O O

134

TL 1975 1001

O

- SN2 reaction O

NBS

OH

JCSCC 1979, 421

SiMe3

SiMe3 Br OH CO2Me

Tf2O, CH2Cl2

CO2Me

O

C5H11

HO

TL 1980,21 , 445

C5H11

OTBS

OH

OH Br

Br CO2Me

HO

(MeO)3P, DEAD

O

CO2Me

O

C5H11

C5H11

O

OH

JACS 1984, 107 , 6372

OH

- sulfur ylides O

O

-

O

O

NTs

H2C_ S Me

S

NTs

O

JACS 1979, 101 , 6135

CH3

β-Lactones R2

R1

LDA, THF SPh

H

BnO

N H

O

R3

CO2H

R1

R2 R1

O BnO

JOC 1991, 56 , 1176

SPh

R4

DEAD, Ph3P

O O

R4

O R3

OH

O

O-

O

R2CuLi

O N H

R2

R3

R4

CO2H

R BnO

NH

JACS 1987, 109 , 4649

O O

1) MgBr2•OEt2 2) KF, H2O, CH3CN

O OBn O

OBn

O

O

+ H

SiMe3

H

96 % de

TL 1995, 36, 4159

BALDWIN'S RULES FOR RING CLOSURE Baldwin's Rules (Suggestions) for Ring Closure JOC 1977, 42 , 3846 JCSCC 1976, 734, 736, 738

135

Approach Vector Analysis - for an SN2 displacement at a tetrahedral center, the approach vector of the entering nucleophile is 180° from the departing leaving group Nu:

Nu

L

Nu

L

:L

- for the addition of a nucleophile to an Sp2 center, the nucleophile approaches perpendicular to the π-system. Nu Nu

Nomenclature 1. indicate ring size being formed 3 membered ring = 3 4 membered ring = 4 etc. 2. indicate geometry of electrophilic atom if Y= Sp3 center; then Tet (tetrahedral) if Y= Sp2 center; then Trig (trigonal) if Y= Sp center; then Dig (digonal) Z Y

X:

3. indicate where displaced electrons end up - if the displaced electron pair ends up out side the ring being formed; then Exo - if the displaced electron pair ends up within the ring being formed; then Endo Z Y

X: Exo

Z

Y

X: Endo

BALDWIN'S RULES FOR RING CLOSURE 136 4. Ring forming reaction is designated as Favored or Disfavored disfavored does not imply the reaction can't or won't occur- it only means the reaction is more difficult than favored reactions. Rules (Suggestions) for Ring Closure - All Exo-Tet reactions are favored

X:

Y

Z

X: Y

X:

Z

X:

Y Z

3-

4-

X:

Y

Y Z

5-

Z

6-

7-

- - - - - - - - - - - - - - - - - - - - - - - - --Favored- - - - - - - - - - - - - - - - - - - - - - - - -

- 5-Endo-Tet and 6-Endo-Tet are disfavored Z

Z

Y

Y X:

X: 5-

6-

- - - - -Disfavored- - - - -

- All Exo-Trig reactions are favored

X:

Y

Z

X: Y

Z

X:

X:

Y Z

3-

4-

5-

X:

Y

Y

Z

Z

6-

7-

- - - - - - - - - - - - - - - - - - - - - - - - --Favored- - - - - - - - - - - - - - - - - - - - - - - - -

- 3-Endo-Trig, 4-Endo-Trig and 5-Endo-Trig are disfavored; 6-Endo-Trig, 7Endo-Trig, etc. are favored

Z

Z X:

Y 3-

Z

X: Y 4-

Z

Y

X: 5-

- - - - - - - - - - - - -Disfavored- - - - - - - - - - - -

X:

Y

X:

Z Y

76- - - - - -Favored- - - - - -

BALDWIN'S RULES FOR RING CLOSURE 137 - 3-Exo-Dig and 4-Exo-Dig are disfavored; 5-Exo-Dig, 6-Exo-Dig, 7-Exo-Dig, etc. are favored

X:

X:

Y

X:

Y

Z

Z

3-

4-

Y

X:

Z

Z

5-

- - - - - -Disfavored- - - - - -

X:

Y

Y Z

6-

7-

- - - - - - - - - - - - -Favored- - - - - - - - - - - -

- All Endo-Dig are favored Z Z X:

Z Y

Y X:

X:

3-

4-

Z

Y

5-

X: 6-

Y

X:

Z Y 7-

- - - - - - - - - - - - - - - - - - - - - - - - - - --Favored- - - - - - - - - - - - - - - - - - - - - - - - - - - - -

EXCEPTION: There are many !!!

(see March p 212-214)

FIVE-MEMBERED RING FORMATION 5 Membered Rings HO

O CO2H

HO

CO2H

HO

OH

OH

PGF2α

PGE2 Me H

Me Me Me Me

Me Me Me Hirsutene

1. 2. 3. 4.

5. 6. 7. 8. 9.

Isocumene

Modhephane

Intramolecular SN2 Reactions Intramolecular Aldol Condensation and Michael Addition Intramolecular Wittig Olefination Ring Expansion and Contraction Reactions a. 3 → 5 b. 4 → 5 c. 6 → 5 1,3-Dipolar additions Nazarov Cyclization Arene-Olefin Photocyclization Radical Cyclizations Others Synthesis 1973, 397; ACIEE 1982, 21 , 480;

Intramolecular SN2 Reaction

5-exo-tet: favored O LDA

O

JCSCC 1973, 233

Br

O

O CO2Me

O CO2Me

Br

CN

CN

OH JACS 1974, 96 , 5268

O

138

FIVE-MEMBERED RING FORMATION O

Cl JACS 1979, 101 , 5081

O RO2C

RO2C

Cl

CO2R

CO2R

O

O NaH, DMSO

JCSCC 1979, 817

TsO

Intramolecular Aldol Condensation 5-exo-trig: favored intramolecular aldol condensation of 1,4-diketones O

O R

R

O R' R' O

O CH3

CH3

O

TL 1982, 29, 2237

CH3 CH3

Br

Br

O

NaOMe, MeOH O

O

CO2Me

JACS 1980, 102 , 4262

CO2Me O

O JOC 1983, 48 , 1217

NaOH O

O

O

Intramolecular Michael Addition 5-exo-tet: favored Organic Reactions 1995, 47, 315-552 O O MeO2C

O

O

JACS 1979, 101 , 7107

139

FIVE-MEMBERED RING FORMATION Tetrahedron 1980, 36, 1717

Intramolecular Wittig Olefination O

O

1) (MeO) 2P(O)CH2 -

O

P(OMe)2

O

2) Collins

O

C5H11

THPO

OTHP

C5H11

THPO

OTHP

CO2H

O

JOC 1981, 46 , 1954

base C5H11

THPO

OTHP

C5H11

HO

OH

Ring Expansion Reactions - 3 → 5: Vinyl Cyclopropane Rearrangement _

O

SPh2

O

Organic Reactions 1985, 33, 247.

O

H

OTMS

O

O

O

H O

H O

TMSO

JACS 1979, 101 , 1328 O

H O

- 4 → 5: Reaction of cyclobutanones with Diazomethane Cl

Me

CH2N2

Me

Me

Cl O

O O Me

Cl

Me

Me

CH2N2

TL 1980, 21 , 3059

O

Cl Cl

Cl

Cl

O

O

Cl

Cl CH3 O

CH3

CH2N2

Cl CH3 O

CH3 JACS 1987,109 , 4752

Ph Ph

140

FIVE-MEMBERED RING FORMATION Ring Contraction Reactions - 6 → 5: Favorskii Reaction

Organic Reactions 1960, 11 , 261 _

O

O Cl

OH

-

O OH CO2H

_

1,3-Dipolar Addition to Olefins 1,3-Dipolar Cycloaddition Chemistry , vol 1 & 2 (A. Padwa ed.) (Wiley, NY 1984); ACIEE 1977, 16 , 10. Chem Rev. 1998, 98, 863. 1,3-Dipole (4π) LUMO

+ _ b c a

4πs + 2πs

- trimethylenemethane (TMM)

Alkene (2π) HOMO

ACIEE 1986, 25 , 1. Synlett 1992, 107.

high temperature

• • TMM

CO2Me

MeO2C

∆, MeCN N

MeO2C H



• •

-N2

N

CO2Me CH3

H



JACS 1981, 103 , 2744

note: TMM usually reacts poorly w/ electron defficient olefins _ Pd(0) Me3Si

OAc

+ PdLn

O O

Me3Si

H

OAc

O O TL 1986, 27 , 4137

Pd(0), 80°C

MeO2C

CO2Me

Ni(COD)2, 35 °C CO2Me

ACIEE 1985, 24 , 316 TL 1983, 24 , 5847 CO2Me

- α,α'-dihaloketones O OFeLn

O Fe(CO)9

Ar

ACR 1979, 12 , 61

+ Br

Br

Ar

141

FIVE-MEMBERED RING FORMATION ACR 1979, 12 , 396; Organic Reactions, 1988, 36 , 1

- nitrones

O+N

O R1 N

R2

R1

JACS 1964, 86 , 3756

O O

N

N

Me O

N

MeNHOH, EtONa H

O Me

NH

N S

Me

NH

JOC 1984, 49 , 5021

H

H

MeO2C

Bn

OH

Me2N

1) MeI 2) H2, Pd/C

H

H

MeO2C

R3

R2

R3 Nitrone

-

142

Bn N

O-

JACS 1983, 104 , 6460

O S

C 4H 9

C 4H 9

- nitrile oxides O

O

+ O N

N

N O

ArNCO

CO2Et

CO2Et O

O N

O

O

O N

O

O

OTHP

+ _ N O

THPO

N O

OH OTHP

Bu3Sn (80%)

OTHP H2, Raney Ni, H2 O, MeOH, B(OMe)3

(88%)

1) TBS-Cl, DMF, imidazole 2) nBuLi, THF

OH

O O

OH O

HF•pyridine, CH3 CN, pyridine

3) Swern (48%) TBSO

O

(88%)

O

1) PPTS, EtOH, ↑↓ 2) (CH3 )2C(OMe)2 , PPTS

O O

(89%)

OH

Ph

(99%)

N

1) Swern 2) NH2OH•HCl AcONa, MeOH

OTHP

(60%)

OTHP NaOCl, H2 O, CH2 Cl2

CO2Et

1) DHP, PPTS CH2 Cl2, ↑↓ 2) LAH

CH3

H

Ph

JACS 1992, 104 , 4023

CO2Et

OH

Bu 2BOTf, iPr 2EtN, CH2 Cl2, -78°C

OH

O

H2

Cassiol TL 1996, 37, 9292

(73%) O

TBSO O

HO

OH OH

FIVE-MEMBERED RING FORMATION OTBS

OTBS

OTBS LAH

N + O _

TL 1993, 34, 3017

O N

OH

NH2

Arene -Olefin Photocyclization Organic Photochemistry 1989, 10 , 357 - the photochemistry of benzene is dominated by the singlet state 1

*



*

*



OAc OAc

* Me2CuLi

O

O

JACS 1982, 104 , 5805 JCSCC, 1986 247

Me

1) FVP 2) H2, Pd/C

* * hν +

isocume

+

* Tetrahedron 1981,37 , 4445

*

*

*



OMe

OMe

JACS 1981, 103 , 688 OMe cedrane

AcO



AcO

*

*

HO H+

H TL 1982, 23 , 3983 TL 1983, 24 , 5325

143

FIVE-MEMBERED RING FORMATION

144

Intramolecular Photochemical [2+2] "Rule of Five" O

O hν O

O

O

O

JOC 1975, 40 , 2702 JOC 1979, 44 , 1380

hν O

O

Nazarov Cyclization review: Synthesis 1983, 429 - cyclization of allyl vinyl or divinyl ketones

Organic Reaction 1994, 45, 1 O

O H3PO4/HCO2H

O

O H3PO4/HCO2H

- 1,4-hydroxy-acetylenes

HO

H2SO4, MeOH OH

H

O

OH

JOC 1989, 54 , 3449

H OH

- Silicon-Directed Nazarov O

O Tetreahedron 1986, 42 , 2821

FeCl3 SiMe3 Me

- Tin -directed Nazarov

Me

TL 1986, 27 , 5947

Radical Cyclization B. Giese Radicals in Organic Synthesis: Formation of Carbon-Carbon Bonds (Pergamon Press; NY) 1986; Bull. Soc. Chim. Fr. 1990, 127 , 675; Tetrahedron 1981, 37 , 3073; Tetrahedron 1987, 43, 3541; Advances in Free Radical Chemistry 1990, 1, 121. Organic Reactions 1996, 48, 301-856. Radical Addition to multiple bonds: 1. Free radical addition is a two stage process involving an addition step followed by an atom transfer step. 2. In general, the preferred regioselectivity of the addition is in a manor to give the most stable radical (thermodynamic control)

FIVE-MEMBERED RING FORMATION Advantages of free radical reactions: 1. non-polar, little or no solvent effect 2. highly reactive- good for hindered or strained sysntems 3. insensitive to acidic protons in the substrates (i.e. hydroxyl groups do not necessarily need to be protected Mechanism of radical chain reactions 1. initiation 2. propagation 3. termination (bad) Formation of carbon centered radicals: tin hydride reduction of alkyl, vinyl and aryl halides, alcohol derivatives: xanthates, thionocarbonate, thiocarbonylimidazolides organosselenium & boron compounds carboxylic acid derivatives (Barton esters) reduction of organomercurials thermolysis of organolead compounds thermolysis or photolysis of azoalkanes. Radical Ring Closure For irreversible ring closure reaction, the kinetic product will predominate. Both the 5-exo-trig and 6-endo trip are favored reactions, with the 6 exo-trig mode producing the most stable radical. However, the 5-exo-trig is about 50 time faster •

• k1,5



k1,6

thermodynamically favored

kinetically favored

• •

+



100 : 0 •



+ •





100 : 0 •

3-exo-trig vs 4-endo-trig 4-exo-trig vs 5-endo-trig 5-exo-trig vs 6-endo-trig

+ 98:2 • •



+

6-exo-trig vs 7-endo-trig

85:15 • •



+ 100 : 0

7-exo-trig vs 8-endo-trig

145

FIVE-MEMBERED RING FORMATION •

146



and

radicals open up fast and are not synthetically useful; often used as probes for radical reaction

Effects of substituent on the regiochemistry of the 5-hexenyl radical cyclization • •



+ 48:1 •





+ >200:1 • •



+ 2:3 • •



+ < 1:100

Stereochemistry of 5-hexenyl radical cyclization 1-, or 3-substitued 5-hexenyl radicals give cis disubstituted cyclopentanes 2-, or 4-substituted 5-hexenyl radicals give trans disubstitued cyclopentanes 2 Br

Bu3SnH

H R



R

H

R

+

R

65 : 35 prefered transition state R • 3

R

R

Br

R

+

H H

75 : 25

H 4 Br R

H R 1

Bu3SnH Br

+

R



R

R •

83 : 17

R

+ H R

73 : 27

R

FIVE-MEMBERED RING FORMATION Br

OH

nBuSnH, AIBN

147

JACS 1983, 105 , 3720 OH CN

CN

Bu3SnH, AIBN CO2Me

JACS 1990, 112, 5601

Br

CO2Me

Br

OH

Si O

H Si O

Bu3SnH

THPO

H

H2O2

THPO

OH

THPO

multiple cyclizations: D. Curran Advances in Free Radical Chemistry 1990, 1, 121. OTBS

O

OAc

1) NaBH4, CeCl3 2) Ac2O, Et3N

1) LDA, THF 2) TBSCl

O

70 °C

O OTBS

PhSe H2O2

O

PhSeCl, CH2Cl2

O

O THPO

O

1) PPTS, EtOH 2) LAH CO2H

I

CuSMe2 Li+

THPO

I

1) Me3Si Li (1.0 equiv) I

3) (CF3SO2)2O pyridine 4) Bu4NI, PhH

2) CsF

Me H



Bu3SnH, AIBN PhH, reflux

JACS 1985, 107 , 1148 H H (±)-hirsutene

O O

CH3MgBr, CuBr•SMe2

HO

O

O

1) I2 2) DBU

O

O

O

MgBr

CuBr•SMe2, THF O O

HO

O

1) LAH 2) CH3SO2Cl 3) NaI

CO2Me

1) CH3MgBr (excess)

Li 4) 5) CrO3, H2SO4, H2O Bu3SnH, AIBN PhH, reflux

Br

2) Me3SiBr H H

• H 3C

H

(±)-capnellene Tetrahedron Lett. 1985, 41, 3943

FIVE-MEMBERED RING FORMATION O

O

Me

nBuSnH, AIBN

O

148

O JACS 1986, 108, 1106 Me

Me Me

Br

OH H

CHO

O

JACS 1988, 110 , 5064

O

SmI2, THF

O

O H

radical trapping OEt I

OEt

OEt

O O

nBuSnH, AIBN RO

O

tBuNC

JACS 1986, 108 , 6384



RO

CN

RO

can also be trapped with acrylate esters or acrylonitrile. OEt

OEt I O

O

nBuSnH, AIBN

O Me3Si

OEt C5H11

OEt

O SiMe3 C5H11

• RO

RO

RO

1) (S)-BINAL-H 2) HCl, H2O, THF

O

C5H11 RO

1) Wittig 2) deprotect

CHO

RO

CO2H

HO

OH

(+)-PGF2α

O

Paulson-Khand Reaction

Tetrahedron 1985, 41 , 5855; Organic Reactions 1991, 40 , 1. O

O R

R''

R'

R

R

R''

Organometallics 1982, 1 , 1560

+

Co2(CO)6 R'

R'

R''

O JOC 1992, 57 , 5277

Co2(CO)8, NMO O

O

O

O

OTMS

HO

C5H11

C5H11 RO

HO

Brook rearrangement

O

OEt Pd(OAc)2, CH3 CN

O

140°

FIVE-MEMBERED RING FORMATION Ph

Cp2TiCl2, EtMgBr, CO

EtO2C CO2Et

CO2Et

Ph

Ring-Closing Metathesis

CO2Et

Tetrahedron 1998, 54, 4413, Acc. Chem. Res. 1995, 25, 446.

O

O N

O

OH

Bu2BOTf, CH2Cl 2, -78°C

P(C6H11)3 Ph Cl Ru Cl P(C6H11)3

O

O N

O

(97%)

CHO (82 %)

Ph

Ph OH

JOC 1992, 57 , 5803

O

O

O N

LiBH4, THF, MeOH

OH

J. Org. Chem. 1996, 61, 4192

O OH

(78%) Ph

Diazoketones

Tetrahedron 1981, 37 , 2407; Organic Reactions 1979, 26 , 361 O R1

O N2

BF3

R1 TL 1975, 4225

R2

R2

FVP of Acetylenic Ketones O

O

R1 R2

FVP H

R3

R1 R2

O R3

H ••

R1 R2

R3

TL 1986, 27 , 19

149

6-MEMBERED RING FORMATION Six Membered Rings 1. 2. 3. 4. 5.

Diels-Alder Reaction o-Quinodimethanes Intramolecular ene reaction Cation olefin cyclizations Robinson annulation

Diels-Alder Reaction ACIEE 1984, 23 , 876; ACIEE 1977, 16 , 10; Organic Reactions 1984, 32 , 1 W. Carruthers Cycloadditions Reactions in Organic Synthesis (Pergamon Press, Oxford) 1990 - reaction of a 1,3-diene with an olefin to give a cyclohexene. - thermal symmetry allowed pericyclic reaction - diene must reaction is an s-cis conformation - highly stereocontrolled process- geometry of starting material is preserved in the product - possible control of 4 contiguous stereocenters in one step B

B A A

X

B

A

Y

X

Y

Y Y X

+ X

Y X X

+ Y

B A

B

A

B A

- Alder Endo Rule: In order to maximize secondary orbital interactions, the endo TS is favored in the D-A rxn. Tetrahedron 1983, 39, 2095 X X

B

Y Y B A A

A

X Y Y X

B

B A

O O

exo

O

O

minor

O

H H O

H

endo

O

H

O

major

O O O O

Orientation Rules X

X

+

Y

X Y

+ Y

major

minor

150

6-MEMBERED RING FORMATION X

Y

+

X

+

X

151

Y

Y major

minor

- when both the diene and dienophile are "unactivated" the D-A rxn is sluggish - D-A rxns with electron rich dienes and electron defficient dienophiles work the best. Some electron deficient dienophiles are quinones, maleic ahydride, nitroalkenes, α,βunsaturated ketones, esters and nitriles. - D-A rxns with electron deficient dienes and electron rich dienophiles also work well. These are refered to as reverse demand D-A rxns. - D-A rxns are sensitive to steric effects of the dienephiles, particularly at the 1- and 2postions. Steric bulk at the 1-position may prevent approach of the dienophile while steric bulk at the 2-position may prevent the diene from adopting the s-cis conformation. - The D-A rxn is promoted by Lewis acids (TiCl4, BF3 AlCl3, AlEt2Cl, SnCl4,...) - The D-A rxn is promoted by high pressure (1 kbar ~ 14200 psi) Synthesis 1985, 1. O

OMe

OMe

5 Kbar

Me

Me O

+

JACS 1986, 108 , 3040

O O

H

NHBOC 20 kbars, 50°C

H

+

Eu(fod)3

O

OMe

O

NHBOC

H +

O

OMe

NHBOC Synthesis 1986, 928

OMe

- The D-A rxn is usually insenstive to solvent effects, except for water. ACR 1991, 24 , 159 O isooctane +

O

+

krel=1 O endo

H exo 4:1

H2O

JACS 1980, 102 , 7816 TL 1983, 24 , 1901 TL 1984 , 25 , 1239

20 : 1

krel= 700

CO2- Na+ CHO

OHC

CO2H

CO2H

H

H

+ OHC

H2O (70%)

(15%)

JOC 1984, 49 , 5257 JOC 1985, 50 , 1309 Tetrahedron 1986, 42 , 2847

6-MEMBERED RING FORMATION

152

- The mechanism of the D-A rxn is believed to be a one-step, concerted, non-synchronous process. - concerted- bond making and bond breaking processes take place in a single kinetic step (no dip in the transition state) - synchronous- bond making and bond breaking take place at the same time and to the same extent. M.J.S. Dewar JACS 1984, 104 , 203, 209.

+

- study of secondary D-isotope effects have indicated a highly symmetrical T.S. D H

D H

k1

D

D

H

H

+

D

JACS 1972, 94 , 1168

D

D D

k2

D

D

D

D

+

Diels Alder Reactions: O

O PhH, reflux

O O

H

MeO2C

H

CHO

MeO2C

CO2H

OAc OMe

HO2C MeO Tetrahedron 1958, 2 , 1

N

N

H H O2CAr

MeO2C reserpine

H

CHO

H

+ NHCO2Bn

OMe

H

O

O trans

trans

H

CHO

BnO2CHN

BnO2CHN

NHCO2Bn

JACS 1978, 100 , 5179 N H pumiliotoxin C

CH3 H

6-MEMBERED RING FORMATION OMe

MeO O

R

+

O

O R

MeO2C

CO2Me

O

MeO2C

R

O MeO2C

PhS

SPh

PhS

SPh MeO

OMe O

O JACS 1986, 108 , 5908

O

Compactin

OMe

OMe R

R

R

+ Me3SiO

ACR 1981, 14 , 400

O

Me3Si-O

Danishefsky's Diene

OMe MeO2C

OH

O

CO2Me

H

O

+

H

Me3SiO

O

O H

Vernolepin

O

Hetero Diels-Alder Reactions - Heterodienophiles OMe CO2Bn N

+

PhH, reflux O

EtO2C

Me3SiO

N

CO2Bn

JACS 1982, 102 , 1428

CO2Et

O CO2Me

Me3SiO

+

HO

N

N H

TsN

Ts

TL 1987, 28 , 813

HO

PhH, 5°C

OH

CO2Me

CH(OMe)2

CH(OMe)2 O +

CH3

N

O CO2Bn

N CH3

OAc AcO

CO2Bn

AcO

OH NAc CH3

TL 1985 27 , 4727

153

6-MEMBERED RING FORMATION

Me

Me

Me

Me

O +

S

Me

O

S

Me

NTs

Me

Me

Me

S Ph

S

S

Ph

Ph

H

Ph

1) MgBr2

O

2) H3O+

OMe Me3SiO

Tetrahedron, 1983, 39 , 1487

S

O

+

TL 1983, 24 , 987

Me

Me

NTs

NHTs

Me

O

H OBn

JACS 1985, 107 , 1256

O OBn

OMe

+

Ar

H

Yb(fod)3

Me

Me3SiO

OMe

OMe

O

Me

HF, Pyridine

O

O

Ar

Me3SiO Me

JACS 1985, 107 , 6647

Me Ar

O Me

Me

O

Me

C3F7

M O

O C3F7

M

O 3

3

M(fod)3

M(hfc)3

- Heterodienes O

H

O

O

+

AcO

OAc

Me

O

PhS OAc

+

OEt

O

O

H O ACIEE 1983, 22 , 887

+ MeO2C

MeO2C

H

EtO

endo:exo 10 : 1

O

H

endo: exo 1:2

Me

MeO2C AcO

HO

H

O

OH

SPh OAc

Me

OH

ACR 1986, 19 , 250

154

6-MEMBERED RING FORMATION

155

OR' N

O

N

O

+

RO

O

O

RO RO

RO

N OR

OR

OR'

OR'

O

O

RO

N

OMe

RO

JACS 1987, 109 , 285

NHAc OR

CO2R'

H JOC 1985, 50 , 2719



AcO

N

N

N

O

O

O

Intramolecular Diels-Alder Reactions - Type I IDA rxns

(IDA)

Fused bicyclc

Bridged Bicyclic

- Generally, for E-dienes, the fused product is observed unless the connecting chain is very long. For Z-dienes, either the fused or bicyclic products are possible. - Type II IDA rxns: gives bridgehead olefin

395°C

JACS 1982, 104 , 5708, 5715 O

O O

O

O

O

185°C EtO2C

EtO2C

JACS 1987, 109 , 447

O EtO2C

CO2Et AcO

NHBz O 155°C

Ph

O

OH ACIEE 1983, 24 , 419

O OH

O

O

H OH BzO AcO Taxol

O

6-MEMBERED RING FORMATION

156

- IDA reactions to give fused 6•5 (hydroindene) and 6•6 (hydronaphthalene) ring systems are usually favorable reactions. R'

R'

R (CH2)n

R

n= 1 or 2

(CH2)n

- Intramolecular D-A rxns that give medium sized rings (7,8,9, 10) are much less favorable. - Intramolecular D-A rxn which form large rings are often favorable reactions with the diene and olefin portions act as if they were seperate molecules H H

O

O

O +

H

H

O

O

O

endo

+

JACS 1980, 102 , 1390

H

O

O

O

O

O

H

O

O

O

O O

exo

6.2 : 6.8 : 1 (77% combined yield)

- Preference for endo or exo transition state depends on the substituition of the diene, dieneophile and connecting chain. - For intramolecular D-A rxns, geometric constraints can now reverse the normal regiochemistry of the addition as compared to the intermolecular rxn. + CO2R'

CO2R'

R

R CO2R'

CO2R'

- for intramolecular D-A reactions, we will use endo and exo to described the disposition of the connecting chain R

R'

R'

R

H (CH2)n

R' H

(CH2)n

R (CH2)n

endo R

R'

R'

R

H (CH2)n

(CH2)n exo

H

6-MEMBERED RING FORMATION

157

- Lewis acids can greatly effect the endo/exo ratio of IDA reactions especially when the olefin portion is E. The effects for Z-olefins is much more subtle MeO2C MeO2C

MeO2C

H

H

+

JACS 1982, 104 , 2269 H

H 75 : 25

150°C

(75% combined yield)

100 : 0

(RO)2AlCl2, rt

CO2Me

(72% combined yield)

MeO2C

MeO2C

H

H

+ H

H 75 : 25

180°C

63 : 37

EtAlCl2, rt

(74% combined yield) (60% combined yield)

- the effect of substituents on the connectinng chain can influence the stereochemical course of the IDA reaction HN O HN

EtAlCl2 O

JOC 1981, 46 , 1509

H

MeO2C

rt

MeO2C

H

R O

R R

R O H

H

Intramolecular Diels-Alder Reactions: CO2Me MeO2C 130°C TBSO

H

TBSO

JOC 1981, 46 , 1506

H

O

O O

150°C

O

TL 1973, 4477

6-MEMBERED RING FORMATION OTBS

OTBS CF3CO2H

H

JACS 1981, 103 , 4948

H H

H O

TBSO

OTBS

CO2Me

H

CO2Me JOC 1982, 47 , 180

EtAlCl2 H

Asymmetric Diels-Alder Reactions - Chiral Auxillaries Chem. Rev. 1992, 92 , 953; Tetrahedron 1987, 43 , 1969 OAc O

OAc O

M O

+

HO

OH

BF3•OEt2, -40°C H

CO2H

HO

H

OH

OAc

OAc

(-)- Shikimic Acid

>98% d.e.

O O

O

TiCl4, 0°C

+

JOC 1983, 48, 1137 JOC 1983, 43, 4441 ACIEE 1985, 24, 1

TL 1984, 25 , 2191

O R*

O

O

(97:3)

X

O

∆ = 4.5 %ee TiCl4= 78% ee iBu2AlCl= 90% ee

CO2R*

O O

CO2R*

O O Ph O

O

O Ph +

O +

OR* O

Ph

Ph

O

JOC 1989, 54 , 2209

OR*

18 : 82 w/ BF3•OEt2

O Ar O

O

Et2AlCl

+ OTMS

d.e > 95%

92 : 8

R*

CH2Cl2, -80°C

Chem Lett. 1989, 2149

O

up to 70% d.e.

O Ph H O Me

O OR* +

Eu(hfc)3 OTMS

Ph O

PhCHO

O

Me

8 : 92

OR* O

Me

JACS 1986, 108, 7060

158

6-MEMBERED RING FORMATION CH3

O Ph O

O R*

SnCl4, -78C

N +

O

O N

O

O S

S O

R

O

R*

PhMgBr

R 97% d.e.

159

R*

N H O S Ph

O

OH N

R

R

JCSCC , 1985 , 1449 TL 1986, 27 , 1853

O O H

(iPrO)2TiCl2, CH2Cl2, 0°C

O

H O SO2 N

Tetrahedron 1987, 43 , 1969

H CO2R*

O TL 1989, 30 , 6963

endo/exo= 86:4 98% de

X EtAlCl2, -78°C

Oppolzer Auxillary O

O Evan's auxillaries

N

R

O

O O

N

R Me

O

Ph

+ O

O

CO2R*

Me2AlCl N

O

CO2R*

endo (major)

endo CO2R*

CO2R*+ exo endo/exo endo A/ endoB k(rel)

O

O

+ O

O

Me2AlCl N

O

2 equiv. Me2AlCl 60:1 20:1 100

_ Me2AlCl2

_ Me2ClAl

exo

1 equiv Me2AlCl 20:1 4:1 1

N

O

Me2AlCl

+ Al O O N

O O

H

N

Al O O

6-MEMBERED RING FORMATION

O

O N

R

O O

OH

R* EtAlCl2, -100°C

Ph

O

O

O

R* N

Ph

R* H

Me2AlCl

H + TL 1984 , 25 , 4071 JACS 1988, 110 , 1238

Me H O

O

JACS 1984, 106, 4261 JACS 1988, 110 , 1238

(R)-(+)-α-terpineol

O O

160

H 15 : 85

O N Me2AlCl

Ph

95 : 5

- Chiral Dienes O OMe O

Ph

O CHO

OHC

OMe

BF3•OEt2, -20°C BF3•OEt2, -78°C

Ph

4:1 94:6

O

O

OH

OMe O

π-stacking arrangement

MeO H

Ph

O

+

O Ph

O OMe

O

H

O

OH

O O

H

approach of dienophile

O only product

- Chiral Catalysts Chem. Rev. 1992, 92 , 1007; Synthesis 1991, 1; OPPI 1994, 26, 129158 OH

CHO

CHO + EtAlCl2, -78°C

JCSCC 1979, 437

72% (ee)

OtBu Me +

Eu(hfc)3, -10°C

O O

TMSO Me

TL 1983, 24 , 3451

PhCHO Ph Me

6-MEMBERED RING FORMATION

161

Ph Ph Ph O O

O

OH OH

Me O

Ph Ph N

+

O

CL 1986, 1967

N

O

(iPrO)2TiCl2

O O O

O

Ph Ph O

O N

Ph O

OH

O

Me O

OH

H

O

Ph Ph

O

70% yield 95% ee

O

(iPrO)2TiCl2 O

N

O

(30 mol % catalyst)

H

Ph O

O OH OH +

OH

O

JACS 1986, 108 , 3510

Ph BH3, -78°C

OAc

OH

O

OAc

(98%)

Ts N Et B Br

O

CHO

O

N H

CHO

+ -78°C

Br

JACS 1992, 114 , 8290

(> 99% ee)

H N O Br

CHO

O N B Bu Ts

+

CHO Br

CH2Cl2, -78 °C, 16 hrs

Br

Br

O

(81%, 99% ee)

OC O

H N

O B O N Ts

OH

HO

approach of diene

CO2H Gibberellic Acid

Br

Br

H

H N OTBS

H 3C H

CHO +

O O

O

1) NaBH4 2) DDQ

OTBS

O O N B H Ts

CH2Cl2, PhCH3 -78 °C, 42 hrs

3) F

O JACS 1994, 116, 3611

-

4) HCl CHO (83%, 97% ee) O

O

OH

HO Cassiol

OH

6-MEMBERED RING FORMATION

162

Ketene Equivalents in the D-A reaction - ketenes undergo thermal [2+2] cycloaddition with dienes to give vinyl cyclobutanones. - 2-chloroacrylonitrile as a ketene equiv. for D-A rxns. AcO

+

Cl

Cl

CN AcO

ortho-Quinodimathanes

CN

KOH, tBuOH 70 °C

R R R

o-quinodimethane

benzocyclobutane

1)

Nature (London) 1994, 367, 630 ACIEE 1995, 34, 2079

Synthesis 1978, 793; Tetrahedron 1987, 43 , 2873 R

O

O

HO

O

OTMS

LiNH2, NH3, THF

MgBr

CuI, TMS-Cl

Me3Si

SiMe3

CpCo(CO)2

I

O

O

O 170 °C Me3Si Me3Si

H

H

H

Me3Si

Me3Si O

1) CF3CO2H, CCl4, -30°C 2) Pb(O2CCF3)4

H H

ACIEE 1984, 23 , 539 JACS 1977, 99 , 5483 JACS 1979, 101 , 215 JACS 1980, 102 , 241

H

HO Estrone

O O HN HN

JACS 1971, 93 , 3836

155 °C H

O

Me N

O

Me N

155 °C

ACIEE 1971, 11 , 1031

O

O

Me O

O

N

Me N

155 °C N

O

N

ACIEE 1971, 11 , 1031 N

OMe

Me

O

OMe Me

N ACIEE 1971, 11 , 1031

180 °C

N

H

Me3Si

Me3Si

NH

H

6-MEMBERED RING FORMATION Photoenolization

Tetrahedron 1976, 22, 405

R O

R •



H

R O•

OH

R MeO2C

OH

CO2Me

CO2Me

H CO2Me

R

R

R O

R O

O

-H2O R

OH O

R

CO2Me O CO2Me O

R

Intramolecular Ene Reactions

R

ACIEE 1984, 23 , 876, Synthesis 1991, 1 H SnCl4 CHO

JOC 1985, 50, 4144

OH

H

JACS 1991, 113 , 2071

Me2AlCl BnO

H

OH

CHO OBn

Binaphthol

Tetrahedron Lett. 1985, 26, 5535

CHO

OH

Me2Zn

90% ee)

Polyene Cyclization +

Terpene Biosynthesis terpenes sesquiterpenes diterpenes steroids

C 10 C 15 C 20 C 30

+

geraniol farnesol geranylgeraniol squalene

- isoprene- basic building block OPP isopentyl-PP

isoprene unit

OPP OPP OPP

geraniol-PP (C10)

Farnesyl-PP (C15)

geranylgeraniol-PP (C20)

squalene (C30)

163

6-MEMBERED RING FORMATION

164

O HO2C

α-Cedrane

Camphor

Abietic Acid

Biosynthesis of camphor:

+ OPP

O +

Biosynthesis of cedrane: OPP

H

+

+

H + +

Stork-Eschenmoser Hypothesis- Olefin Geometry is preserved in the cyclization reaction, i.e. trans olefin leads to a trans fused ring jucntion A. Eschenmoser HCA 1955, 38, 1890; G. Stork JACS 1955, 77, 5068 Me

Me + Me

Me

R H

Me

Me Me

Me

R

H H

H

Biosynthesis of Abietic acid: OPP

OPP

+

OPP

H

+

H

H

H +

H + H H

H H

H

HO2C

H

6-MEMBERED RING FORMATION

165

-Steroid Biosynthesis: H+

H squalene cyclase

squalene epoxidase

squalene

H

+

H

H HO

+O

H HO

H

H

Protosterol

squalene oxide

H

H

+ H H

H

H H

H HO

H HO

H

H

HO

H Lanosterol

Cholesterol

- Polyene cyclization in synthesis ACR 1968, 1, 1; Bioorg. Chem. 1976, 5, 51; Asymmetric Synthesis 1984, 3, 341-409; ACIEE 1976, 15, 9 SnCl4 CH3NO2, 0°C

H

E.E. van Tamelen JACS 1972, 94 , 8229

(8%) H HO

O

H

δ- Amyrin

OH CHO

SnCl4 PhH, rt

JACS 1974, 96 , 3333

H

(38%) MeO

MeO

H

SnCl4 pentane H

(27%) O

O HO CO2Me OPO(OEt)2

O

W.S. Johnson JACS 1974, 96, 3979 H

H

CO2Me

1) Hg(O2CCF3)2 2) NaCl

O ClHg H

JACS 1980, 102 , 7612

6-MEMBERED RING FORMATION Robinson Annulation

166

Synthesis 1976, 777; Tetrahedron 1976, 32, 3. O

acid or base (thermodynamic conditions)

O

O

- unfavorable equillibium for the Michael addition under kinetic conditions O

base (kinetic conditions)

-

O

-O

O

- stabilizing the resulting enolate of the Michael Addition product can shift the equilibrium as in the case of the vinyl silane shown below O Me3Si

Me3Si

-

O

-

O

O

O

base (kinetic conditions)

OR

OR

OR

a) MeLi

Me3SiO

O

H

b) Me3Si

O

O

JACS 1974, 96 , 6181

O H

O O

O

c) MeONa

- Methyl Vinyl Ketone equivalents I +

mCPBA -

O

O

Me3Si

Me3Si

Me3Si

Aldol O

O

O

H+

O O

JACS 1974, 96 , 3862

I +

O O

O -

O

O

O

O

6-MEMBERED RING FORMATION Intramolecular Aldol Condensation of 1,5-Diketones 6-exo-trig; favored process O O

R'

R'

base - H2O R

R

O

- DeMayo reaction to 1,5-diketones O

O

O O

O

H

DIBAL-H



O

O H 3C

Intramolecular Alkylations (SN2 reaction) Radical Cyclizations Acyloin Reaction Birch Reduction

Organic Reactions 1992, 42, 1.

Aromatic Substitution

(Carey & Sundberg, Chapter 11)

Intramolecular Wittig Reaction Sigmatropic Rearrangements

O

167

7-MEMBERED RING FORMATION

168

Medium Sized Rings 7-Membered Rings [4+2] cycloadditions - [4+2] cycloadditions between dienes and allylcations leads to cycloheptadienes review: ACIEE 1984, 23 , 1; ACIEE 1973, 12 , 819 +

+

+

I

+

CF3CO2Ag

ACIEE 1973, 12, 819 ACIEE 1984, 23, 1

-78°C Me

ZnCl2, -30°C

ACIEE 1982, 21, 442

+ +

H

H

F3CCO2 SiMe3

SiMe3

O

O OTMS +

JACS 1982, 104 , 1330

ZnCl2 Br

O

OMe O O

+

O

O

O

+

O

O

O

O O

JACS 1979, 101, 226

O

mCPBA O HO

- Noyori [4+2] cycloaddition of α,α'-dibromoketones and dienes review: ACR 1979, 12 , 61 O R

R Br

Br

Fe2(CO)9 -orZn-Cu

O

OM R

R R

+

R

ACR 1979, 61 Organic Reactions 1983, 29, 163

7-MEMBERED RING FORMATION

O

O

Br

Fe2(CO)9

Br Br

O

Br

Zn

Br O

O

Br

169

O

CO2Me O

N

Br

Br Br

MeO2C

1)

N

JACS 1978, 100 , 1786 Tetrahedron 1985, 41, 5879

Fe2(CO)9 2) Zn

Br

O

O

OMe O O

O

+

O

O

+

O

O

O

O O

O

mCPBA

JACS 1979, 101, 226

O HO

O

O Me

Me Br

Br

1) H2, Pd/C 2) CF3CO3H

O

1) Fe2(CO)9

OH

O O

O

H

O

JACS 1972, 94, 3940 JOC 1976, 41, 2075 H

O

O

O

1) CF3CO3H 2) LDA, MeI

O O

Me

O

Me

CO2H

Me OH JCSCC 1985, 55

O Me OBn

OBn

OH

OH

OBn

O

- [4+2] cycloaddition between pentadienyl cations and olefins +

+

O HO

OH ∆

O

-

O +

O

+

O O

Tetrahedron 1966, 22, 2387 JOC 1987, 52, 759

7-MEMBERED RING FORMATION O

MeO OMe OMe

SnCl4 OMe

O

O

JACS 1977, 99, 8073 JACS 1979, 101, 6767 JACS 1981, 103, 2718

Seven-Membered Rings from Funnctionalization of Tropone Organic Reactions 1997, 49, 331-425 O-

O

O

Cl

R

R

JOC 1988, 53, 4596 JACS 1987, 109, 3147

O

- [6+4] cycloadditions of tropones with dienes [6+4]

O

JACS 1986, 108, 4655 JOC 1986, 51, 2400

150°C

O

(88%)

O

HO HO HO

OH

Ingenol

- [4+2] cycloaddition between tropone and olefins O

O [4+2] 150°C (81%)

Radical Ring Expansion Reactions - one carbon ring expansions O

O CO2Me (CH2)n

n = 1,2,3

NaH, CH2Br2

Br CO2Me

nBu3SnH

CO2Me (CH2)n

(CH2)n

• CO2Me

(CH2)n

(CH2)n O

•O

O

nBu3SnH, AIBN

• CO2Me

O JACS 1987,109, 3493 JACS 1987,109 , 6548 (CH2)n

CO2Me

170

7-MEMBERED RING FORMATION

O

O Bu3SnLi BrCH2SePh

CH3

171

O •

SePh SnBu3

SnBu3 Tetrahedron 1989, 45, 909 Tetrahedron 1991, 47, 6795

O •O +

Bu3Sn•

SnBu3

- more than one carbon expansion O

R

O

X (CH2)n

(CH2)n

SnBu3

Tetrahedron 1989, 45, 909 Tetrahedron 1991, 47, 6795

R

X= I, SePh n= 1,2

Br

Bu3SnH, AIBN, C6H6, reflux

Br

+

JOC 1992, 57, 7163

O

O

Eight-Membered Rings [4+4] Cycloaddition of Dienes

O

review:Tetrahedron 1992, 48 , 5757. Ni(COD)2, Ph3P

MeO2C MeO2C

JACS 1986, 108 , 4678

MeO2C MeO2C

60°C

O

O

O

O

O

O Ni(COD)2, Ph3P

H

60°C

H O Asteriscanolide

Carbonyl Coupling Reactions - Acyloin Reaction CO2Me

O

Na

CO2Me

JACS 1971, 93 , 1673

OH

- McMurry Reaction R

R

CO2Et

Ti (0)

O O

JACS 1988, 110 , 5904

7-MEMBERED RING FORMATION

172

CHO OHC TiCl3, Zn-Cu H

JACS 1986, 108 , 3513

H

Aldol-like Condensations O O

TiCl4

O OTMS

OH

JCSCC 1983, 703

O

SiMe3

SiMe3

(CH2)n

R

Lewis Acid

R O

O

(CH2)n

OMe

O

JACS 1986, 108 , 3516

n= 1,2 Me3Si

Cl SnCl4 OTs O

tBuPh2SiO

Cl

Cl

Me3Si

OTs

O

O

tBuPh2SiO OEt

Laurenyne

TiCl4 O

SiMe3

JACS 1988, 110 , 2248

JOC 1988, 53 , 50 O

Pinocol Rearrangement O

Me3SiO

SnCl4

Me3SiO +

OMe

JACS 1989, 111 , 1514

OMe OMe

H

OMe

Tiffeneu-Demyanov Ring Expansion - one carbon ring expansion for virtually any size ring O

HO 1) Me3SiCN 2) LAH

N2

NH2 HONO

O H

O JOC 1980, 45 , 185

- also see Beckman and Schmidt rearrangements as a one atom ring expansion for the conversion of cyclic ketones to lactams.

7-MEMBERED RING FORMATION

173

DeMayo Reaction O O

O

O



O

O

+ OAc AcO O JCSCC 1984, 1695 O

O

O

O

O

O

hν, pyrex

O

O

CO2Me

H

pTSA, MeOH reflux

JACS 1986, 108 , 6425

O O H

Ring Expansion/Contraction via Sigmatropic Rearrangements - Cope Rearrangement

O

O SMe

TL 1991, 32 , 6969

55°C O

O O

O

- Anion Accelerated Cope KH, 18-C-6 115°C

O

JACS 1989, 111, 8284

OH

O

OCOCH2OH Pleuromutilin

- Claisen Rearrangement O

O O

Tebbe reagent

O

185°C H

TL 1990, 31 , 6799

7-MEMBERED RING FORMATION - Ester Enolate Claisen- 4 carbon ring contractions O 1) LDA, TBS-Cl 2) 110°C

O

CO2H JACS 1982, 104 , 4030 Tetrahedron 1986, 42 , 2831

MOMO

MOMO H O O

1) LDA, TBS-Cl 2) 110°C

JOC 1988, 53 , 4141

H

H CO2TBS

174

SELECTIVITY 1 SELECTIVITY Science 1983, 219 ...

Barium Manganate. BCSJ 1983, 56, 914. Manganese Dioxide. Review: Synthesis 1976, 65, 133. - Selective oxidation of α,β-unsatutrated (allylic, benzylic, ...... Cyclic Sulfites and Sulfates (epoxide equivalents). Synthesis 1992, 1035. R1. R2. OH. OH. SOCl2, Et3N. O. S. O. R1. R2. O sulfite. O. S. O. R1. R2. O sulfate. O.

1MB Sizes 0 Downloads 158 Views

Recommend Documents

Learning a Selectivity-Invariance-Selectivity ... - Semantic Scholar
performed with an estimation method which guarantees consistent (converging) estimates [2]. We introduce the image data next before turning to the model in ...

Learning a Selectivity-Invariance-Selectivity Feature Extraction ...
Since we are interested in modeling spatial features, we removed the DC component from the images and normalized them to unit norm before the learning of the features. We compute the norm of the images after. PCA-based whitening. Unlike the norm befo

ESTIMATION OF FREQUENCY SELECTIVITY FOR ...
Abstract. In this paper, estimation of both global (long term) and local (in- stantaneous) ... file (PDP) parameters corresponding to a specific PDP model is given. ... discussed. 2 System model. An OFDM based system model is used. Time domain sample

Selectivity of amidrazones towards activated nitriles ... - Arkivoc
Amidrazones bearing electron-donating groups in the phenyl group attached to N3 .... we could not detect all the nitrogen atoms; however, the signals detected ...

Anatomical connectivity patterns predict face selectivity in the ... - Nature
Dec 25, 2011 - formally explored, though various frameworks for such an analysis have been suggested2. In the absence of any extra information, can structural connectivity accurately predict the location ..... Note: Supplementary information is avail

pdf-14104\functional-selectivity-of-g-protein-coupled-receptor ...
Download. Connect more apps... Try one of the apps below to open or edit this item. pdf-14104\functional-selectivity-of-g-protein-coupled- ... drug-discovery-the-receptors-from-brand-humana-pre.pdf. pdf-14104\functional-selectivity-of-g-protein-coupl

Peer review delay and selectivity in ecology journals - CiteSeerX
Reviewers are keen to do a prompt review job for these relatively few and well regarded ... their manuscripts if the journal has a good reputation, due to the widespread feeling that it is important to .... IEEE Computer, 23, 46–51. Tobin, M. J. ..

Peer review delay and selectivity in ecology journals
Abstract Peer review is fundamental to science as we know it, but is also a source of delay in getting discoveries communicated to the world. .... different disciplines, and as journals can try to surreptitiously manipulate their impact ... review de

Learning a selectivity–invariance–selectivity feature ...
University of Helsinki - p. 1. Learning a selectivity–invariance–selectivity .... them fixed, learn the parameters of layer three. □ For layer one and two, fit the pdf.

Image statistics underlying natural texture selectivity of ... - PNAS
Dec 17, 2014 - Our daily visual experiences are inevitably linked to recognizing the rich variety of textures. However, how the brain encodes and differentiates a plethora of natural textures remains poorly understood. Here, we show that many neurons

Selectivity of Migrants from Mexico: What does Net ...
census. 1. 1 Department of Economics and Graduate School of International Relations and Pacific Studies, UC San. Diego, respectively. ... selection of migrants, as the better educated face better prospects in Mexico. Ibarraran and ... have far higher

Image statistics underlying natural texture selectivity of ... - PNAS
Dec 17, 2014 - Because the statistics of each image can be com- puted from responses of upstream neurons in ... Email: [email protected]. This article contains supporting information ...... Victor JD, Conte MM (2012) Local image statistics: Maxi

Distinct, genome-wide, gene-specific selectivity patterns of ... - CiteSeerX
Nov 4, 2014 - Yu EJ, Kim S-H, Kim MJ, Seo W-Y, Song K, Kang. M-S, Yang CK, Stallcup MR, Kim JH: SUMOylation of. ZFP282 potentiates its positive effect ...

Selectivity for the Human Body in the Fusiform Gyrus
Aug 4, 2004 - while viewing segments of a James Bond movie, showed that bodies activate the fusiform gyrus near the FFA (Bartels and. Zeki 2004). The stimuli in this study were not controlled, so it is not clear whether bodies, faces, or both were re

Activity, selectivity and stability of Ni–Pt loaded zeolite-ß ...
Further Ni addition above the threshold value leads to fall in activity and ... Zeolite-ß based Ni-Pt catalysts always show higher activity and selectivity than ...

Bond and mode selectivity in the reaction of atomic ...
Jan 8, 2004 - 1033.06. 5. Torsion. 1331.28. 6. CH2 antisymmetric stretch. 3012.26. 7. CH2 rock ... (0,1) transitions,36,37 and the methyl radi- cal products are ...

Activity, selectivity and stability of Ni–Pt loaded zeolite-ß ...
tetraethyl ammonium bromide as template. .... because of the migration of Ni particles during the re- ... 853.3 eV; NiAl2O4: 857.2 eV in Phi ESCA data book).

Contributions of Ih to feature selectivity in layer II ...
Contributions of Ih to feature selectivity in layer II stellate cells of the entorhinal ... this work has concentrated on the link between subthresh- old oscillations ...

Activity, selectivity and stability of Ni–Pt loaded zeolite-ß ...
tent and found that a maximum n-heptane conversion of 39.2wt.% with high selectivity to isomerisation and simultaneous suppression of fuel gas formation.

College Prep 1 Regular - Niles Township High Schools District 219
May 22, 2016 - STATE GOAL 4: Listen and speak effectively in a variety of ... B. Speak effectively using language appropriate to ...... Argentina's “Dirty War.

1983 litigation BJE.pdf
plaintiff's first decision point signifies that the plaintiff cannot distinguish the two branches. when making his decision.) Since his decision cannot depend on the ...

Imsa 1983.pdf
Chevrolet Camaro : 80,16 mm. Chevrolet Corvette : 77,78 mm. Chevrolet Corvette C4 : 76,25 mm. Chevrolet Montecarlo: 81,85 mm. Chevrolet Monza Dekon (S): ...

Dual hindi 1983
Though this bomb raid. iscompletely ineffective, it is ... January radio control.John green pdf. ... 2 german.Dual hindi 1983.Windows 10 pro x64 1511 10586.

Quarterback princess 1983
Page 1 of 23. Night ofthecomet (1984).Bass boost pro.09877979385 - Download Quarterback princess 1983.Xmas 2005.When I got back out ofthe. woods, I found my family- veryworried and panicking. They didn'texpectmeto climb that which forevermoreshall be